Day-627 | Daily MCQs | UPSC Prelims | CURRENT AFFAIRS

Day-627

Time limit: 0

Quiz-summary

0 of 5 questions completed

Questions:

  1. 1
  2. 2
  3. 3
  4. 4
  5. 5

Information

DAILY MCQ

You have already completed the quiz before. Hence you can not start it again.

Quiz is loading...

You must sign in or sign up to start the quiz.

You have to finish following quiz, to start this quiz:

Results

0 of 5 questions answered correctly

Your time:

Time has elapsed

You have reached 0 of 0 points, (0)

Categories

  1. Not categorized 0%
  1. 1
  2. 2
  3. 3
  4. 4
  5. 5
  1. Answered
  2. Review
  1. Question 1 of 5
    1. Question

    1. Consider the following statements:
    1. Snow leopard is the smallest member of genus Panthera.
    2. The World Life Fund for Nature (WWF) along with National Tiger Conservation Agency (NTCA) carried out the first-ever scientific Snow Leopard Population Assessment of India (SPAI) for the years 2019-2023.
    3. According to the SPAI, Uttarakhand and Himachal Pradesh collectively recorded the highest snow leopard population in the country.
    How many of the above statements are correct?

    Correct

    Answer. A
    Explanation:
    Statement 1 is correct: The snow leopard (Panthera uncia with IUCN Status of ‘Vulnerable’) is the smallest member of the genus Panthera of the family Felidae and is the sister species to the tiger (Panthera tigris). It is adapted to montane habitats in Central Asia, including principally the Altai, Tian Shan, Kun Lun, Pamir, Hindu Kush, Karakorum, and Himalayan mountain ranges, where it preys on ungulates, particularly blue sheep (Pseudois spp.), goats and ibex (Capra spp.), marmots (Marmota spp.), and lagomorphs.
    Statement 2 is incorrect: The Snow Leopard Population Assessment in India (SPAI) Program is the first-ever scientific exercise that reports Snow leopard population of 718 individuals in India. The Wildlife Institute of India (WII)is the National Coordinator for this exercise that was carried out with the support of all snow leopard range states and two conservation partners, the Nature Conservation Foundation, Mysuru and WWF-India.
    Statement 3 is incorrect: A total of 241 unique Snow leopards were photographed. Based on data analysis, the estimated population in different states are as follows: Ladakh (477), Uttarakhand (124), Himachal Pradesh (51), Arunachal Pradesh (36), Sikkim (21), and Jammu and Kashmir (9). Therefore, it is clear that even collectively the states of Himachal Pradesh and Uttarakhand recorded lesser snow leopards as compared to the UT of Ladakh alone.

    Incorrect

    Answer. A
    Explanation:
    Statement 1 is correct: The snow leopard (Panthera uncia with IUCN Status of ‘Vulnerable’) is the smallest member of the genus Panthera of the family Felidae and is the sister species to the tiger (Panthera tigris). It is adapted to montane habitats in Central Asia, including principally the Altai, Tian Shan, Kun Lun, Pamir, Hindu Kush, Karakorum, and Himalayan mountain ranges, where it preys on ungulates, particularly blue sheep (Pseudois spp.), goats and ibex (Capra spp.), marmots (Marmota spp.), and lagomorphs.
    Statement 2 is incorrect: The Snow Leopard Population Assessment in India (SPAI) Program is the first-ever scientific exercise that reports Snow leopard population of 718 individuals in India. The Wildlife Institute of India (WII)is the National Coordinator for this exercise that was carried out with the support of all snow leopard range states and two conservation partners, the Nature Conservation Foundation, Mysuru and WWF-India.
    Statement 3 is incorrect: A total of 241 unique Snow leopards were photographed. Based on data analysis, the estimated population in different states are as follows: Ladakh (477), Uttarakhand (124), Himachal Pradesh (51), Arunachal Pradesh (36), Sikkim (21), and Jammu and Kashmir (9). Therefore, it is clear that even collectively the states of Himachal Pradesh and Uttarakhand recorded lesser snow leopards as compared to the UT of Ladakh alone.

  2. Question 2 of 5
    2. Question

    2. Consider the following statements:
    1. Lightning and volcanoes are the natural sources contributing to acid rain.
    2. Acid rain harms plants by inducing soil acidification and nutrient deficiency.
    3. India is the latest member to join EANET, an intergovernmental initiative aimed to develop an understanding of acid deposition problems.
    How many of the above statements are correct?

    Correct

    Answer. B
    Explanation:
    Statement 1 is correct: The combustion of fossil fuels at higher temperature leads to generation of NOx and SO2 emissions which react with moisture in the air to give rise to acid rain. The natural factors that lead to acid rain include lightning, volcanoes, earthquakes and forest fires which release SO2 and NOx into the atmosphere.
    The rapid heating and cooling of the gases within a lightning bolt produces nitric oxide (NO) which combines with Oxygen to form NO2, especially when it is moving downwind.
    Statement 2 is correct: Acid rain harms plants by inducing soil acidification and nutrient deficiency. It triggers leaching of soil nutrients such as aluminium, thereby lowering their fertility. Further, it decreases the pH of the soil causing its acidity to increase, which negatively impacts the overall growth of plants and crops.
    Statement 3 is incorrect: India is not a member of EANET. EANET (Acid deposition and air quality management from data to policy) was established in 2001 as an intergovernmental initiative to create a common understanding on the state of acid deposition problems in East Asia, provide useful inputs for decision making at various levels, and promote cooperation among countries.
    The participating countries in the EANET include Indonesia, Malaysia, Philippines, Vietnam, Cambodia, Laos PDR, Thailand, Myanmar, China, Mongolia, Russia, South Korea and Japan.

    Incorrect

    Answer. B
    Explanation:
    Statement 1 is correct: The combustion of fossil fuels at higher temperature leads to generation of NOx and SO2 emissions which react with moisture in the air to give rise to acid rain. The natural factors that lead to acid rain include lightning, volcanoes, earthquakes and forest fires which release SO2 and NOx into the atmosphere.
    The rapid heating and cooling of the gases within a lightning bolt produces nitric oxide (NO) which combines with Oxygen to form NO2, especially when it is moving downwind.
    Statement 2 is correct: Acid rain harms plants by inducing soil acidification and nutrient deficiency. It triggers leaching of soil nutrients such as aluminium, thereby lowering their fertility. Further, it decreases the pH of the soil causing its acidity to increase, which negatively impacts the overall growth of plants and crops.
    Statement 3 is incorrect: India is not a member of EANET. EANET (Acid deposition and air quality management from data to policy) was established in 2001 as an intergovernmental initiative to create a common understanding on the state of acid deposition problems in East Asia, provide useful inputs for decision making at various levels, and promote cooperation among countries.
    The participating countries in the EANET include Indonesia, Malaysia, Philippines, Vietnam, Cambodia, Laos PDR, Thailand, Myanmar, China, Mongolia, Russia, South Korea and Japan.

  3. Question 3 of 5
    3. Question

    3. Consider the following statements about ferns:
    1. They are the most diverse group of vascular plants on Earth.
    2. They grow naturally in a soilless condition.
    3. They can propagate through their rhizomes and spores.
    4. Eravikulam National Park is the first protected area in India to set up a ‘fernarium’ inside it.
    How many of the above statements are correct?

    Correct

    Answer. C
    Explanation:
    Statement 1 is incorrect: Today, ferns are the second-most diverse group of vascular plants on Earth, outnumbered only by flowering plants. With around 10,500 living species, ferns outnumber the remaining non-flowering vascular plants (the lycophytes and gymnosperms) by a factor of 4 to 1.
    Statement 2 is correct: Ferns are part of the Epiphytic family. They grow naturally in a soilless condition. The plants obtain water and nutrients through leaching from trees.
    Statement 3 is correct: Ferns are the plants which bear no flowers. They can propagate or reproduce both sexually and asexually. They reproduce sexually via spores. Some ferns may reproduce via asexual means through apogamy, the proliferation of baby ferns, and rhizome proliferation.
    Statement 4 is correct: India’s first ‘Fernarium’ was established April 20, 2023 in Eravikulam National Park, known for housing Nilgiri Tahr, in Kerala.

    Incorrect

    Answer. C
    Explanation:
    Statement 1 is incorrect: Today, ferns are the second-most diverse group of vascular plants on Earth, outnumbered only by flowering plants. With around 10,500 living species, ferns outnumber the remaining non-flowering vascular plants (the lycophytes and gymnosperms) by a factor of 4 to 1.
    Statement 2 is correct: Ferns are part of the Epiphytic family. They grow naturally in a soilless condition. The plants obtain water and nutrients through leaching from trees.
    Statement 3 is correct: Ferns are the plants which bear no flowers. They can propagate or reproduce both sexually and asexually. They reproduce sexually via spores. Some ferns may reproduce via asexual means through apogamy, the proliferation of baby ferns, and rhizome proliferation.
    Statement 4 is correct: India’s first ‘Fernarium’ was established April 20, 2023 in Eravikulam National Park, known for housing Nilgiri Tahr, in Kerala.

  4. Question 4 of 5
    4. Question

    4. Consider the following statements about Liquified Natural Gas (LNG):
    1. It comprises mostly methane.
    2. It cannot be used as a transportation fuel in heavy duty vehicles.
    3. The Liquified Natural Gas (LNG) plants emit carbon monoxide, sulphur dioxide, and volatile organic compounds (VOCs).
    How many of the above statements are correct?

    Correct

    Answer. B
    Explanation:
    Statement 1 is correct: Liquefied natural gas (LNG) is natural gas that has been cooled to a liquid state, at about -260° Fahrenheit. It comprises predominantly of methane, CH4, with some mixture of ethane, C2H6, propane, butane and nitrogen. LNG typically contains 85-95% methane.
    Statement 2 is incorrect: LNG can be used as a transportation fuel in medium and heavy commercial vehicles. To adopt LNG for transport application, the Government of India also modified the Central Motor Vehicles Rules, 1989 in 2017, to include LNG as transport fuel.
    Statement 3 is correct: The disadvantage associated with the use of LNG is that LNG plants emit carbon monoxide, sulphur dioxide, and volatile organic compounds (VOCs).

    Incorrect

    Answer. B
    Explanation:
    Statement 1 is correct: Liquefied natural gas (LNG) is natural gas that has been cooled to a liquid state, at about -260° Fahrenheit. It comprises predominantly of methane, CH4, with some mixture of ethane, C2H6, propane, butane and nitrogen. LNG typically contains 85-95% methane.
    Statement 2 is incorrect: LNG can be used as a transportation fuel in medium and heavy commercial vehicles. To adopt LNG for transport application, the Government of India also modified the Central Motor Vehicles Rules, 1989 in 2017, to include LNG as transport fuel.
    Statement 3 is correct: The disadvantage associated with the use of LNG is that LNG plants emit carbon monoxide, sulphur dioxide, and volatile organic compounds (VOCs).

  5. Question 5 of 5
    5. Question

    5. The ‘International Single Species Action Plan’ belongs to which one of the following?

    Correct

    Answer. C
    Explanation: International Single Species Action Plans are the key instrument developed under the Agreement on the Conservation of African-Eurasian Migratory Waterbirds (AEWA) for the purpose of implementing coordinated measures to restore migratory waterbird species to a favourable conservation status.
    The Agreement on the Conservation of African-Eurasian Migratory Waterbirds (AEWA) is an intergovernmental treaty dedicated to the conservation of migratory waterbirds and their habitats across Africa, Europe, Middle East, Central Asia, Greenland and Canadian archipelago. It is developed under the framework of the Conservation on Migratory Species (CMS) and administered by the United Nations Environment Programme (UNEP).

    Incorrect

    Answer. C
    Explanation: International Single Species Action Plans are the key instrument developed under the Agreement on the Conservation of African-Eurasian Migratory Waterbirds (AEWA) for the purpose of implementing coordinated measures to restore migratory waterbird species to a favourable conservation status.
    The Agreement on the Conservation of African-Eurasian Migratory Waterbirds (AEWA) is an intergovernmental treaty dedicated to the conservation of migratory waterbirds and their habitats across Africa, Europe, Middle East, Central Asia, Greenland and Canadian archipelago. It is developed under the framework of the Conservation on Migratory Species (CMS) and administered by the United Nations Environment Programme (UNEP).

window.wpAdvQuizInitList = window.wpAdvQuizInitList || []; window.wpAdvQuizInitList.push({ id: '#wpAdvQuiz_668', init: { quizId: 668, mode: 0, globalPoints: 10, timelimit: 0, resultsGrade: [0], bo: 0, qpp: 0, catPoints: [10], formPos: 0, lbn: "Finish quiz", json: {"3124":{"type":"single","id":3124,"catId":0,"points":2,"correct":[1,0,0,0]},"3125":{"type":"single","id":3125,"catId":0,"points":2,"correct":[0,1,0,0]},"3126":{"type":"single","id":3126,"catId":0,"points":2,"correct":[0,0,1,0]},"3127":{"type":"single","id":3127,"catId":0,"points":2,"correct":[0,1,0,0]},"3128":{"type":"single","id":3128,"catId":0,"points":2,"correct":[0,0,1,0]}} } });




Day-626 | Daily MCQs | UPSC Prelims | SCIENCE AND TECHNOLOGY

Day-626

Time limit: 0

Quiz-summary

0 of 5 questions completed

Questions:

  1. 1
  2. 2
  3. 3
  4. 4
  5. 5

Information

DAILY MCQ

You have already completed the quiz before. Hence you can not start it again.

Quiz is loading...

You must sign in or sign up to start the quiz.

You have to finish following quiz, to start this quiz:

Results

0 of 5 questions answered correctly

Your time:

Time has elapsed

You have reached 0 of 0 points, (0)

Categories

  1. Not categorized 0%
  1. 1
  2. 2
  3. 3
  4. 4
  5. 5
  1. Answered
  2. Review
  1. Question 1 of 5
    1. Question

    1. Consider the following statements:
    Statement I: Autoclaved Aerated Concrete (AAC) blocks are lightweight concrete building materials.
    Statement II: AAC blocks have a high compressive strength and are resistant to fire and pests.
    Which one of the following is correct in respect of the above statements?

    Correct

    Answer: B
    Explanation
    Statement 1 is correct: AAC blocks are lightweight, precast concrete building materials that contain millions of small, evenly distributed air pockets. These air pockets result from the addition of aluminum powder during the manufacturing process, which reacts with lime and cement to form hydrogen gas, creating the aerated structure.
    Statement 2 is correct: AAC blocks possess high compressive strength, making them suitable for load-bearing applications in construction. Additionally, they exhibit excellent fire resistance due to their inorganic composition and are not susceptible to damage from pests such as termites.

    Thus, both statements are correct but there is no correlation as to statements 2 is explaining the statement 1.

    Incorrect

    Answer: B
    Explanation
    Statement 1 is correct: AAC blocks are lightweight, precast concrete building materials that contain millions of small, evenly distributed air pockets. These air pockets result from the addition of aluminum powder during the manufacturing process, which reacts with lime and cement to form hydrogen gas, creating the aerated structure.
    Statement 2 is correct: AAC blocks possess high compressive strength, making them suitable for load-bearing applications in construction. Additionally, they exhibit excellent fire resistance due to their inorganic composition and are not susceptible to damage from pests such as termites.

    Thus, both statements are correct but there is no correlation as to statements 2 is explaining the statement 1.

  2. Question 2 of 5
    2. Question

    2. With reference to the “Deep Web,” consider the following statements:
    1. It refers to the vast portion of the internet that is not indexed by search engines.
    2. Specialized software like Tor is commonly used to access the Deep Web and ensure user anonymity.
    3. It also includes legal content such as private social media profiles and password-protected websites.
    How many of the above statements are correct?

    Correct

    Answer: C
    Explanation
    Statement 1 is correct: The Deep Web consists of web pages and content not indexed by traditional search engines, making it not publicly accessible.
    Statement 2 is correct: Specialized software like Tor is commonly used to access the Deep Web, providing anonymity to users.
    Statement 3 is correct: While the Deep Web does host illicit activities, not all activities are illegal, and it includes legitimate platforms, private databases like social media profiles and password protected websites.
    Deep Web vs Dark Web
    The Deep Web is primarily designed to keep things private, but Deep Web sites typically aren’t anonymous. The Dark Web is designed to be anonymous, which makes it a better fit for illegal activities than the Deep Web.

    Incorrect

    Answer: C
    Explanation
    Statement 1 is correct: The Deep Web consists of web pages and content not indexed by traditional search engines, making it not publicly accessible.
    Statement 2 is correct: Specialized software like Tor is commonly used to access the Deep Web, providing anonymity to users.
    Statement 3 is correct: While the Deep Web does host illicit activities, not all activities are illegal, and it includes legitimate platforms, private databases like social media profiles and password protected websites.
    Deep Web vs Dark Web
    The Deep Web is primarily designed to keep things private, but Deep Web sites typically aren’t anonymous. The Dark Web is designed to be anonymous, which makes it a better fit for illegal activities than the Deep Web.

  3. Question 3 of 5
    3. Question

    3. With reference to bisphenol, consider the following statements:
    1. It is commonly used in the production of thermal paper used for receipts.
    2. It is a type of naturally occurring resin found in certain plant species.
    3. Bisphenol exposure has been associated with potential health risks, including endocrine disruption.
    How many of the above statements are correct?

    Correct

    Answer: B
    Explanation
    Statement 1 is correct: Bisphenol is commonly used in the production of thermal paper, including that used for receipts. The glossy part of the paper is coated with Bisphenol A (BPA) which is heat sensitive and produces text without the use of ink.

    Statement 2 is incorrect: Bisphenol is not a naturally occurring resin; it is a synthetic chemical compound.
    Statement 3 is correct: Exposure to bisphenol has indeed been associated with potential health risks, including endocrine disruption.
    About Bisphenol
    Bisphenol refers to a class of chemical compounds that contain two phenol functional groups. These compounds are characterized by the presence of two hydroxyphenyl groups connected by a bridge, often consisting of carbon or another element. One well-known member of the bisphenol family is Bisphenol A (BPA).
    BPA is commonly used in the production of polycarbonate plastics and epoxy resins. Polycarbonate plastics are used in numerous consumer products, such as water bottles, food storage containers, and eyeglass lenses. Epoxy resins are used as coatings on the inside of food and beverage cans, among other applications.
    It’s important to note that while bisphenols have various industrial applications, concerns have been raised about the potential health effects of exposure to certain bisphenols, particularly BPA. BPA, in particular, has been studied for its potential endocrine-disrupting properties, as it can mimic the action of the hormone estrogen in the body. As a result, there have been efforts to limit BPA exposure in certain products, leading to the development and use of BPA-free alternatives.

    Incorrect

    Answer: B
    Explanation
    Statement 1 is correct: Bisphenol is commonly used in the production of thermal paper, including that used for receipts. The glossy part of the paper is coated with Bisphenol A (BPA) which is heat sensitive and produces text without the use of ink.

    Statement 2 is incorrect: Bisphenol is not a naturally occurring resin; it is a synthetic chemical compound.
    Statement 3 is correct: Exposure to bisphenol has indeed been associated with potential health risks, including endocrine disruption.
    About Bisphenol
    Bisphenol refers to a class of chemical compounds that contain two phenol functional groups. These compounds are characterized by the presence of two hydroxyphenyl groups connected by a bridge, often consisting of carbon or another element. One well-known member of the bisphenol family is Bisphenol A (BPA).
    BPA is commonly used in the production of polycarbonate plastics and epoxy resins. Polycarbonate plastics are used in numerous consumer products, such as water bottles, food storage containers, and eyeglass lenses. Epoxy resins are used as coatings on the inside of food and beverage cans, among other applications.
    It’s important to note that while bisphenols have various industrial applications, concerns have been raised about the potential health effects of exposure to certain bisphenols, particularly BPA. BPA, in particular, has been studied for its potential endocrine-disrupting properties, as it can mimic the action of the hormone estrogen in the body. As a result, there have been efforts to limit BPA exposure in certain products, leading to the development and use of BPA-free alternatives.

  4. Question 4 of 5
    4. Question

    4. Consider the following statements about liposomes:
    1. They are microscopic spheres made from a special type of peptide molecule.
    2. Their hollow structure allows them to encapsulate drugs or other substances for targeted delivery within the body.
    Which of the statements given above is/are correct?

    Correct

    Answer: B
    Explanation
    Statement 1 is incorrect: Liposomes are microscopic spheres made from a special type of fat molecule. These molecules usually form a lipid bilayer (and not peptide), creating a hollow sphere-like structure. Moreover, elasticity of lipid molecules helps in creating smaller sacs which is absent in case of peptide molecules.

    Statement 2 is correct: The hollow structure of liposomes allows them to encapsulate drugs or other substances. This property makes them useful in medicine for targeted drug delivery within the body. The liposomes can carry the therapeutic payload to specific tissues or cells, improving the effectiveness of the treatment while minimizing side effects.

    Incorrect

    Answer: B
    Explanation
    Statement 1 is incorrect: Liposomes are microscopic spheres made from a special type of fat molecule. These molecules usually form a lipid bilayer (and not peptide), creating a hollow sphere-like structure. Moreover, elasticity of lipid molecules helps in creating smaller sacs which is absent in case of peptide molecules.

    Statement 2 is correct: The hollow structure of liposomes allows them to encapsulate drugs or other substances. This property makes them useful in medicine for targeted drug delivery within the body. The liposomes can carry the therapeutic payload to specific tissues or cells, improving the effectiveness of the treatment while minimizing side effects.

  5. Question 5 of 5
    5. Question

    5. Consider the following statements regarding “feathering”, a term recently seen in the news:
    1. It is a technique used in aircraft maneuvering during engine failure or shutdown to reduce drag and increase gliding distance.
    2. It involves changing the pitch or angle of the propeller blades to align them parallel to the airflow.
    Which of the statements given above is/are correct?

    Correct

    Answer: C
    Explanation
    Statement 1 is correct: Feathering is indeed a technique used in aircraft maneuvering during engine failure or shutdown to reduce drag and increase gliding distance. This is achieved by adjusting the pitch or angle of the propeller blades.

    Statement 2 is correct: Feathering involves changing the pitch or angle of the propeller blades to align them parallel to the airflow. This reduces the drag on the propeller, allowing the aircraft to glide more efficiently.

    Incorrect

    Answer: C
    Explanation
    Statement 1 is correct: Feathering is indeed a technique used in aircraft maneuvering during engine failure or shutdown to reduce drag and increase gliding distance. This is achieved by adjusting the pitch or angle of the propeller blades.

    Statement 2 is correct: Feathering involves changing the pitch or angle of the propeller blades to align them parallel to the airflow. This reduces the drag on the propeller, allowing the aircraft to glide more efficiently.

window.wpAdvQuizInitList = window.wpAdvQuizInitList || []; window.wpAdvQuizInitList.push({ id: '#wpAdvQuiz_667', init: { quizId: 667, mode: 0, globalPoints: 10, timelimit: 0, resultsGrade: [0], bo: 0, qpp: 0, catPoints: [10], formPos: 0, lbn: "Finish quiz", json: {"3119":{"type":"single","id":3119,"catId":0,"points":2,"correct":[0,1,0,0]},"3120":{"type":"single","id":3120,"catId":0,"points":2,"correct":[0,0,1,0]},"3121":{"type":"single","id":3121,"catId":0,"points":2,"correct":[0,1,0,0]},"3122":{"type":"single","id":3122,"catId":0,"points":2,"correct":[0,1,0,0]},"3123":{"type":"single","id":3123,"catId":0,"points":2,"correct":[0,0,1,0]}} } });




Day-625 | Daily MCQs | UPSC Prelims | HISTORY

Day-625

Time limit: 0

Quiz-summary

0 of 5 questions completed

Questions:

  1. 1
  2. 2
  3. 3
  4. 4
  5. 5

Information

DAILY MCQ

You have already completed the quiz before. Hence you can not start it again.

Quiz is loading...

You must sign in or sign up to start the quiz.

You have to finish following quiz, to start this quiz:

Results

0 of 5 questions answered correctly

Your time:

Time has elapsed

You have reached 0 of 0 points, (0)

Categories

  1. Not categorized 0%
  1. 1
  2. 2
  3. 3
  4. 4
  5. 5
  1. Answered
  2. Review
  1. Question 1 of 5
    1. Question

    1. With reference to the Nyaya School of Philosophy, consider the following statements:
    1. It believes in the process of logical reasoning to obtain salvation.
    2. It only accepted perception and inference as sources of valid knowledge.
    Which of the statements given above is/are correct?

    Correct

    Answer: A
    Explanation:
    Statement 1 is correct: It believes in the process of logical reasoning to obtain salvation, as the name of the school implies. It is considered as a technique of logical thinking. According to Nyaya Philosophy, nothing is acceptable unless it is in accordance with reason and experience (scientific approach).
    Statement 2 is incorrect: Vaisheshika school of philosophy only accepted perception and inference as sources of valid knowledge. Nyaya accepts four kinds of Pramā (valid presentation): (1) perception, (2) inference, (3) comparison or analogy, (4) word or testimony. Nyāya defends a form of direct realism and a theory of substances (dravya).
    Additional information:
    Nyaya School of Philosophy
    ● Nyaya school is one of the six astika schools of Indian Philosophy. This school’s most significant contributions to Indian philosophy were the systematic development of the theory of logic, methodology, etc.
    ● They regard life, death, and redemption as enigmas that can be deciphered via rational and analytical reasoning.
    ● Gautama, who is also known as the author of the Nyaya Sutra, is said to have developed this school of thought.
    Vaisheshika school of philosophy
    ● The reality according to this philosophy has many bases or categories which are substance, attribute, action, genus, distinct quality and inherence.
    ● Vaisheshika thinkers believe that all objects of the universe are composed of five elements–earth, water, air, fire and ether.
    ● They believe that God is the guiding principle. The living beings were rewarded or punished according to the law of karma, based on actions of merit and demerit.
    ● The Vaisheshika and Nyaya schools eventually merged because of their closely related metaphysical theories (Vaisheshika only accepted perception and inference as sources of valid knowledge).

    Incorrect

    Answer: A
    Explanation:
    Statement 1 is correct: It believes in the process of logical reasoning to obtain salvation, as the name of the school implies. It is considered as a technique of logical thinking. According to Nyaya Philosophy, nothing is acceptable unless it is in accordance with reason and experience (scientific approach).
    Statement 2 is incorrect: Vaisheshika school of philosophy only accepted perception and inference as sources of valid knowledge. Nyaya accepts four kinds of Pramā (valid presentation): (1) perception, (2) inference, (3) comparison or analogy, (4) word or testimony. Nyāya defends a form of direct realism and a theory of substances (dravya).
    Additional information:
    Nyaya School of Philosophy
    ● Nyaya school is one of the six astika schools of Indian Philosophy. This school’s most significant contributions to Indian philosophy were the systematic development of the theory of logic, methodology, etc.
    ● They regard life, death, and redemption as enigmas that can be deciphered via rational and analytical reasoning.
    ● Gautama, who is also known as the author of the Nyaya Sutra, is said to have developed this school of thought.
    Vaisheshika school of philosophy
    ● The reality according to this philosophy has many bases or categories which are substance, attribute, action, genus, distinct quality and inherence.
    ● Vaisheshika thinkers believe that all objects of the universe are composed of five elements–earth, water, air, fire and ether.
    ● They believe that God is the guiding principle. The living beings were rewarded or punished according to the law of karma, based on actions of merit and demerit.
    ● The Vaisheshika and Nyaya schools eventually merged because of their closely related metaphysical theories (Vaisheshika only accepted perception and inference as sources of valid knowledge).

  2. Question 2 of 5
    2. Question

    2. Consider the following statements:
    Statement I: Home Rule Movement was India’s response to the First World War.
    Statement II: Indian people faced hardships caused by high taxation and rise in prices of goods during the first world war.
    Which one of the following is correct in respect of the above statements?

    Correct

    Answer: A
    Explanation:
    Statement 1 is correct: Home Rule Movement was India’s response to the First World War.
    Statement 2 is correct: Indian people faced hardships caused by high taxation and rise in prices of goods during the first world war.
    There were several factors that contributed to the formation of the Home Rule Movement. One of them was consequences of world war. The war, which was fought between the major imperialist powers of the time and was backed by open propaganda against each other, exposed the myth of white superiority. The world war led Indian people to feel the burden of war time miseries caused by high taxation and a rise in prices.
    Additional information:
    Tilak and Annie Besant were ready to assume the leadership, and the movement started with great vigour. Two Indian Home Rule Leagues were organised on the lines of the Irish Home Rule Leagues and they represented the emergence of a new trend of aggressive politics. The League campaign aimed to convey to the common man the message of home rule as self-government.
    Objectives of Home Rule Movement:
    ● To achieve self-government in India.
    ● To promote political education and discussion to set up agitation for self-government.
    ● To build confidence among Indians to speak against the government’s suppression.
    ● To demand a larger political representation for Indians from the British government.
    ● To revive political activity in India while maintaining the principles of the Congress Party.

    Incorrect

    Answer: A
    Explanation:
    Statement 1 is correct: Home Rule Movement was India’s response to the First World War.
    Statement 2 is correct: Indian people faced hardships caused by high taxation and rise in prices of goods during the first world war.
    There were several factors that contributed to the formation of the Home Rule Movement. One of them was consequences of world war. The war, which was fought between the major imperialist powers of the time and was backed by open propaganda against each other, exposed the myth of white superiority. The world war led Indian people to feel the burden of war time miseries caused by high taxation and a rise in prices.
    Additional information:
    Tilak and Annie Besant were ready to assume the leadership, and the movement started with great vigour. Two Indian Home Rule Leagues were organised on the lines of the Irish Home Rule Leagues and they represented the emergence of a new trend of aggressive politics. The League campaign aimed to convey to the common man the message of home rule as self-government.
    Objectives of Home Rule Movement:
    ● To achieve self-government in India.
    ● To promote political education and discussion to set up agitation for self-government.
    ● To build confidence among Indians to speak against the government’s suppression.
    ● To demand a larger political representation for Indians from the British government.
    ● To revive political activity in India while maintaining the principles of the Congress Party.

  3. Question 3 of 5
    3. Question

    3. In the context of administration of Bahmani kingdom, consider the following statements:
    1. During the rule of Bahamani kingdom, nobility was divided into “Khanzada” and ‘’Afaqis’’.
    2. For administrative purposes, the Bahamani kingdom was divided into four “Taraf,” or provinces headed by Amirs.
    3. Bahmani rulers were influenced by the “Iqtadari System” of administration from the Delhi Sultanate.
    How many of the statements given above are correct?

    Correct

    Answer: A
    Explanation:
    • Statement 1 is incorrect: In the early decades of the foundation of the Bahmani Kingdom, feuds arose among the nobility and the nobles were divided into “Deccanies” and “Afaqis”. The struggle between the Afaqis and Deccanis had a destabilizing effect on the Bahmani Sultanate’s political structure. Succession disputes and conflicts over the throne were often exacerbated by the internal divisions, leading to periods of political instability.
    • Statement 2 is incorrect: For administrative purposes, the Bahamani kingdom was divided into four “Taraf,” or provinces headed by Tarafdars or Subedars not Amirs.
    • Statement 3 is correct: Bahmani rulers were influenced by the “Iqtadari System” for administration adopted from the Delhi Sultanate.
    Additional information:
    ● The Bahmani Kingdom was a Muslim dynasty in southern India during the medieval period in Indian history.
    ● The founder of the Bahmani kingdom was Alauddin Hassan Gangu Bahaman Shah. He rebelled against Muhammad Bin Tughlaq in 1347.
    ● Bahmani Kingdom’s capital city was Gulbarga in present-day Karnataka. This Bahmani kingdom was ruled by 14 sultans. Among them, Alauddin Bahman Shah, Muhammad Shah I, and Firoz Shah were important rulers.
    ● The power of the Bahmani kingdom reached its peak during the reign of Muhammad Shah III. The borders of the Bahmani kingdom extend from Goa to Bombay in the west and from Krishna to Kakinada in the east.
    ● The Bahmani dynasty was the first to unite the Deccan part of the Indian subcontinent with their kingdom. The Bahmani kingdom served as a cultural bridge between the northern and southern regions of the Indian subcontinent.
    ● Dakhini Urdu was a popular language widely propagated from the Bahmani kingdom and Dakhini poetry also flourished during that time and is still performed in Hyderabad today.

    Incorrect

    Answer: A
    Explanation:
    • Statement 1 is incorrect: In the early decades of the foundation of the Bahmani Kingdom, feuds arose among the nobility and the nobles were divided into “Deccanies” and “Afaqis”. The struggle between the Afaqis and Deccanis had a destabilizing effect on the Bahmani Sultanate’s political structure. Succession disputes and conflicts over the throne were often exacerbated by the internal divisions, leading to periods of political instability.
    • Statement 2 is incorrect: For administrative purposes, the Bahamani kingdom was divided into four “Taraf,” or provinces headed by Tarafdars or Subedars not Amirs.
    • Statement 3 is correct: Bahmani rulers were influenced by the “Iqtadari System” for administration adopted from the Delhi Sultanate.
    Additional information:
    ● The Bahmani Kingdom was a Muslim dynasty in southern India during the medieval period in Indian history.
    ● The founder of the Bahmani kingdom was Alauddin Hassan Gangu Bahaman Shah. He rebelled against Muhammad Bin Tughlaq in 1347.
    ● Bahmani Kingdom’s capital city was Gulbarga in present-day Karnataka. This Bahmani kingdom was ruled by 14 sultans. Among them, Alauddin Bahman Shah, Muhammad Shah I, and Firoz Shah were important rulers.
    ● The power of the Bahmani kingdom reached its peak during the reign of Muhammad Shah III. The borders of the Bahmani kingdom extend from Goa to Bombay in the west and from Krishna to Kakinada in the east.
    ● The Bahmani dynasty was the first to unite the Deccan part of the Indian subcontinent with their kingdom. The Bahmani kingdom served as a cultural bridge between the northern and southern regions of the Indian subcontinent.
    ● Dakhini Urdu was a popular language widely propagated from the Bahmani kingdom and Dakhini poetry also flourished during that time and is still performed in Hyderabad today.

  4. Question 4 of 5
    4. Question

    4. In the context of ancient India, the terms ‘Nishaka’ and ‘Satmana’ refer to:

    Correct

    Answer: A
    Explanation:
    ● Satamana and Nishka were gold coins which were used for trading during vedic period.
    ● The unit value of goods was a gold bar called “nishka” weighing three hundred and twenty ratis, which was also the weight of a satamana.
    Additional information:
    ● The Coinage of India began anywhere between early 1st millennium BCE to the 6th century BCE, and consisted mainly of copper and silver coins in its initial stage. The coins of this period were Karshapanas or Pana.
    ● A variety of earliest Indian coins, however, unlike those circulated in West Asia, were stamped bars of metal, suggesting that the innovation of stamped currency was added to a pre-existing form of token currency which had already been present in the Janapadas and Mahajanapada kingdoms of the Early historic India.
    ● The kingdoms that minted their own coins included Gandhara, Kuntala, Kuru, Magadha, Panchala, Shakya, Surasena, Surashtra and Vidarbha etc.

    Incorrect

    Answer: A
    Explanation:
    ● Satamana and Nishka were gold coins which were used for trading during vedic period.
    ● The unit value of goods was a gold bar called “nishka” weighing three hundred and twenty ratis, which was also the weight of a satamana.
    Additional information:
    ● The Coinage of India began anywhere between early 1st millennium BCE to the 6th century BCE, and consisted mainly of copper and silver coins in its initial stage. The coins of this period were Karshapanas or Pana.
    ● A variety of earliest Indian coins, however, unlike those circulated in West Asia, were stamped bars of metal, suggesting that the innovation of stamped currency was added to a pre-existing form of token currency which had already been present in the Janapadas and Mahajanapada kingdoms of the Early historic India.
    ● The kingdoms that minted their own coins included Gandhara, Kuntala, Kuru, Magadha, Panchala, Shakya, Surasena, Surashtra and Vidarbha etc.

  5. Question 5 of 5
    5. Question

    5. Consider the following:
    Festivals/Fairs – Associated States
    1. Nabakalebar Festival – Odisha
    2. Tyagaraja Aradhana – Karnataka
    3. Sarhul – Rajasthan
    4. Saga Dawa – Sikkim
    How many of the above pairs are correctly matched?

    Correct

    Answer: B
    Explanation:
    Given below is the correctly matched pairs:
    Festivals/Fairs – Associated states
    1) Nabakalebar festival – Odisha
    2) Tyagaraja Aradhana – Andhra Pradesh and Tamil Nadu
    3) Sarhul – Jharkhand, Odisha and West Bengal
    4) Saga Dawa – Sikkim
    Additional information:
    Nabakalebar festival
    ● Nabakalebar festival is observed at Shri Jagannath temple, Puri (Odisha) at a pre-defined time (after every 8 to 19 years) according to the Hindu calendar.
    ● Nabakalebar means new body, i.e. the idols of Lord Jaganath, Balabhadra, Subhadra and Sudarshan are replaced with new idols. The new idols are made from the log (daru) of 04 separate neem trees which are selected as per set criteria and after a rigorous search.
    ● From the daru or log of the selected neem trees, the idols are carved and are replaced during the Adhik Masa (intercalary month).
    ● Lakhs and Lakhs of pilgrims attend to worship the selected neem tree and the ceremony of replacement of idols.
    Tyagaraja Aradhana
    ● It is held annually to commemorate the ‘Samadhi’ day of the famous Telugu Saint and composer Tyagaraja.
    ● It is held in Andhra Pradesh and Tamil Nadu, primarily at Thiruvaiyaru (where he attained Samadhi).
    ● The festival is attended by leading exponents of Carnatic music who come to pay their tribute to the saint.
    ● Saint Tyagaraja along with Muthuswami Dikshitar and Syama Sastri, comprise the Trinity of Carnatic music.
    Sarhul
    ● Sarhul marks the beginning of the New Year for the tribals of Jharkhand, Odisha and West Bengal.
    ● It is mainly celebrated by the Munda, Oraon and Ho tribes.
    ● Sarhul literally means ‘Worship of Sal’. It is celebrated in the spring season, i.e. in the month of Phalgun according to the Hindu calendar.
    ● Tribals hold nature in great reverence and Mother Earth is worshipped during the festival.
    ● Sarhul is celebrated for several days during which the main traditional dance Sarhul is performed. It is related to religion called “Sarnaism
    Saga Dawa (Triple Blessed Festival)
    ● It is mostly celebrated in the Buddhist communities living in the State of Sikkim. It is celebrated on the full moon day that falls in the middle of the Tibetan lunar month called the Saga Dawa.
    ● This day is considered to be a very auspicious day for the Tibetan community. This falls between May and June and this month is called Saga Dawa or the ‘Month of merits’.
    ● The festival is celebrated to commemorate the birth, enlightenment and death (parinirvana) of Buddha. Most people make a pilgrimage to the monasteries and offer incense sticks, Dhog and water. People also circumambulate the Gompas of the monastery and chant mantras, recite the religious texts and turn the prayer wheels.
    ● Throughout the month of Saga Dawa, the community of Buddhists has to follow three teachings of Buddhism: generosity (dana), morality (sila), and meditation or good feelings (bhavana).

    Incorrect

    Answer: B
    Explanation:
    Given below is the correctly matched pairs:
    Festivals/Fairs – Associated states
    1) Nabakalebar festival – Odisha
    2) Tyagaraja Aradhana – Andhra Pradesh and Tamil Nadu
    3) Sarhul – Jharkhand, Odisha and West Bengal
    4) Saga Dawa – Sikkim
    Additional information:
    Nabakalebar festival
    ● Nabakalebar festival is observed at Shri Jagannath temple, Puri (Odisha) at a pre-defined time (after every 8 to 19 years) according to the Hindu calendar.
    ● Nabakalebar means new body, i.e. the idols of Lord Jaganath, Balabhadra, Subhadra and Sudarshan are replaced with new idols. The new idols are made from the log (daru) of 04 separate neem trees which are selected as per set criteria and after a rigorous search.
    ● From the daru or log of the selected neem trees, the idols are carved and are replaced during the Adhik Masa (intercalary month).
    ● Lakhs and Lakhs of pilgrims attend to worship the selected neem tree and the ceremony of replacement of idols.
    Tyagaraja Aradhana
    ● It is held annually to commemorate the ‘Samadhi’ day of the famous Telugu Saint and composer Tyagaraja.
    ● It is held in Andhra Pradesh and Tamil Nadu, primarily at Thiruvaiyaru (where he attained Samadhi).
    ● The festival is attended by leading exponents of Carnatic music who come to pay their tribute to the saint.
    ● Saint Tyagaraja along with Muthuswami Dikshitar and Syama Sastri, comprise the Trinity of Carnatic music.
    Sarhul
    ● Sarhul marks the beginning of the New Year for the tribals of Jharkhand, Odisha and West Bengal.
    ● It is mainly celebrated by the Munda, Oraon and Ho tribes.
    ● Sarhul literally means ‘Worship of Sal’. It is celebrated in the spring season, i.e. in the month of Phalgun according to the Hindu calendar.
    ● Tribals hold nature in great reverence and Mother Earth is worshipped during the festival.
    ● Sarhul is celebrated for several days during which the main traditional dance Sarhul is performed. It is related to religion called “Sarnaism
    Saga Dawa (Triple Blessed Festival)
    ● It is mostly celebrated in the Buddhist communities living in the State of Sikkim. It is celebrated on the full moon day that falls in the middle of the Tibetan lunar month called the Saga Dawa.
    ● This day is considered to be a very auspicious day for the Tibetan community. This falls between May and June and this month is called Saga Dawa or the ‘Month of merits’.
    ● The festival is celebrated to commemorate the birth, enlightenment and death (parinirvana) of Buddha. Most people make a pilgrimage to the monasteries and offer incense sticks, Dhog and water. People also circumambulate the Gompas of the monastery and chant mantras, recite the religious texts and turn the prayer wheels.
    ● Throughout the month of Saga Dawa, the community of Buddhists has to follow three teachings of Buddhism: generosity (dana), morality (sila), and meditation or good feelings (bhavana).

window.wpAdvQuizInitList = window.wpAdvQuizInitList || []; window.wpAdvQuizInitList.push({ id: '#wpAdvQuiz_666', init: { quizId: 666, mode: 0, globalPoints: 10, timelimit: 0, resultsGrade: [0], bo: 0, qpp: 0, catPoints: [10], formPos: 0, lbn: "Finish quiz", json: {"3114":{"type":"single","id":3114,"catId":0,"points":2,"correct":[1,0,0,0]},"3115":{"type":"single","id":3115,"catId":0,"points":2,"correct":[1,0,0,0]},"3116":{"type":"single","id":3116,"catId":0,"points":2,"correct":[1,0,0,0]},"3117":{"type":"single","id":3117,"catId":0,"points":2,"correct":[1,0,0,0]},"3118":{"type":"single","id":3118,"catId":0,"points":2,"correct":[0,1,0,0]}} } });




Day-624 | Daily MCQs | UPSC Prelims | POLITY

Day-624

Time limit: 0

Quiz-summary

0 of 5 questions completed

Questions:

  1. 1
  2. 2
  3. 3
  4. 4
  5. 5

Information

DAILY MCQ

You have already completed the quiz before. Hence you can not start it again.

Quiz is loading...

You must sign in or sign up to start the quiz.

You have to finish following quiz, to start this quiz:

Results

0 of 5 questions answered correctly

Your time:

Time has elapsed

You have reached 0 of 0 points, (0)

Categories

  1. Not categorized 0%
  1. 1
  2. 2
  3. 3
  4. 4
  5. 5
  1. Answered
  2. Review
  1. Question 1 of 5
    1. Question

    Consider the following statements:

    1. Laws promoting the Directive Principles of State Policy (DPSP) cannot be challenged in court simply because they restrict Fundamental Rights.
    2. When enacting laws based on the DPSP, the government cannot infringe upon the right to equality (Article 14) and freedom of expression (Article 19).

    Which of the statements given above is/are correct?

    Correct

    Answer: B

    Explanation:

    Statement 1 is incorrect: In the Kesavananda Bharati case (1973), the Supreme Court held that Judicial Review is a basic feature of the Constitution and hence, cannot be taken away. Hence, laws which seek to implement the Directive Principles of State Policy can be questioned in courts on the ground that they violate the Fundamental rights.

    Statement 2 is correct: No law which seeks to implement the Socialist Directive Principles specified in Article 39(b) and 39(c) shall be void on the ground of contravention of the Fundamental Rights conferred by Article 14 and 19. (Not all Directive Principles).

    Incorrect

    Answer: B

    Explanation:

    Statement 1 is incorrect: In the Kesavananda Bharati case (1973), the Supreme Court held that Judicial Review is a basic feature of the Constitution and hence, cannot be taken away. Hence, laws which seek to implement the Directive Principles of State Policy can be questioned in courts on the ground that they violate the Fundamental rights.

    Statement 2 is correct: No law which seeks to implement the Socialist Directive Principles specified in Article 39(b) and 39(c) shall be void on the ground of contravention of the Fundamental Rights conferred by Article 14 and 19. (Not all Directive Principles).

  2. Question 2 of 5
    2. Question

    Consider the following statements:

    1. National emergency grants the Parliament law making power over state subjects, overriding state legislatures.
    2. The President can suspend the enforcement of all fundamental rights during a national emergency.
    3. National emergency allows the Parliament to extend the term of the Lok Sabha, one year at a time, for an indefinite period.

    How many of the above given statements are correct?

    Correct

    Answer: B

    Explanation:

    Statement 1 is correct: During National Emergency, the parliament becomes empowered to make laws on any subject mentioned in the state list. Although the legislative power of the state legislature is not suspended, it becomes subject to the overriding power of the parliament.

    Statement 2 is incorrect: The Constitution authorizes the President to suspend the right to move any court for the enforcement of fundamental rights during the national emergency.

    Statement 3 is correct: While a proclamation of national emergency is in operation, the life of the Lok Sabha may be extended beyond its normal term of five years by law of the parliament for one year at a time, for any length of time. However, this extension cannot continue beyond a period of six months after the emergency has ceased to operate.

    Incorrect

    Answer: B

    Explanation:

    Statement 1 is correct: During National Emergency, the parliament becomes empowered to make laws on any subject mentioned in the state list. Although the legislative power of the state legislature is not suspended, it becomes subject to the overriding power of the parliament.

    Statement 2 is incorrect: The Constitution authorizes the President to suspend the right to move any court for the enforcement of fundamental rights during the national emergency.

    Statement 3 is correct: While a proclamation of national emergency is in operation, the life of the Lok Sabha may be extended beyond its normal term of five years by law of the parliament for one year at a time, for any length of time. However, this extension cannot continue beyond a period of six months after the emergency has ceased to operate.

  3. Question 3 of 5
    3. Question

    Consider the following statements about the Foreign Contribution Regulation Act (FCRA):

    1. It is granted only to associations engaged in areas of cultural, economic, educational, religious, or social programs.
    2. An NGO is required to open a bank account for the receipt of the foreign funds in any branch of the State Bank of India across the country.
    3. In case FCRA license is cancelled, an NGO is ineligible for ten years before re-applying for the license.

    How many of the above statements are correct?

    Correct

    Answer: D

    Explanation:

    Statement 1 is incorrect: The Foreign Contribution (Regulation) Act, established in 1976, aims to safeguard India’s sovereignty as a democratic republic. It achieves this by overseeing foreign donations received by both individuals and organizations. This ensures that such funds are used for legitimate purposes that align with the country’s democratic values and prevents foreign influence from potentially disrupting domestic affairs.

    Statement 2 is incorrect: The FCRA regulates foreign contributions in these defined areas to ensure transparency and compliance with the law. Entities can register under multiple categories based on their programs, allowing for diverse activities. Applicants need to open a bank account for the receipt of the foreign funds at a specified branch of State Bank of India in New Delhi.

    Statement 3 is incorrect: FCRA registration is valid for five years, and NGOs are required to apply for renewal within six months of the registration’s expiry. The government has the authority to cancel an NGO’s FCRA registration for various reasons, including violations of the Act or a lack of reasonable activity in their chosen field for two consecutive years. Once canceled, an NGO is ineligible for re-registration for three years.

    Incorrect

    Answer: D

    Explanation:

    Statement 1 is incorrect: The Foreign Contribution (Regulation) Act, established in 1976, aims to safeguard India’s sovereignty as a democratic republic. It achieves this by overseeing foreign donations received by both individuals and organizations. This ensures that such funds are used for legitimate purposes that align with the country’s democratic values and prevents foreign influence from potentially disrupting domestic affairs.

    Statement 2 is incorrect: The FCRA regulates foreign contributions in these defined areas to ensure transparency and compliance with the law. Entities can register under multiple categories based on their programs, allowing for diverse activities. Applicants need to open a bank account for the receipt of the foreign funds at a specified branch of State Bank of India in New Delhi.

    Statement 3 is incorrect: FCRA registration is valid for five years, and NGOs are required to apply for renewal within six months of the registration’s expiry. The government has the authority to cancel an NGO’s FCRA registration for various reasons, including violations of the Act or a lack of reasonable activity in their chosen field for two consecutive years. Once canceled, an NGO is ineligible for re-registration for three years.

  4. Question 4 of 5
    4. Question

    Consider the following statements about the judges of the Supreme Court:

    1. After ceasing to hold office, he/she cannot appear as a lawyer before any court or authority in India.
    2. There is no cooling-off period for judges after their retirement before they could join political parties or contest elections.

    Which of the statements given above is/are correct?

    Correct

    Answer: C

    Explanation:

    Statement 1 is correct: A retired Supreme Court judge cannot practice law in regular courts or before authorities in India. This is to uphold the dignity of the judge’s position and avoid any conflict of interest.

    Statement 2 is correct: There’s no legal bar on retired judges joining politics or contesting elections. However, some argue there should be an unofficial cooling-off period to avoid using the judge’s position for political gain.

    Incorrect

    Answer: C

    Explanation:

    Statement 1 is correct: A retired Supreme Court judge cannot practice law in regular courts or before authorities in India. This is to uphold the dignity of the judge’s position and avoid any conflict of interest.

    Statement 2 is correct: There’s no legal bar on retired judges joining politics or contesting elections. However, some argue there should be an unofficial cooling-off period to avoid using the judge’s position for political gain.

  5. Question 5 of 5
    5. Question

    Consider the following:

    Statement-I: Members of Parliament (MPs) can claim immunity from prosecution for bribes to cast a vote or make a speech in the House in a particular fashion.

    Statement-II: Article 105(2) of the Indian Constitution confers on MPs immunity from prosecution in respect of anything said or any vote given in Parliament or on any parliamentary committee.

    Which one of the following is correct in respect of the above statements?

    Correct

    Answer: D

    Explanation:

    Article 105(2) of the Indian Constitution confers on MPs immunity from prosecution in respect of anything said or any vote given in Parliament or on any parliamentary committee. Similarly, Article 194(2) grants protection to MLAs.

    In a judgment delivered in March 2024 (Sita Soren V. Union of India), the seven judge bench of the Supreme Court unanimously held that Members of Parliament (MPs) and Members of Legislative Assemblies (MLAs) cannot claim any immunity from prosecution for accepting bribes to cast a vote or make a speech in the House in a particular fashion. The Bench overruled the 3:2 Constitution Bench decision held in P.V Narasimha Rao v State (1998).

    Incorrect

    Answer: D

    Explanation:

    Article 105(2) of the Indian Constitution confers on MPs immunity from prosecution in respect of anything said or any vote given in Parliament or on any parliamentary committee. Similarly, Article 194(2) grants protection to MLAs.

    In a judgment delivered in March 2024 (Sita Soren V. Union of India), the seven judge bench of the Supreme Court unanimously held that Members of Parliament (MPs) and Members of Legislative Assemblies (MLAs) cannot claim any immunity from prosecution for accepting bribes to cast a vote or make a speech in the House in a particular fashion. The Bench overruled the 3:2 Constitution Bench decision held in P.V Narasimha Rao v State (1998).

window.wpAdvQuizInitList = window.wpAdvQuizInitList || []; window.wpAdvQuizInitList.push({ id: '#wpAdvQuiz_665', init: { quizId: 665, mode: 0, globalPoints: 10, timelimit: 0, resultsGrade: [0], bo: 0, qpp: 0, catPoints: [10], formPos: 0, lbn: "Finish quiz", json: {"3109":{"type":"single","id":3109,"catId":0,"points":2,"correct":[0,1,0,0]},"3110":{"type":"single","id":3110,"catId":0,"points":2,"correct":[0,1,0,0]},"3111":{"type":"single","id":3111,"catId":0,"points":2,"correct":[0,0,0,1]},"3112":{"type":"single","id":3112,"catId":0,"points":2,"correct":[0,0,1,0]},"3113":{"type":"single","id":3113,"catId":0,"points":2,"correct":[0,0,0,1]}} } });




Day-623 | Daily MCQs | UPSC Prelims | ECONOMY

Day-623

Time limit: 0

Quiz-summary

0 of 5 questions completed

Questions:

  1. 1
  2. 2
  3. 3
  4. 4
  5. 5

Information

DAILY MCQ

You have already completed the quiz before. Hence you can not start it again.

Quiz is loading...

You must sign in or sign up to start the quiz.

You have to finish following quiz, to start this quiz:

Results

0 of 5 questions answered correctly

Your time:

Time has elapsed

You have reached 0 of 0 points, (0)

Categories

  1. Not categorized 0%
  1. 1
  2. 2
  3. 3
  4. 4
  5. 5
  1. Answered
  2. Review
  1. Question 1 of 5
    1. Question

    1. With reference to ‘technical recession’, consider the following statements:
    1. It occurs when there is a decline in output for three consecutive quarters.
    2. It is generally caused by a single event and is shorter in duration.
    Which of the statements given above is/are correct?

    Correct

    Answer: B
    Explanation
    Statement 1 is incorrect: A technical recession is a term used to describe two consecutive quarters of decline in output. In the case of a nation’s economy, the term usually refers to back-to-back contractions in real GDP.
    Statement 2 is correct: It is most often caused by a single event and is generally shorter in duration. For example, covid-19 induced lockdown pushed many countries into technical recession.
    Recently, several countries have again slipped into recession and many are at risk. The UK and Japan, Ireland and Finland are going through the phase of technical recession.

    Incorrect

    Answer: B
    Explanation
    Statement 1 is incorrect: A technical recession is a term used to describe two consecutive quarters of decline in output. In the case of a nation’s economy, the term usually refers to back-to-back contractions in real GDP.
    Statement 2 is correct: It is most often caused by a single event and is generally shorter in duration. For example, covid-19 induced lockdown pushed many countries into technical recession.
    Recently, several countries have again slipped into recession and many are at risk. The UK and Japan, Ireland and Finland are going through the phase of technical recession.

  2. Question 2 of 5
    2. Question

    2. Consider the following:
    1. Increase in the savings rate in the economy
    2. Decrease in consumption activity in the economy
    3. Decrease in socio-economic inequality in the country
    4. Increase in investment activity in the country
    How many of the above are the likely effects of increase in financial inclusion in the country?

    Correct

    Answer: C
    Explanation:
    Financial inclusion is the process of ensuring access to financial products and services needed by vulnerable groups at an affordable cost in a transparent manner by institutional players.
    ● The term is broadly used to describe the provision of savings and loan services to the poor in an inexpensive and easy-to-use form.
    ● The concept of financial inclusion was first introduced in India in 2005 by the Reserve Bank of India.
    An increase in financial inclusion leads to many positive effects in the economy. These includes:
    ● Increase in the savings rate in the economy.
    ● Decrease in socio-economic inequality in the country.
    ● Increase in investment activity in the country.
    ● Improving access to credit for consumption and production purposes.
    It doesn’t lead to a decrease in consumption activity in the economy. In fact, research has shown that financial inclusion helps in streamlining the consumption activity of the vulnerable class during the times of economic shock as they have savings to fall back on.

    Incorrect

    Answer: C
    Explanation:
    Financial inclusion is the process of ensuring access to financial products and services needed by vulnerable groups at an affordable cost in a transparent manner by institutional players.
    ● The term is broadly used to describe the provision of savings and loan services to the poor in an inexpensive and easy-to-use form.
    ● The concept of financial inclusion was first introduced in India in 2005 by the Reserve Bank of India.
    An increase in financial inclusion leads to many positive effects in the economy. These includes:
    ● Increase in the savings rate in the economy.
    ● Decrease in socio-economic inequality in the country.
    ● Increase in investment activity in the country.
    ● Improving access to credit for consumption and production purposes.
    It doesn’t lead to a decrease in consumption activity in the economy. In fact, research has shown that financial inclusion helps in streamlining the consumption activity of the vulnerable class during the times of economic shock as they have savings to fall back on.

  3. Question 3 of 5
    3. Question

    3. With reference to the Prompt Corrective Action Plan, consider the following statements:
    1. It is a framework under which banks with weak financial metrics are put under watch by the RBI.
    2. The RBI monitors majorly five parameters to determine whether it needs to put a bank under the PCA plan.
    3. Lending operations are immediately prohibited for banks which are put under the framework.
    How many of the above statements are correct?

    Correct

    Answer: A
    Explanation:
    Statement 1 is correct: PCA is a system that the RBI imposes on banks showing signs of financial stress. It was introduced in 2002 by the RBI for scheduled commercial banks.
    Statement 2 is incorrect: The RBI monitors majorly three parameters to determine whether it needs to put a bank under the PCA plan. These are Capital, Asset Quality and Leverage. Indicators to be tracked for Capital, Asset Quality and Leverage would be CRAR/ Common Equity Tier I Ratio, Net NPA Ratio and Tier-I Leverage Ratio, respectively.
    Statement 3 is incorrect: When a bank is placed under PCA, one or more of the following corrective actions may be prescribed:

    Specifications – Mandatory actions
    Risk Threshold 1 – Restriction on dividend distribution/remittance of profits. Promoters/Owners/Parent (in the case of foreign banks) to bring in capital
    Risk Threshold 2 – In addition to mandatory actions of Threshold 1, i. Restriction on branch expansion; domestic and/or overseas
    Risk Threshold 3 – In addition to mandatory actions of Threshold 1 & 2, Appropriate restrictions on capital expenditure, other than for technological upgradation within Board approved limits
    Lending operations are not immediately prohibited.
    The RBI has recently mandated extending the Prompt Corrective Action plan to government NBFCs from October 2024 onwards.

    Incorrect

    Answer: A
    Explanation:
    Statement 1 is correct: PCA is a system that the RBI imposes on banks showing signs of financial stress. It was introduced in 2002 by the RBI for scheduled commercial banks.
    Statement 2 is incorrect: The RBI monitors majorly three parameters to determine whether it needs to put a bank under the PCA plan. These are Capital, Asset Quality and Leverage. Indicators to be tracked for Capital, Asset Quality and Leverage would be CRAR/ Common Equity Tier I Ratio, Net NPA Ratio and Tier-I Leverage Ratio, respectively.
    Statement 3 is incorrect: When a bank is placed under PCA, one or more of the following corrective actions may be prescribed:

    Specifications – Mandatory actions
    Risk Threshold 1 – Restriction on dividend distribution/remittance of profits. Promoters/Owners/Parent (in the case of foreign banks) to bring in capital
    Risk Threshold 2 – In addition to mandatory actions of Threshold 1, i. Restriction on branch expansion; domestic and/or overseas
    Risk Threshold 3 – In addition to mandatory actions of Threshold 1 & 2, Appropriate restrictions on capital expenditure, other than for technological upgradation within Board approved limits
    Lending operations are not immediately prohibited.
    The RBI has recently mandated extending the Prompt Corrective Action plan to government NBFCs from October 2024 onwards.

  4. Question 4 of 5
    4. Question

    4. With reference to financial instruments, consider the following statements:
    1. Under the call money market, funds are transacted for the period between 2 days and 14 days.
    2. Under the notice money market, funds are transacted on an overnight basis.
    3. Both are categorized as capital market instruments.
    How many of the above statements are correct?

    Correct

    Answer: D
    Explanation:
    Statement 1 is incorrect: Under the call money market, funds are transacted on an overnight basis.
    Statement 2 is incorrect: Under the notice money market, funds are transacted for the period between 2 days and 14 days.
    Participants in the call/notice money market currently include banks (excluding RRBs) and Primary Dealers (PDs), both as borrowers and lenders.
    Statement 3 is incorrect: The call/notice money market forms an important segment of the Money Market.
    ● The money market is a market for short-term financial assets that are close substitutes of money. The most important feature of a money market instrument is that it is liquid and can be turned over quickly at low cost and provides an avenue for equilibrating the short-term surplus funds of lenders and the requirements of borrowers.

    Incorrect

    Answer: D
    Explanation:
    Statement 1 is incorrect: Under the call money market, funds are transacted on an overnight basis.
    Statement 2 is incorrect: Under the notice money market, funds are transacted for the period between 2 days and 14 days.
    Participants in the call/notice money market currently include banks (excluding RRBs) and Primary Dealers (PDs), both as borrowers and lenders.
    Statement 3 is incorrect: The call/notice money market forms an important segment of the Money Market.
    ● The money market is a market for short-term financial assets that are close substitutes of money. The most important feature of a money market instrument is that it is liquid and can be turned over quickly at low cost and provides an avenue for equilibrating the short-term surplus funds of lenders and the requirements of borrowers.

  5. Question 5 of 5
    5. Question

    5. With reference to India’s Five-Year Plans, consider the following statements:
    1. The Green Revolution was introduced in the fifth Five-Year Plan.
    2. The Monopolistic and Restrictive Trade Practices Act was introduced in the fourth Five-Year Plan.
    3. The Mid-Day Meal Scheme was introduced in the eighth Five-Year Plan.
    How many of the above statements are correct?

    Correct

    Answer: B
    Explanation:
    Statement 1 is incorrect: Green Revolution as a strategy was introduced during the annual plans of 1966-69. The period was termed as Plan holiday as there was a departure from the five-year plans. The tenure for fifth FYP was 1974-79.
    Statement 2 is correct: The Monopolistic and Restrictive Trade Practices Act, 1969 was introduced in the fourth five-year plan (1969-74). It was aimed at preventing the monopolistic and restrictive trade practices that could be detrimental to fair competition and consumer interests.
    It was replaced by the Competition Commission Act, 2002.
    Statement 3 is correct: Mid-day meal scheme was introduced in 1995, which was the eighth plan (1992-97).
    The scheme was introduced under the National Program of Nutritional Support to Primary Education for enhancement of enrolment, retention, improvement of attendance and quality of education and improving nutritional levels among children. By the year 1997-98, the program was introduced in all districts of the country.

    Incorrect

    Answer: B
    Explanation:
    Statement 1 is incorrect: Green Revolution as a strategy was introduced during the annual plans of 1966-69. The period was termed as Plan holiday as there was a departure from the five-year plans. The tenure for fifth FYP was 1974-79.
    Statement 2 is correct: The Monopolistic and Restrictive Trade Practices Act, 1969 was introduced in the fourth five-year plan (1969-74). It was aimed at preventing the monopolistic and restrictive trade practices that could be detrimental to fair competition and consumer interests.
    It was replaced by the Competition Commission Act, 2002.
    Statement 3 is correct: Mid-day meal scheme was introduced in 1995, which was the eighth plan (1992-97).
    The scheme was introduced under the National Program of Nutritional Support to Primary Education for enhancement of enrolment, retention, improvement of attendance and quality of education and improving nutritional levels among children. By the year 1997-98, the program was introduced in all districts of the country.

window.wpAdvQuizInitList = window.wpAdvQuizInitList || []; window.wpAdvQuizInitList.push({ id: '#wpAdvQuiz_664', init: { quizId: 664, mode: 0, globalPoints: 10, timelimit: 0, resultsGrade: [0], bo: 0, qpp: 0, catPoints: [10], formPos: 0, lbn: "Finish quiz", json: {"3104":{"type":"single","id":3104,"catId":0,"points":2,"correct":[0,1,0,0]},"3105":{"type":"single","id":3105,"catId":0,"points":2,"correct":[0,0,1,0]},"3106":{"type":"single","id":3106,"catId":0,"points":2,"correct":[1,0,0,0]},"3107":{"type":"single","id":3107,"catId":0,"points":2,"correct":[0,0,0,1]},"3108":{"type":"single","id":3108,"catId":0,"points":2,"correct":[0,1,0,0]}} } });




Day-622 | Daily MCQs | UPSC Prelims | GEOGRAPHY

Day-622

Time limit: 0

Quiz-summary

0 of 5 questions completed

Questions:

  1. 1
  2. 2
  3. 3
  4. 4
  5. 5

Information

DAILY MCQ

You have already completed the quiz before. Hence you can not start it again.

Quiz is loading...

You must sign in or sign up to start the quiz.

You have to finish following quiz, to start this quiz:

Results

0 of 5 questions answered correctly

Your time:

Time has elapsed

You have reached 0 of 0 points, (0)

Categories

  1. Not categorized 0%
  1. 1
  2. 2
  3. 3
  4. 4
  5. 5
  1. Answered
  2. Review
  1. Question 1 of 5
    1. Question

    1. Badwater Basin, seen in the news recently, is located in which one of the following countries?

    Correct

    Answer: D
    Explanation:
    About Badwater Basin:
    ● Badwater Basin is an endorheic basin in Death Valley National Park, Death Valley, California (U.S.A).
    ● It is noted as the lowest point in North America and the United States, with a depth of 282 ft (86 m) below sea level.
    ● Near Mount Whitney, the highest point in the contiguous United States.
    ● Badwater Basin’s endorheic nature means water flows into it but not out, typically resulting in rapid evaporation and ephemeral lakes. However, increased precipitation over the past six months has disrupted this pattern.
    Recent phenomenon which has been observed:
    ● Recent heavy rainfall, following Hurricane Hilary in August 2023, has altered the landscape of the region and has resulted in the appearance of a Lake.
    ● The lake initially reduced in size, but surprisingly persisted through the fall and winter seasons. Its recovery in February 2024 was facilitated by a robust atmospheric river that replenished its waters.
    ● Usually, Death Valley recieves an average annual rainfall of only 51 millimetres but a remarkable 125 mm fell in the past half-year alone, primarily due to two significant weather events. The remnants of Hurricane Hilary deposited 55.88 mm precipitation on August 20, 2023, followed by an atmospheric river bringing another 38.1 mm from February 4-7, 2024.

    Incorrect

    Answer: D
    Explanation:
    About Badwater Basin:
    ● Badwater Basin is an endorheic basin in Death Valley National Park, Death Valley, California (U.S.A).
    ● It is noted as the lowest point in North America and the United States, with a depth of 282 ft (86 m) below sea level.
    ● Near Mount Whitney, the highest point in the contiguous United States.
    ● Badwater Basin’s endorheic nature means water flows into it but not out, typically resulting in rapid evaporation and ephemeral lakes. However, increased precipitation over the past six months has disrupted this pattern.
    Recent phenomenon which has been observed:
    ● Recent heavy rainfall, following Hurricane Hilary in August 2023, has altered the landscape of the region and has resulted in the appearance of a Lake.
    ● The lake initially reduced in size, but surprisingly persisted through the fall and winter seasons. Its recovery in February 2024 was facilitated by a robust atmospheric river that replenished its waters.
    ● Usually, Death Valley recieves an average annual rainfall of only 51 millimetres but a remarkable 125 mm fell in the past half-year alone, primarily due to two significant weather events. The remnants of Hurricane Hilary deposited 55.88 mm precipitation on August 20, 2023, followed by an atmospheric river bringing another 38.1 mm from February 4-7, 2024.

  2. Question 2 of 5
    2. Question

    2. Consider the following statements:
    Statement-I: Rip currents are one of the most well-known coastal hazards on beaches around the world.
    Statement-II: Rip currents are channeled currents of water flowing away from the shore at surf beaches.
    Which one of the following is correct in respect of the above statements?

    Correct

    Answer: A
    Explanation: Both Statement-I and Statement-II are correct and Statement-II is the correct explanation for Statement-I
    Statement-I is correct:
    ● Rip currents are present on many beaches every day of the year, but they are usually too slow to be dangerous to beachgoers. However, under certain wave, tide, and beach shape conditions they can increase to dangerous speeds. The strength and speed of a rip current will likely increase as wave height and wave period increase.

    ● Rip currents are a potential source of danger for people in shallow water with breaking waves, whether this is in seas, oceans or large lakes.
    ● Rip currents are the proximate cause of 80% of rescues carried out by beach lifeguards.
    Statement-II is correct and Statement-II is the correct explanation for Statement-I:
    ● Rip currents: Rip currents are channeled currents of water flowing away from shore at surf beaches. They typically extend from near the shoreline, through the surf zone and past the line of breaking waves. (The surf zone is the area between the high tide level on the beach to the seaward side of breaking waves.)
    How do rip currents form?
    ● Rip currents form when waves break near the shoreline, piling up water between the breaking waves and the beach. One of the ways this water returns to sea is to form a rip current, a narrow stream of water moving swiftly away from shore, often perpendicular to the shoreline.
    ● Drowning deaths occur when people pulled offshore are unable to keep themselves afloat and swim to shore. This may be due to any combination of fear, panic, exhaustion, or lack of swimming skills.

    ● Rip currents are the greatest surf zone hazard to all beachgoers. They can sweep even the strongest swimmer out to sea. Rip currents are particularly dangerous for weak and non-swimmers.

    Incorrect

    Answer: A
    Explanation: Both Statement-I and Statement-II are correct and Statement-II is the correct explanation for Statement-I
    Statement-I is correct:
    ● Rip currents are present on many beaches every day of the year, but they are usually too slow to be dangerous to beachgoers. However, under certain wave, tide, and beach shape conditions they can increase to dangerous speeds. The strength and speed of a rip current will likely increase as wave height and wave period increase.

    ● Rip currents are a potential source of danger for people in shallow water with breaking waves, whether this is in seas, oceans or large lakes.
    ● Rip currents are the proximate cause of 80% of rescues carried out by beach lifeguards.
    Statement-II is correct and Statement-II is the correct explanation for Statement-I:
    ● Rip currents: Rip currents are channeled currents of water flowing away from shore at surf beaches. They typically extend from near the shoreline, through the surf zone and past the line of breaking waves. (The surf zone is the area between the high tide level on the beach to the seaward side of breaking waves.)
    How do rip currents form?
    ● Rip currents form when waves break near the shoreline, piling up water between the breaking waves and the beach. One of the ways this water returns to sea is to form a rip current, a narrow stream of water moving swiftly away from shore, often perpendicular to the shoreline.
    ● Drowning deaths occur when people pulled offshore are unable to keep themselves afloat and swim to shore. This may be due to any combination of fear, panic, exhaustion, or lack of swimming skills.

    ● Rip currents are the greatest surf zone hazard to all beachgoers. They can sweep even the strongest swimmer out to sea. Rip currents are particularly dangerous for weak and non-swimmers.

  3. Question 3 of 5
    3. Question

    3. Consider the following statements:
    1. River Subansiri is a right bank tributary of the Brahmaputra river.
    2. River Gomati is the left bank tributary of the Ganga river.
    3. River Zanskar is the left bank tributary of the Indus river.
    4. River Vaitarna is a left bank tributary of the Tapi river.
    How many of the above statements are correct?

    Correct

    Answer: C
    Explanation: Only Statement 4 is incorrect.
    Statement 1 is correct:
    ● Subansiri river also known as the “Gold River” is the largest tributary of the Upper Brahmaputra river.
    ● It originates from the Tibetan Himalayas and makes its way to India via (Miri Hills) Arunachal Pradesh.
    ● It flows through Arunachal Pradesh and joins Brahmaputra as its right bank tributary in Assam.

    ● Subansiri river has been in news for the landslide that took place at Subansiri Lower Hydroelectric Project on the border of Arunachal Pradesh and Assam after a large part of the hill on the left side of the dam collapsed into its reservoir.
    Statement 2 is correct:
    About Gomati river:
    ● Gomti is a left bank tributary of the Ganga River and an alluvial river of the Ganga Plain.
    ● It originates near Mainkot, from Gomat Taala lake (Fulhar Jheel) in Madhotanda, around 30 kilometers from Pilibhit town.
    ● The river flows through Sitapur, Lucknow, Barabanki, Sultanpur, and Jaunpur before meeting the Ganga at Kaithi, Ghazipur district.
    ● Tributaries: Kathina, Bhainsi, Sarayan, Gon, Reth, Sai, Pili, and Kalyani
    ● The total length of the river is about 960 kilometers.
    ● Gomti is the source of drinking water to Lucknow city.
    In news: Recently, over 600 turtles seized from smugglers were released into the Gomti river in forest area of Malihabad near Chandrika Devi Temple.

    Statement 3 is correct:
    Zanskar river:
    ● The Zanskar River is the major left bank tributary of the Indus River.

    ● It flows entirely within Ladakh, India.
    ● It originates northeast of the Great Himalayan range and drains both the Himalayas and the Zanskar Range within the region of Zanskar.
    ● Zanskar River has two main branches, one is Doda with main source near Pansi-La Pass and other branches are formed by Kargyag River (source near Shingo La) and Tsarap River (source near Baralacha La).
    ● The river meanders north –westwards through the dramatic gorge of Zanskar and finally meets the Indus River near Nimmu of Ladakh region.
    In news: The Chief of the Naval Staff recently inaugurated the Indian Navy’s Chadar Trek expedition at INS Shivaji. It takes place on the frozen Zanskar River in Ladakh.
    Statement 4 is incorrect:
    ● The river Vaitarna is one of the west flowing rivers in the region North of Mumbai and South of the Tapi River.
    ● The river rises in the Sahyadri hill range at Trimbak in the Nasik district of Maharashtra State and after traversing a distance of about 120 km in Maharashtra towards west, it joins the Arabian Sea. So, it does not join any other river as a tributary.
    ● Main tributaries: Pinjal, Ganjai, Surya, Daharji, Tansa.
    ● Significance – Vaitarna supplies much of Mumbai’s drinking water. It is the largest river in the Northern Konkan region and drains Maharashtra’s whole Palghar district.
    In news: A fisherman from Maharashtra’s Palghar district was attacked by a bull shark in the Vaitarna river.

    Incorrect

    Answer: C
    Explanation: Only Statement 4 is incorrect.
    Statement 1 is correct:
    ● Subansiri river also known as the “Gold River” is the largest tributary of the Upper Brahmaputra river.
    ● It originates from the Tibetan Himalayas and makes its way to India via (Miri Hills) Arunachal Pradesh.
    ● It flows through Arunachal Pradesh and joins Brahmaputra as its right bank tributary in Assam.

    ● Subansiri river has been in news for the landslide that took place at Subansiri Lower Hydroelectric Project on the border of Arunachal Pradesh and Assam after a large part of the hill on the left side of the dam collapsed into its reservoir.
    Statement 2 is correct:
    About Gomati river:
    ● Gomti is a left bank tributary of the Ganga River and an alluvial river of the Ganga Plain.
    ● It originates near Mainkot, from Gomat Taala lake (Fulhar Jheel) in Madhotanda, around 30 kilometers from Pilibhit town.
    ● The river flows through Sitapur, Lucknow, Barabanki, Sultanpur, and Jaunpur before meeting the Ganga at Kaithi, Ghazipur district.
    ● Tributaries: Kathina, Bhainsi, Sarayan, Gon, Reth, Sai, Pili, and Kalyani
    ● The total length of the river is about 960 kilometers.
    ● Gomti is the source of drinking water to Lucknow city.
    In news: Recently, over 600 turtles seized from smugglers were released into the Gomti river in forest area of Malihabad near Chandrika Devi Temple.

    Statement 3 is correct:
    Zanskar river:
    ● The Zanskar River is the major left bank tributary of the Indus River.

    ● It flows entirely within Ladakh, India.
    ● It originates northeast of the Great Himalayan range and drains both the Himalayas and the Zanskar Range within the region of Zanskar.
    ● Zanskar River has two main branches, one is Doda with main source near Pansi-La Pass and other branches are formed by Kargyag River (source near Shingo La) and Tsarap River (source near Baralacha La).
    ● The river meanders north –westwards through the dramatic gorge of Zanskar and finally meets the Indus River near Nimmu of Ladakh region.
    In news: The Chief of the Naval Staff recently inaugurated the Indian Navy’s Chadar Trek expedition at INS Shivaji. It takes place on the frozen Zanskar River in Ladakh.
    Statement 4 is incorrect:
    ● The river Vaitarna is one of the west flowing rivers in the region North of Mumbai and South of the Tapi River.
    ● The river rises in the Sahyadri hill range at Trimbak in the Nasik district of Maharashtra State and after traversing a distance of about 120 km in Maharashtra towards west, it joins the Arabian Sea. So, it does not join any other river as a tributary.
    ● Main tributaries: Pinjal, Ganjai, Surya, Daharji, Tansa.
    ● Significance – Vaitarna supplies much of Mumbai’s drinking water. It is the largest river in the Northern Konkan region and drains Maharashtra’s whole Palghar district.
    In news: A fisherman from Maharashtra’s Palghar district was attacked by a bull shark in the Vaitarna river.

  4. Question 4 of 5
    4. Question

    4. Consider the following pairs:
    Islands in news – Countries
    1. Socotra Island – Turkey
    2. Kuril Island – Japan
    3. Agalega Island – Maldives
    How many of the above pairs are correctly matched?

    Correct

    Answer: D
    Explanation: None of the pair is matched correctly
    Pair 1 is matched incorrectly:
    About Socotra Island:
    ● Socotra is an island of the Republic of Yemen in the Indian Ocean.

    ● The island of Socotra represents around 95% of the landmass of the Socotra archipelago.
    ● It lies 380 kilometres south of the Arabian Peninsula and 240 kilometres east of Somalia.
    ● Despite being controlled by Yemen, it is geographically part of Africa.
    ● The island is isolated and home to a high number of endemic species. Up to a third of its plant life is endemic. It has been described as “the most alien-looking place on Earth.
    ● The island measures 132 kilometres in length and 42 kilometres in width.
    ● In 2008 Socotra was recognised as a UNESCO World Heritage Site.
    In news: Recently, a commercial vessel taking tons of fertilizers has sunk in the Southern Red Sea, posing a threat of an environmental catastrophe in the Red Sea including in UNESCO Heritage Site of the Socotra Archipelago.
    Pair 2 is matched incorrectly:
    About Kuril Island:
    ● The Kuril Islands are a volcanic archipelago administered as part of Sakhalin Oblast in the Russian Far East.

    ● These are a set of four islands situated between the Sea of Okhotsk and the Pacific Ocean near the north of Japan’s northernmost prefecture, Hokkaido.
    ● Japan refers to them as Northern territories, Russia calls them the Kuril Islands and South Korea named them as Dokdo islands.
    ● These are part of the Pacific Ring of Fire belt and have over 100 volcanoes, of which 35 are said to be active volcanoes along with hot springs.
    ● Both Russia and Japan claim sovereignty over them though the islands have been under Russian control since the end of World War II.
    ● The Soviet Union had seized the islands at the end of World War II and by 1949 had expelled its Japanese residents.
    ● Tokyo claims that the disputed islands have been part of Japan since the early 19th century.
    In news: A massive earthquake shook the island recently. Also there is a possibility of a Japan-Russia diplomatic heat-up on the ownership claims over the island amid the Ukraine war.
    Pair 3 is matched incorrectly:
    About Agalega Island:
    ● Agalega islands belong to Mauritius and lie approximately 1,050 km north of Mauritius.
    ● It is a group of two islands with a total area of 26 sq km and a native population of about 300 people.

    In news: Recently, India and Mauritius have jointly inaugurated an airstrip and the St James Jetty on North Agaléga Island in the Indian Ocean.

    Incorrect

    Answer: D
    Explanation: None of the pair is matched correctly
    Pair 1 is matched incorrectly:
    About Socotra Island:
    ● Socotra is an island of the Republic of Yemen in the Indian Ocean.

    ● The island of Socotra represents around 95% of the landmass of the Socotra archipelago.
    ● It lies 380 kilometres south of the Arabian Peninsula and 240 kilometres east of Somalia.
    ● Despite being controlled by Yemen, it is geographically part of Africa.
    ● The island is isolated and home to a high number of endemic species. Up to a third of its plant life is endemic. It has been described as “the most alien-looking place on Earth.
    ● The island measures 132 kilometres in length and 42 kilometres in width.
    ● In 2008 Socotra was recognised as a UNESCO World Heritage Site.
    In news: Recently, a commercial vessel taking tons of fertilizers has sunk in the Southern Red Sea, posing a threat of an environmental catastrophe in the Red Sea including in UNESCO Heritage Site of the Socotra Archipelago.
    Pair 2 is matched incorrectly:
    About Kuril Island:
    ● The Kuril Islands are a volcanic archipelago administered as part of Sakhalin Oblast in the Russian Far East.

    ● These are a set of four islands situated between the Sea of Okhotsk and the Pacific Ocean near the north of Japan’s northernmost prefecture, Hokkaido.
    ● Japan refers to them as Northern territories, Russia calls them the Kuril Islands and South Korea named them as Dokdo islands.
    ● These are part of the Pacific Ring of Fire belt and have over 100 volcanoes, of which 35 are said to be active volcanoes along with hot springs.
    ● Both Russia and Japan claim sovereignty over them though the islands have been under Russian control since the end of World War II.
    ● The Soviet Union had seized the islands at the end of World War II and by 1949 had expelled its Japanese residents.
    ● Tokyo claims that the disputed islands have been part of Japan since the early 19th century.
    In news: A massive earthquake shook the island recently. Also there is a possibility of a Japan-Russia diplomatic heat-up on the ownership claims over the island amid the Ukraine war.
    Pair 3 is matched incorrectly:
    About Agalega Island:
    ● Agalega islands belong to Mauritius and lie approximately 1,050 km north of Mauritius.
    ● It is a group of two islands with a total area of 26 sq km and a native population of about 300 people.

    In news: Recently, India and Mauritius have jointly inaugurated an airstrip and the St James Jetty on North Agaléga Island in the Indian Ocean.

  5. Question 5 of 5
    5. Question

    5. Consider the following pairs:
    Types of plains – Examples
    1. Structural plain – Great Plains of U.S.A.
    2. Depositional plain – Plains of North Canada
    3. Erosional plain – Ganga-Yamuna Plains of India
    How many of the above pairs are correctly matched?

    Correct

    Answer: A
    Explanation: Only Pair 1 is matched correctly
    Plains:
    ● Plains are the most important landforms found on the earth surface.
    ● A plain is nothing but a low-lying relatively flat land surface with very gentle slope and minimum local relief.
    ● About 55% of the earth’s land surface is occupied by plains.
    Classification of plains
    On the basis of their mode of formation, plains can be classified as:
    a) Structural plain
    b) Erosional plains
    c) Depositional plains
    Pair 1 is matched correctly:
    Structural plains:
    ● They are undisturbed plain landforms on the Earth’s surface.
    ● They are structurally depressed areas which make up some of the most expanse natural lowlands on Earth.
    ● Usually, they are formed from horizontally bedded rocks relatively undisturbed by crust movements of the Earth. This type of plain landform is usually created by an uplifting diastrophic movement of a large landmass.
    ● Examples of structural plain landforms are:
    ⮚ The plains of the United States of America
    ⮚ The central lowlands of Australia
    Pair 2 is matched incorrectly:
    Depositional Plains:
    This type of plain landform is formed from materials deposited by rivers, glaciers, waves and wind. The type of sediment which constitutes this plain landform has a significant impact on the fertility and economic relevance of the consequent plain formed by deposition.
    Depositional Plains based on the material deposited by rivers: Alluvial plains
    ● Alluvial plains are vast, sweeping stretches of plain land that are formed by the deposition of sediments called alluvium.
    ● An alluvial plain usually includes floodplains as part of its area but extends beyond such plains. An alluvial plain represents the pattern of floodplain shift over geological time. As a river flows down mountains or hills, it carries sediments resulting from erosion and transports the sediments to the lower plain. In this way, floodplains continue growing and adding up to form massive stretches of alluvial plains.
    ⮚ Example – The Indo-Gangetic Plain in India and the Po Valley in Italy are examples of alluvial plains.
    Pair 3 is matched incorrectly:
    Erosional plains:
    ● These are plain landforms that are produced from the action of various agents of denudation on an existing landform.
    ● These agents of denudation – River, running water, glacier and wind, wear out the rugged surface on an existing landform and smoothens them, giving rise to low undulating plain landforms.
    ● Two major types of plains result from this mode of formation of plain landforms:
    ⮚ Peneplains (Almost plain): Which results from the action of the various agents of denudation mentioned above, exempting wind.
    ⮚ Pediplains: Erosional plains formed from wind action
    ● Examples of plains in this category include:
    ⮚ Northern Canada plains
    ⮚ Ice eroded plains of Western Africa

    Incorrect

    Answer: A
    Explanation: Only Pair 1 is matched correctly
    Plains:
    ● Plains are the most important landforms found on the earth surface.
    ● A plain is nothing but a low-lying relatively flat land surface with very gentle slope and minimum local relief.
    ● About 55% of the earth’s land surface is occupied by plains.
    Classification of plains
    On the basis of their mode of formation, plains can be classified as:
    a) Structural plain
    b) Erosional plains
    c) Depositional plains
    Pair 1 is matched correctly:
    Structural plains:
    ● They are undisturbed plain landforms on the Earth’s surface.
    ● They are structurally depressed areas which make up some of the most expanse natural lowlands on Earth.
    ● Usually, they are formed from horizontally bedded rocks relatively undisturbed by crust movements of the Earth. This type of plain landform is usually created by an uplifting diastrophic movement of a large landmass.
    ● Examples of structural plain landforms are:
    ⮚ The plains of the United States of America
    ⮚ The central lowlands of Australia
    Pair 2 is matched incorrectly:
    Depositional Plains:
    This type of plain landform is formed from materials deposited by rivers, glaciers, waves and wind. The type of sediment which constitutes this plain landform has a significant impact on the fertility and economic relevance of the consequent plain formed by deposition.
    Depositional Plains based on the material deposited by rivers: Alluvial plains
    ● Alluvial plains are vast, sweeping stretches of plain land that are formed by the deposition of sediments called alluvium.
    ● An alluvial plain usually includes floodplains as part of its area but extends beyond such plains. An alluvial plain represents the pattern of floodplain shift over geological time. As a river flows down mountains or hills, it carries sediments resulting from erosion and transports the sediments to the lower plain. In this way, floodplains continue growing and adding up to form massive stretches of alluvial plains.
    ⮚ Example – The Indo-Gangetic Plain in India and the Po Valley in Italy are examples of alluvial plains.
    Pair 3 is matched incorrectly:
    Erosional plains:
    ● These are plain landforms that are produced from the action of various agents of denudation on an existing landform.
    ● These agents of denudation – River, running water, glacier and wind, wear out the rugged surface on an existing landform and smoothens them, giving rise to low undulating plain landforms.
    ● Two major types of plains result from this mode of formation of plain landforms:
    ⮚ Peneplains (Almost plain): Which results from the action of the various agents of denudation mentioned above, exempting wind.
    ⮚ Pediplains: Erosional plains formed from wind action
    ● Examples of plains in this category include:
    ⮚ Northern Canada plains
    ⮚ Ice eroded plains of Western Africa

window.wpAdvQuizInitList = window.wpAdvQuizInitList || []; window.wpAdvQuizInitList.push({ id: '#wpAdvQuiz_663', init: { quizId: 663, mode: 0, globalPoints: 10, timelimit: 0, resultsGrade: [0], bo: 0, qpp: 0, catPoints: [10], formPos: 0, lbn: "Finish quiz", json: {"3099":{"type":"single","id":3099,"catId":0,"points":2,"correct":[0,0,0,1]},"3100":{"type":"single","id":3100,"catId":0,"points":2,"correct":[1,0,0,0]},"3101":{"type":"single","id":3101,"catId":0,"points":2,"correct":[0,0,1,0]},"3102":{"type":"single","id":3102,"catId":0,"points":2,"correct":[0,0,0,1]},"3103":{"type":"single","id":3103,"catId":0,"points":2,"correct":[1,0,0,0]}} } });




Day-621 | Daily MCQs | UPSC Prelims | CURRENT DEVELOPMENTS

Day-621

Time limit: 0

Quiz-summary

0 of 5 questions completed

Questions:

  1. 1
  2. 2
  3. 3
  4. 4
  5. 5

Information

DAILY MCQ

You have already completed the quiz before. Hence you can not start it again.

Quiz is loading...

You must sign in or sign up to start the quiz.

You have to finish following quiz, to start this quiz:

Results

0 of 5 questions answered correctly

Your time:

Time has elapsed

You have reached 0 of 0 points, (0)

Categories

  1. Not categorized 0%
  1. 1
  2. 2
  3. 3
  4. 4
  5. 5
  1. Answered
  2. Review
  1. Question 1 of 5
    1. Question

    1. The location of the thermal equator is not identical to that of the geographic Equator because of which one of the following factors?

    Correct

    Answer: D
    Explanation:
    ● The location of the thermal equator is not identical to that of the geographic Equator.
    ● This is not due to the level of insolation received in both hemispheres. They are virtually equal.
    ● What creates the difference is the distribution of land and water that causes changes in the way heat is distributed and transported across the planets.
    ● Land dominated areas will tend to have a large variation in temperature and a higher average temperature under the same conditions (as compared to a water dominated area).
    ● Thus, the thermal equator lies a bit north of the geographical equator.

    Incorrect

    Answer: D
    Explanation:
    ● The location of the thermal equator is not identical to that of the geographic Equator.
    ● This is not due to the level of insolation received in both hemispheres. They are virtually equal.
    ● What creates the difference is the distribution of land and water that causes changes in the way heat is distributed and transported across the planets.
    ● Land dominated areas will tend to have a large variation in temperature and a higher average temperature under the same conditions (as compared to a water dominated area).
    ● Thus, the thermal equator lies a bit north of the geographical equator.

  2. Question 2 of 5
    2. Question

    2. With reference to trade cooperation, consider the following statements:
    1. Under the Single Market, there are no tariffs or taxes or quotas on goods and/or services from one country entering another country.
    2. Under the Free Trade Area, free movement of goods, services, capital and people is allowed.
    3. Under the Custom Union, the countries agree to apply the same tariffs to goods from outside the union.
    How many of the above statements are correct?

    Correct

    Answer: B
    Explanation:
    Statement 1 is correct: The Single Market creates a unified trading territory that functions without the border regulations and tariffs, which typically apply to trade between countries. The Single Market allows the unrestricted movement of goods, services, capital, and people throughout the territory. For example, the European Single market created by European Union members and four non-EU countries that are members of the European Free Trade Association (EFTA).
    Statement 2 is incorrect: A free trade area is one where there are no tariffs or taxes or quotas on goods and/or services from one country entering another.
    ● A free trade area isn’t necessarily a physical location.
    Statement 3 is correct: A customs union is an agreement between two or more countries to remove trade barriers and lower or eliminate tariffs. Members of a customs union generally apply a common external tariff on imports from non-member countries.

    Incorrect

    Answer: B
    Explanation:
    Statement 1 is correct: The Single Market creates a unified trading territory that functions without the border regulations and tariffs, which typically apply to trade between countries. The Single Market allows the unrestricted movement of goods, services, capital, and people throughout the territory. For example, the European Single market created by European Union members and four non-EU countries that are members of the European Free Trade Association (EFTA).
    Statement 2 is incorrect: A free trade area is one where there are no tariffs or taxes or quotas on goods and/or services from one country entering another.
    ● A free trade area isn’t necessarily a physical location.
    Statement 3 is correct: A customs union is an agreement between two or more countries to remove trade barriers and lower or eliminate tariffs. Members of a customs union generally apply a common external tariff on imports from non-member countries.

  3. Question 3 of 5
    3. Question

    3. With reference to the PM Mega Integrated Textile Regions and Apparel (MITRA) scheme, consider the following statements:
    1. Under the scheme, 10 mega textile parks would be established on the designated sites.
    2. Development Capital Support is available for greenfield sites but not for brownfield sites.
    Which of the statements given above is/are correct?

    Correct

    Answer: D
    Explanation:
    Statement 1 is incorrect: Under the scheme, seven parks have been approved by the government. The Parks will come up in Tamil Nadu, Telangana, Gujarat, Karnataka, Madhya Pradesh, Uttar Pradesh and Maharashtra.
    ● To enable scale, the minimum estimated area per MITRA Park is 1,000 acres.
    Statement 2 is incorrect: An SPV owned by Centre and State Government jointly will be set up for each park which will oversee the implementation of the project.
    ● For a greenfield MITRA Park, a Development Capital Support (DCS) of 30% of the Project Cost, with a cap of ₹ 500 crore per park will be offered.
    ● In respect of partly developed brownfield sites, an assessment will be made of infrastructure already developed and also the textile specific infrastructure yet to be developed. Based on such assessment, DCS @30% of project cost of balance infrastructure and other support facilities to be developed and restricted to a limit of ₹ 200 crore per brownfield park will be provided by the Union government.
    ● Hence, DCS for both the greenfield as well as brownfield sites is available.

    Incorrect

    Answer: D
    Explanation:
    Statement 1 is incorrect: Under the scheme, seven parks have been approved by the government. The Parks will come up in Tamil Nadu, Telangana, Gujarat, Karnataka, Madhya Pradesh, Uttar Pradesh and Maharashtra.
    ● To enable scale, the minimum estimated area per MITRA Park is 1,000 acres.
    Statement 2 is incorrect: An SPV owned by Centre and State Government jointly will be set up for each park which will oversee the implementation of the project.
    ● For a greenfield MITRA Park, a Development Capital Support (DCS) of 30% of the Project Cost, with a cap of ₹ 500 crore per park will be offered.
    ● In respect of partly developed brownfield sites, an assessment will be made of infrastructure already developed and also the textile specific infrastructure yet to be developed. Based on such assessment, DCS @30% of project cost of balance infrastructure and other support facilities to be developed and restricted to a limit of ₹ 200 crore per brownfield park will be provided by the Union government.
    ● Hence, DCS for both the greenfield as well as brownfield sites is available.

  4. Question 4 of 5
    4. Question

    4. Consider the following statements:
    Statement-I: Monetary Policy Committee uses Consumer Price Index (CPI) instead of Wholesale Price Index for inflation targeting.
    Statement-II: WPI does not capture price movement in the service sector.
    Which one of the following is correct in respect of the above statements?

    Correct

    Answer: A
    Explanation:
    Both Statement-I and Statement-II are correct and Statement-II is the correct explanation for Statement-I
    Statement 1 is correct: The Urijit Patel committee had suggested that the apex bank should adopt the new CPI (consumer price index) as the measure of the nominal anchor for policy communication.
    ● Before this, RBI had given more weightage to Wholesale Price Index (WPI) than CPI as the key measure of inflation for all policy purposes.
    ● The target for inflation was decided at 4 per cent with a band of +/- 2 per cent around it.
    Statement 2 is correct: As a reason to target CPI-based inflation, the committee had pointed out that wholesale inflation “does not capture price movements in non-commodity producing sectors like services, which constitute close to two-thirds of economic activity in India”.
    ● Secondly, it pointed out that wholesale inflation “does not generally reflect price movements in all wholesale markets”. This happens because price quotations for some important commodities such as milk, LPG etc. are taken from retail markets.
    ● Thirdly, movements in WPI often reflect large external shocks and as such, the wholesale inflation rate is often subject to large revisions.
    Thus, CPI-based inflation was chosen over WPI-based inflation for anchoring inflation.

    Incorrect

    Answer: A
    Explanation:
    Both Statement-I and Statement-II are correct and Statement-II is the correct explanation for Statement-I
    Statement 1 is correct: The Urijit Patel committee had suggested that the apex bank should adopt the new CPI (consumer price index) as the measure of the nominal anchor for policy communication.
    ● Before this, RBI had given more weightage to Wholesale Price Index (WPI) than CPI as the key measure of inflation for all policy purposes.
    ● The target for inflation was decided at 4 per cent with a band of +/- 2 per cent around it.
    Statement 2 is correct: As a reason to target CPI-based inflation, the committee had pointed out that wholesale inflation “does not capture price movements in non-commodity producing sectors like services, which constitute close to two-thirds of economic activity in India”.
    ● Secondly, it pointed out that wholesale inflation “does not generally reflect price movements in all wholesale markets”. This happens because price quotations for some important commodities such as milk, LPG etc. are taken from retail markets.
    ● Thirdly, movements in WPI often reflect large external shocks and as such, the wholesale inflation rate is often subject to large revisions.
    Thus, CPI-based inflation was chosen over WPI-based inflation for anchoring inflation.

  5. Question 5 of 5
    5. Question

    5. With reference to the Multilateral Investment Guarantee Agency (MIGA), consider the following statements:
    1. It is a member of the International Monetary Fund.
    2. It promotes foreign direct investment into developed economies.
    3. Its guarantees protect investments against non-commercial risks.
    How many of the above statements are correct?

    Correct

    Answer: A
    Explanation:
    Statement 1 is incorrect: The Multilateral Investment Guarantee Agency (MIGA) is a member of the World Bank Group and not the International Monetary Fund.
    Statement 2 is incorrect: It promotes cross-border investment in developing countries.
    Statement 3 is correct: MIGA’s guarantees protect investments against non-commercial risks and can help investors obtain access to funding sources with improved financial terms and conditions.
    It covers new investments as well as investments (a) associated with the expansion, modernization, improvement, or enhancement of existing projects, or (b) where the investor demonstrates a medium- or long-term commitment to the project, so long as in both (a) and (b) the project has a high developmental impact in the host country. Acquisitions of existing projects by new investors, including the privatization of state-owned enterprises, may also be eligible.

    Incorrect

    Answer: A
    Explanation:
    Statement 1 is incorrect: The Multilateral Investment Guarantee Agency (MIGA) is a member of the World Bank Group and not the International Monetary Fund.
    Statement 2 is incorrect: It promotes cross-border investment in developing countries.
    Statement 3 is correct: MIGA’s guarantees protect investments against non-commercial risks and can help investors obtain access to funding sources with improved financial terms and conditions.
    It covers new investments as well as investments (a) associated with the expansion, modernization, improvement, or enhancement of existing projects, or (b) where the investor demonstrates a medium- or long-term commitment to the project, so long as in both (a) and (b) the project has a high developmental impact in the host country. Acquisitions of existing projects by new investors, including the privatization of state-owned enterprises, may also be eligible.

window.wpAdvQuizInitList = window.wpAdvQuizInitList || []; window.wpAdvQuizInitList.push({ id: '#wpAdvQuiz_662', init: { quizId: 662, mode: 0, globalPoints: 10, timelimit: 0, resultsGrade: [0], bo: 0, qpp: 0, catPoints: [10], formPos: 0, lbn: "Finish quiz", json: {"3094":{"type":"single","id":3094,"catId":0,"points":2,"correct":[0,0,0,1]},"3095":{"type":"single","id":3095,"catId":0,"points":2,"correct":[0,1,0,0]},"3096":{"type":"single","id":3096,"catId":0,"points":2,"correct":[0,0,0,1]},"3097":{"type":"single","id":3097,"catId":0,"points":2,"correct":[1,0,0,0]},"3098":{"type":"single","id":3098,"catId":0,"points":2,"correct":[1,0,0,0]}} } });




Day-620 | Daily MCQs | UPSC Prelims | GEOGRAPHY

Day-620

Time limit: 0

Quiz-summary

0 of 5 questions completed

Questions:

  1. 1
  2. 2
  3. 3
  4. 4
  5. 5

Information

DAILY MCQ

You have already completed the quiz before. Hence you can not start it again.

Quiz is loading...

You must sign in or sign up to start the quiz.

You have to finish following quiz, to start this quiz:

Results

0 of 5 questions answered correctly

Your time:

Time has elapsed

You have reached 0 of 0 points, (0)

Categories

  1. Not categorized 0%
  1. 1
  2. 2
  3. 3
  4. 4
  5. 5
  1. Answered
  2. Review
  1. Question 1 of 5
    1. Question

    1. Consider the following factors:
    1. Rotation of the Earth
    2. Air pressure and wind
    3. Ocean water density
    4. Revolution of the Earth
    How many of the above factors influence ocean currents?

    Correct

    Answer: C
    Explanation: Ocean currents are not influenced by the revolution of earth.
    Ocean currents are influenced by two types of forces namely:
    Primary forces:
    ● Heating by solar energy: Heating by solar energy causes the water to expand. That is why, near the equator the ocean water is about 8 cm higher in level than in the middle latitudes. This causes a very slight gradient and water tends to flow down the slope.
    ● Wind: Wind blowing on the surface of the ocean pushes the water to move. Friction between the wind and the water surface affects the movement of the water body in its course.
    ● Gravity: Gravity tends to pull the water down the pile and create gradient variation.
    ● Coriolis force: The Coriolis force intervenes and causes the water to move to the right in the northern hemisphere and to the left in the southern hemisphere.These large accumulations of water and the flow around them are called Gyres.These produce large circular currents in all the ocean basins.
    Secondary forces:
    ● Differences in water density: It affects vertical mobility of ocean currents. Water with high salinity is denser than water with low salinity and in the same way cold water is denser than warm water. Denser water tends to sink, while relatively lighter water tends to rise.
    ● Temperature of water: Cold-water ocean currents occur when the cold water at the poles sinks and slowly moves towards the equator. Warm-water currents travel out from the equator along the surface, flowing towards the poles to replace the sinking cold water.

    Incorrect

    Answer: C
    Explanation: Ocean currents are not influenced by the revolution of earth.
    Ocean currents are influenced by two types of forces namely:
    Primary forces:
    ● Heating by solar energy: Heating by solar energy causes the water to expand. That is why, near the equator the ocean water is about 8 cm higher in level than in the middle latitudes. This causes a very slight gradient and water tends to flow down the slope.
    ● Wind: Wind blowing on the surface of the ocean pushes the water to move. Friction between the wind and the water surface affects the movement of the water body in its course.
    ● Gravity: Gravity tends to pull the water down the pile and create gradient variation.
    ● Coriolis force: The Coriolis force intervenes and causes the water to move to the right in the northern hemisphere and to the left in the southern hemisphere.These large accumulations of water and the flow around them are called Gyres.These produce large circular currents in all the ocean basins.
    Secondary forces:
    ● Differences in water density: It affects vertical mobility of ocean currents. Water with high salinity is denser than water with low salinity and in the same way cold water is denser than warm water. Denser water tends to sink, while relatively lighter water tends to rise.
    ● Temperature of water: Cold-water ocean currents occur when the cold water at the poles sinks and slowly moves towards the equator. Warm-water currents travel out from the equator along the surface, flowing towards the poles to replace the sinking cold water.

  2. Question 2 of 5
    2. Question

    2. Consider the following statements regarding the first stage of Demographic Transition:
    1. The first stage has high fertility and high mortality.
    2. Most of the people are engaged in agriculture.
    3. The population growth is high.
    How many of the above statements are correct?

    Correct

    Answer: B
    Explanation:
    Demographic transition theory can be used to describe and predict the future population of any area. The theory tells us that the population of any region changes from high births and high deaths to low births and low deaths as society progresses from rural agrarian and illiterate to urban industrial and literate society. These changes occur in stages which are collectively known as the demographic cycle.
    Statement 1 is correct:
    ● The first stage has high fertility and high mortality because people reproduce more to compensate for the deaths due to epidemics and variable food supply.
    ● Two hundred years ago all the countries of the world were in this stage.
    ● Fertility remains high in the beginning of the second stage but it declines with time. This is accompanied by a reduced mortality rate. Improvements in sanitation and health conditions lead to decline in mortality. Because of this gap the net addition to population is high.
    ● In the last stage, both fertility and mortality decline considerably. The population is either stable or grows slowly.
    Statement 2 is correct and Statement 3 is incorrect:
    ● The population growth is slow and most of the people are engaged in agriculture where large families are an asset. Life expectancy is low, people are mostly illiterate and have low levels of technology.

    Incorrect

    Answer: B
    Explanation:
    Demographic transition theory can be used to describe and predict the future population of any area. The theory tells us that the population of any region changes from high births and high deaths to low births and low deaths as society progresses from rural agrarian and illiterate to urban industrial and literate society. These changes occur in stages which are collectively known as the demographic cycle.
    Statement 1 is correct:
    ● The first stage has high fertility and high mortality because people reproduce more to compensate for the deaths due to epidemics and variable food supply.
    ● Two hundred years ago all the countries of the world were in this stage.
    ● Fertility remains high in the beginning of the second stage but it declines with time. This is accompanied by a reduced mortality rate. Improvements in sanitation and health conditions lead to decline in mortality. Because of this gap the net addition to population is high.
    ● In the last stage, both fertility and mortality decline considerably. The population is either stable or grows slowly.
    Statement 2 is correct and Statement 3 is incorrect:
    ● The population growth is slow and most of the people are engaged in agriculture where large families are an asset. Life expectancy is low, people are mostly illiterate and have low levels of technology.

  3. Question 3 of 5
    3. Question

    3. Consider the following countries:
    1. Sudan
    2. Iran
    3. Eritrea
    4. Yemen
    5. Oman
    6. Djibouti
    How many of the above countries border the Red Sea?

    Correct

    Answer: B
    Explanation: Oman and Iran are the countries that don’t have borders with the Red Sea.
    Red Sea:
    ● The Red Sea is a seawater inlet of the Indian Ocean, lying between Africa and Asia.
    ● Its connection to the ocean is in the south, through the Bab-el-Mandeb strait and the Gulf of Aden.
    ● To its north lie the Sinai Peninsula, the Gulf of Aqaba, and the Gulf of Suez (leading to the Suez Canal).
    ● It is underlain by the Red Sea Rift, which is part of the Great Rift Valley.
    Countries having borders with the Red Sea:
    ● Egypt
    ● Saudi Arabia
    ● Sudan
    ● Eritrea
    ● Yemen
    ● Djibouti

    Incorrect

    Answer: B
    Explanation: Oman and Iran are the countries that don’t have borders with the Red Sea.
    Red Sea:
    ● The Red Sea is a seawater inlet of the Indian Ocean, lying between Africa and Asia.
    ● Its connection to the ocean is in the south, through the Bab-el-Mandeb strait and the Gulf of Aden.
    ● To its north lie the Sinai Peninsula, the Gulf of Aqaba, and the Gulf of Suez (leading to the Suez Canal).
    ● It is underlain by the Red Sea Rift, which is part of the Great Rift Valley.
    Countries having borders with the Red Sea:
    ● Egypt
    ● Saudi Arabia
    ● Sudan
    ● Eritrea
    ● Yemen
    ● Djibouti

  4. Question 4 of 5
    4. Question

    4. Consider the following statements:
    Statement-I: Major estuaries in India occur on the west coast.
    Statement-II: Continental shelves on the east coast of India are wider than that of the west coast.
    Which one of the following is correct in respect of the above statements?

    Correct

    Answer: C
    Explanation: Statement-I is correct but Statement-II is incorrect
    ● As many rivers are east flowing and the coast is an emergent coast where delta forms easily, therefore, major estuaries in India occur on the west coast.
    ● As the west coast is a submergence coast due to faulting, the continental shelves along the west coast of India is wider that of the east coast.

    Incorrect

    Answer: C
    Explanation: Statement-I is correct but Statement-II is incorrect
    ● As many rivers are east flowing and the coast is an emergent coast where delta forms easily, therefore, major estuaries in India occur on the west coast.
    ● As the west coast is a submergence coast due to faulting, the continental shelves along the west coast of India is wider that of the east coast.

  5. Question 5 of 5
    5. Question

    5. Consider the following processes:
    1. Orogeny
    2. Epeirogeny
    3. Earthquakes
    4. Volcanism
    How many of the above processes signify diastrophism?

    Correct

    Answer: B
    Explanation: Option 1 and 2 are correct
    Diastrophism is the process of deformation of the Earth’s crust which involves folding and faulting. Diastrophism can be considered part of geotectonics. The study of diastrophism encompasses the varying responses of the crust to tectonic stresses. These responses include linear or torsional horizontal movements (such as continental drift) and vertical subsidence and uplift of the lithosphere (strain) in response to natural stresses on Earth’s surface such as the weight of mountains, lakes, and glaciers.
    ● Orogeny, mountain-building event, generally one that occurs in geosynclinal areas.
    ● In contrast to Epeirogeny, an orogeny tends to occur during a relatively short time in linear belts and results in intensive deformation. Orogeny is usually accompanied by folding and faulting of strata, development of angular unconformities (interruptions in the normal deposition of sedimentary rock).
    Sudden processes:
    ● An earthquake is the shaking of the surface of the Earth resulting from a sudden release of energy in the Earth’s lithosphere that creates seismic waves.
    ● The theory of plate tectonics states that the Earth’s solid outer crust, the lithosphere, is separated into plates that move over the asthenosphere, the molten upper portion of the mantle. Oceanic and continental plates come together, spread apart, and interact at boundaries all over the planet.
    ● Volcanism: Disruptions on the Earth’s surface are a common cause of many volcanic eruptions. These movements on the earth’s surface are caused by endogenous forces and come from the great depths of the Earth. These forces act very rapidly and can cause disastrous consequences under the Earth and on the surface within minutes.

    Incorrect

    Answer: B
    Explanation: Option 1 and 2 are correct
    Diastrophism is the process of deformation of the Earth’s crust which involves folding and faulting. Diastrophism can be considered part of geotectonics. The study of diastrophism encompasses the varying responses of the crust to tectonic stresses. These responses include linear or torsional horizontal movements (such as continental drift) and vertical subsidence and uplift of the lithosphere (strain) in response to natural stresses on Earth’s surface such as the weight of mountains, lakes, and glaciers.
    ● Orogeny, mountain-building event, generally one that occurs in geosynclinal areas.
    ● In contrast to Epeirogeny, an orogeny tends to occur during a relatively short time in linear belts and results in intensive deformation. Orogeny is usually accompanied by folding and faulting of strata, development of angular unconformities (interruptions in the normal deposition of sedimentary rock).
    Sudden processes:
    ● An earthquake is the shaking of the surface of the Earth resulting from a sudden release of energy in the Earth’s lithosphere that creates seismic waves.
    ● The theory of plate tectonics states that the Earth’s solid outer crust, the lithosphere, is separated into plates that move over the asthenosphere, the molten upper portion of the mantle. Oceanic and continental plates come together, spread apart, and interact at boundaries all over the planet.
    ● Volcanism: Disruptions on the Earth’s surface are a common cause of many volcanic eruptions. These movements on the earth’s surface are caused by endogenous forces and come from the great depths of the Earth. These forces act very rapidly and can cause disastrous consequences under the Earth and on the surface within minutes.

window.wpAdvQuizInitList = window.wpAdvQuizInitList || []; window.wpAdvQuizInitList.push({ id: '#wpAdvQuiz_661', init: { quizId: 661, mode: 0, globalPoints: 10, timelimit: 0, resultsGrade: [0], bo: 0, qpp: 0, catPoints: [10], formPos: 0, lbn: "Finish quiz", json: {"3089":{"type":"single","id":3089,"catId":0,"points":2,"correct":[0,0,1,0]},"3090":{"type":"single","id":3090,"catId":0,"points":2,"correct":[0,1,0,0]},"3091":{"type":"single","id":3091,"catId":0,"points":2,"correct":[0,1,0,0]},"3092":{"type":"single","id":3092,"catId":0,"points":2,"correct":[0,0,1,0]},"3093":{"type":"single","id":3093,"catId":0,"points":2,"correct":[0,1,0,0]}} } });




Day-619 | Daily MCQs | UPSC Prelims | CURRENT DEVELOPMENTS

Day-619

Time limit: 0

Quiz-summary

0 of 5 questions completed

Questions:

  1. 1
  2. 2
  3. 3
  4. 4
  5. 5

Information

DAILY MCQ

You have already completed the quiz before. Hence you can not start it again.

Quiz is loading...

You must sign in or sign up to start the quiz.

You have to finish following quiz, to start this quiz:

Results

0 of 5 questions answered correctly

Your time:

Time has elapsed

You have reached 0 of 0 points, (0)

Categories

  1. Not categorized 0%
  1. 1
  2. 2
  3. 3
  4. 4
  5. 5
  1. Answered
  2. Review
  1. Question 1 of 5
    1. Question

    1. Which of the following causes cervical cancer?

    Correct

    Answer: A
    Context: Recently as per the World Health Organization (WHO) reports that cervical cancer kills more than 300,000 people worldwide every year. Human papillomavirus vaccine has been introduced to fight this disease globally. Which means that by 2030, 90% of the people aged between 35 to 45 years are to be treated.
    What is a the Cervical cancer (Adenocarcinoma)
    Cervical cancer is a growth of cells that starts in the cervix. The cervix is the lower part of the uterus that connects to the vagina. Various strains of the human papillomavirus, also called HPV, play a role in causing most cervical cancers. HPV is a common infection that’s passed through sexual contact.
    What are the symptoms of cervical cancer (Adenocarcinoma)
    ● Persistent pain in the pelvis or lower back.
    ● Bleeding after sexual intercourse.
    ● Menstrual bleeding that lasts longer or is heavier than usual.
    ● Postmenopausal bleeding.
    ● Unusual bleeding between periods.
    ● Pain or discomfort during urination.
    ● Significant and unexplained weight loss.
    ● Persistent fatigue and lack of energy.
    ● Changes in bowel habits, such as constipation or diarrhea.
    Hence, Option (a) is the correct answer.

    Incorrect

    Answer: A
    Context: Recently as per the World Health Organization (WHO) reports that cervical cancer kills more than 300,000 people worldwide every year. Human papillomavirus vaccine has been introduced to fight this disease globally. Which means that by 2030, 90% of the people aged between 35 to 45 years are to be treated.
    What is a the Cervical cancer (Adenocarcinoma)
    Cervical cancer is a growth of cells that starts in the cervix. The cervix is the lower part of the uterus that connects to the vagina. Various strains of the human papillomavirus, also called HPV, play a role in causing most cervical cancers. HPV is a common infection that’s passed through sexual contact.
    What are the symptoms of cervical cancer (Adenocarcinoma)
    ● Persistent pain in the pelvis or lower back.
    ● Bleeding after sexual intercourse.
    ● Menstrual bleeding that lasts longer or is heavier than usual.
    ● Postmenopausal bleeding.
    ● Unusual bleeding between periods.
    ● Pain or discomfort during urination.
    ● Significant and unexplained weight loss.
    ● Persistent fatigue and lack of energy.
    ● Changes in bowel habits, such as constipation or diarrhea.
    Hence, Option (a) is the correct answer.

  2. Question 2 of 5
    2. Question

    2. Consider the following pairs:
    Space Missions – Countries
    1. Crew-9 Mission – USA
    2. Chang’e 6 Mission – Japan
    3. XPoSat Mission – Russia
    4. SLIM Mission – India
    How many pairs given above are correctly matched?

    Correct

    Answer: A
    Explanation:
    Context: China recently announced Chang’e-7 mission schedule of lunch around 2026 for footprint in space exploration.
    Pair 1 is matched correctly: Crew-9 Mission This is a NASA mission that will deliver four crew members to the International Space Station (ISS),Which should be launched by February 2024.
    Pair 2 is matched incorrectly: The Chang’e-6 mission, slated for launch around 2024, is part of China’s broader strategy to deepen its footprint in space exploration.
    ● Chang’e-6 mission is expected to touch down on the moon in 2024.
    ● It is tasked with collecting samples from the far side of the moon.
    Pair 3 is matched incorrectly: ISRO’s PSLV-C58 launches XPoSat satellite into low eastward inclination orbit in January 2024.
    ● The PSLV Orbital Experimental Module-3 (POEM-3) experiment will be executed carrying 10 identified payloads supplied by ISRO and IN-Space.
    Pair 4 is matched incorrectly:The SLIM space mission was successfully landed on the lunar surface by the Japan Space Agency in February 2024. “I believe this was a major step forward,” said Hitoshi Kuninaka, JAXA director general.

    Incorrect

    Answer: A
    Explanation:
    Context: China recently announced Chang’e-7 mission schedule of lunch around 2026 for footprint in space exploration.
    Pair 1 is matched correctly: Crew-9 Mission This is a NASA mission that will deliver four crew members to the International Space Station (ISS),Which should be launched by February 2024.
    Pair 2 is matched incorrectly: The Chang’e-6 mission, slated for launch around 2024, is part of China’s broader strategy to deepen its footprint in space exploration.
    ● Chang’e-6 mission is expected to touch down on the moon in 2024.
    ● It is tasked with collecting samples from the far side of the moon.
    Pair 3 is matched incorrectly: ISRO’s PSLV-C58 launches XPoSat satellite into low eastward inclination orbit in January 2024.
    ● The PSLV Orbital Experimental Module-3 (POEM-3) experiment will be executed carrying 10 identified payloads supplied by ISRO and IN-Space.
    Pair 4 is matched incorrectly:The SLIM space mission was successfully landed on the lunar surface by the Japan Space Agency in February 2024. “I believe this was a major step forward,” said Hitoshi Kuninaka, JAXA director general.

  3. Question 3 of 5
    3. Question

    3. Consider the following:
    1. Optimisation crop irrigation
    2. Reducing water wastage
    3. Prediction of Hurricane in advance
    4. Prediction of Drought
    How many of the above can be achieved by using artificial intelligence?

    Correct

    Answer: D
    Context: At the recent COP28, NASA and IBM announced that an Artificial intelligence (AI) tool called watsonx.ai would be available on the open-source AI platform Hugging Space.
    Watsonx.ai will help users monitor the Earth from space, measuring environmental changes that have already happened while also making predictions about the future.
    ● Hurricane tracks can be predicted with more accuracy three days in advance.
    ● The technology could also apply to businesses, helping disaster response teams to prepare for fires impacting residential housing or helping supply chain logistics companies better understand macro weather patterns.
    ● AI-enabled vehicles have the potential to minimize energy use by mapping and identifying the most efficient routes.
    ● In agriculture, 40 percent of freshwater usage is wasted on average but with AI technology, farmers can optimize crop irrigation, reducing water wastage and leading to more productive harvests.
    ● When this type of generative AI is used in weather forecasting in the future, it may be possible to anticipate hurricanes, droughts, and other catastrophic weather occurrences with greater accuracy.

    Hence, option (d) is the correct answer.

    Incorrect

    Answer: D
    Context: At the recent COP28, NASA and IBM announced that an Artificial intelligence (AI) tool called watsonx.ai would be available on the open-source AI platform Hugging Space.
    Watsonx.ai will help users monitor the Earth from space, measuring environmental changes that have already happened while also making predictions about the future.
    ● Hurricane tracks can be predicted with more accuracy three days in advance.
    ● The technology could also apply to businesses, helping disaster response teams to prepare for fires impacting residential housing or helping supply chain logistics companies better understand macro weather patterns.
    ● AI-enabled vehicles have the potential to minimize energy use by mapping and identifying the most efficient routes.
    ● In agriculture, 40 percent of freshwater usage is wasted on average but with AI technology, farmers can optimize crop irrigation, reducing water wastage and leading to more productive harvests.
    ● When this type of generative AI is used in weather forecasting in the future, it may be possible to anticipate hurricanes, droughts, and other catastrophic weather occurrences with greater accuracy.

    Hence, option (d) is the correct answer.

  4. Question 4 of 5
    4. Question

    4. They are the second largest land mammals, and their name comes from the Afrikaans, West Germanic word “weit”, meaning wide, and refers to the mouth of this mammal. It is also known as having square lips and almost no hair. Scientists recently announced the first land mammalian pregnancy by transferring embryos from laboratory animals to surrogate mothers. The conservation status of this animal is Near-Threatened.
    Identify the land mammal being described above:

    Correct

    Answer: D
    Context: The world’s first white rhinoceros In vitro fertilization (IVF) pregnancy could offer a way to save the subspecies, with Berlin scientists offering hope of saving the critically endangered northern white rhinoceros subspecies from extinction by using this method.
    ● White rhinos are the second-largest land mammal, and their name comes from the Afrikaans, a West Germanic language, word “weit” which means wide and refers to the animal’s mouth. Also known as the square-lipped rhinoceros.
    ● It is listed as Near Threatened in the IUCN-World Conservation Union’s Red List.
    ● The scientists announced the first-ever rhino pregnancy achieved by transferring a lab-made rhino embryo into a surrogate mother. It took 13 attempts for the breakthrough with a southern white rhino.
    ● The majority (98.8%) of the southern white rhinos occur in just four countries: South Africa, Namibia, Zimbabwe, and Kenya.
    ● The international consortium of scientists, named Bio Rescue, is confident that the success can be replicated with 30 embryos of the northern white stored in liquid nitrogen.

    Hence, option (d) is correct.

    Incorrect

    Answer: D
    Context: The world’s first white rhinoceros In vitro fertilization (IVF) pregnancy could offer a way to save the subspecies, with Berlin scientists offering hope of saving the critically endangered northern white rhinoceros subspecies from extinction by using this method.
    ● White rhinos are the second-largest land mammal, and their name comes from the Afrikaans, a West Germanic language, word “weit” which means wide and refers to the animal’s mouth. Also known as the square-lipped rhinoceros.
    ● It is listed as Near Threatened in the IUCN-World Conservation Union’s Red List.
    ● The scientists announced the first-ever rhino pregnancy achieved by transferring a lab-made rhino embryo into a surrogate mother. It took 13 attempts for the breakthrough with a southern white rhino.
    ● The majority (98.8%) of the southern white rhinos occur in just four countries: South Africa, Namibia, Zimbabwe, and Kenya.
    ● The international consortium of scientists, named Bio Rescue, is confident that the success can be replicated with 30 embryos of the northern white stored in liquid nitrogen.

    Hence, option (d) is correct.

  5. Question 5 of 5
    5. Question

    5. The term ‘Kokborok’ often seen in the news is related to-

    Correct

    Answer: D
    Context:
    Recently, the indefinite strike in many parts of Tripura by Tipra Indigenous Students Federation (TISF) – the student wing of the Tipra Motha party – over the Kokborok exam script controversy has drawn mixed response in Tripura.
    Explanation:
    Kokborok is one of the ancient languages of North East India. It was formerly known as Tipra Kok. Kokborok was declared as an Official Language of the State of Tripura , India by the State Government in the year 1979. `
    Hence, Option (d) is the correct answer.

    Incorrect

    Answer: D
    Context:
    Recently, the indefinite strike in many parts of Tripura by Tipra Indigenous Students Federation (TISF) – the student wing of the Tipra Motha party – over the Kokborok exam script controversy has drawn mixed response in Tripura.
    Explanation:
    Kokborok is one of the ancient languages of North East India. It was formerly known as Tipra Kok. Kokborok was declared as an Official Language of the State of Tripura , India by the State Government in the year 1979. `
    Hence, Option (d) is the correct answer.

window.wpAdvQuizInitList = window.wpAdvQuizInitList || []; window.wpAdvQuizInitList.push({ id: '#wpAdvQuiz_660', init: { quizId: 660, mode: 0, globalPoints: 10, timelimit: 0, resultsGrade: [0], bo: 0, qpp: 0, catPoints: [10], formPos: 0, lbn: "Finish quiz", json: {"3084":{"type":"single","id":3084,"catId":0,"points":2,"correct":[1,0,0,0]},"3085":{"type":"single","id":3085,"catId":0,"points":2,"correct":[1,0,0,0]},"3086":{"type":"single","id":3086,"catId":0,"points":2,"correct":[0,0,0,1]},"3087":{"type":"single","id":3087,"catId":0,"points":2,"correct":[0,0,0,1]},"3088":{"type":"single","id":3088,"catId":0,"points":2,"correct":[0,0,0,1]}} } });




Day-618 | Daily MCQs | UPSC Prelims | ENVIRONMENT

Day-618

Time limit: 0

Quiz-summary

0 of 5 questions completed

Questions:

  1. 1
  2. 2
  3. 3
  4. 4
  5. 5

Information

DAILY MCQ

You have already completed the quiz before. Hence you can not start it again.

Quiz is loading...

You must sign in or sign up to start the quiz.

You have to finish following quiz, to start this quiz:

Results

0 of 5 questions answered correctly

Your time:

Time has elapsed

You have reached 0 of 0 points, (0)

Categories

  1. Not categorized 0%
  1. 1
  2. 2
  3. 3
  4. 4
  5. 5
  1. Answered
  2. Review
  1. Question 1 of 5
    1. Question

    1. In the context of the River Cities Alliance, consider the following statements:
    1. It is a joint initiative of India and Bangladesh.
    2. It is envisaged as a facilitatory platform for initiating river-sensitive planning and development.
    3. The alliance is open to only those cities that are part of the Ganga-Brahmaputra River systems.
    How many of the above statements given are correct?

    Correct

    Answer. A
    Explanation:
    Statement 1 is incorrect: River Cities Alliance (RCA) has been launched by National Mission for Clean Ganga (NMCG) on the year 2022, with the objective to provide the member cities with a platform to discuss and exchange information on aspects that are vital for sustainable management of urban rivers, sharing best practices and supporting innovation. The alliance has been launched initially with 30 cities namely Dehradun, Rishikesh, Haridwar, Srinagar, Varanasi, Kanpur, Prayagraj, Farrukhabad, Mirzapur, Mathura, Bijnor, Ayodhya, Patna, Bhagalpur, Begusarai, Munger, Sahibganj, Rajmahal, Howrah, Jangipur, Hugli-Chinsurah, Behrampore, Maheshtala, Aurangabad, Chennai, Bhubaneshwar, Hyderabad, Pune, Udaipur and Vijaywada.
    Statement 2 is correct: RCA is envisaged as a facilitatory platform for initiating river-sensitive planning and development. It has now gone global during the UNFCCC COP28 held in Dubai, United Arab Emirates. The National Mission for Clean Ganga (NMCG) on behalf of River Cities Alliance (RCA), has signed a Memorandum of Common Purpose (MoCP) with the Mississippi River Cities and Towns Initiative (MRCTI), representing 124 cities/towns situated along the banks of the Mississippi River, USA. The signing ceremony took place at the Rotary Hall as part of the ongoing COP28 or the United Nations Climate Change Conference, currently underway
    Statement 3 is incorrect: The River City Alliance is open to all river cities of India. Any river city can join the Alliance at any time.

    Incorrect

    Answer. A
    Explanation:
    Statement 1 is incorrect: River Cities Alliance (RCA) has been launched by National Mission for Clean Ganga (NMCG) on the year 2022, with the objective to provide the member cities with a platform to discuss and exchange information on aspects that are vital for sustainable management of urban rivers, sharing best practices and supporting innovation. The alliance has been launched initially with 30 cities namely Dehradun, Rishikesh, Haridwar, Srinagar, Varanasi, Kanpur, Prayagraj, Farrukhabad, Mirzapur, Mathura, Bijnor, Ayodhya, Patna, Bhagalpur, Begusarai, Munger, Sahibganj, Rajmahal, Howrah, Jangipur, Hugli-Chinsurah, Behrampore, Maheshtala, Aurangabad, Chennai, Bhubaneshwar, Hyderabad, Pune, Udaipur and Vijaywada.
    Statement 2 is correct: RCA is envisaged as a facilitatory platform for initiating river-sensitive planning and development. It has now gone global during the UNFCCC COP28 held in Dubai, United Arab Emirates. The National Mission for Clean Ganga (NMCG) on behalf of River Cities Alliance (RCA), has signed a Memorandum of Common Purpose (MoCP) with the Mississippi River Cities and Towns Initiative (MRCTI), representing 124 cities/towns situated along the banks of the Mississippi River, USA. The signing ceremony took place at the Rotary Hall as part of the ongoing COP28 or the United Nations Climate Change Conference, currently underway
    Statement 3 is incorrect: The River City Alliance is open to all river cities of India. Any river city can join the Alliance at any time.

  2. Question 2 of 5
    2. Question

    2. Which one of the following states in India is the first in the country to unveil its own comprehensive policy to combat the menace of invasive species?

    Correct

    Answer. D
    Explanation: Recognising the need to combat the menace, Tamil Nadu became the first state in the country to unveil its own comprehensive policy, the Tamil Nadu Policy on Invasive Plants and Ecological Restoration (TNPIPER), in June 2022. The state had proposed developing this policy in September 2021.
    The policy will address highly invasive, habitat-degrading plant species in the forest areas of Tamil Nadu for removal and management simultaneously.
    The principal focus is laid upon developing/elaborating strategies, methodologies and protocols for management of the most problematic weeds in forest areas to start with, on a priority basis which needs immediate attention to restore the habitat health in the wildlife habitat to support wildlife survival as well as to avoid straying out of wild animals.

    Incorrect

    Answer. D
    Explanation: Recognising the need to combat the menace, Tamil Nadu became the first state in the country to unveil its own comprehensive policy, the Tamil Nadu Policy on Invasive Plants and Ecological Restoration (TNPIPER), in June 2022. The state had proposed developing this policy in September 2021.
    The policy will address highly invasive, habitat-degrading plant species in the forest areas of Tamil Nadu for removal and management simultaneously.
    The principal focus is laid upon developing/elaborating strategies, methodologies and protocols for management of the most problematic weeds in forest areas to start with, on a priority basis which needs immediate attention to restore the habitat health in the wildlife habitat to support wildlife survival as well as to avoid straying out of wild animals.

  3. Question 3 of 5
    3. Question

    3. Consider the following statements:
    Statement I: Coldspots provide more opportunity for the rapid diversification of plants and animals than hotspots.
    Statement II: The hotspots of biodiversity, although rich in endemic plant species, are highly threatened ecosystems at present.
    Which one of the following is correct in respect of the above statements?

    Correct

    Answer. B
    Explanation:
    Statement 1 is correct: The Amazon rainforest may be a hotspot for animal and plant diversity, but Louisiana State University scientists report that new species form there less often than previously thought. Places such as deserts and mountaintops that do not have many species provide more opportunity for rapid diversification. This paradox of diversity — that new species form at a faster pace in “cold spots” than hotspots — was reported in the journal Science.
    The researchers who studied diversity in a major group of tropical birds found that although cold spots might be extreme, with dry, unstable environments, they are also relatively empty, giving new species the elbow room to evolve. In contrast, biodiversity hotspots such as the Amazon rainforest are the result of the gradual accumulation of species over time.
    Statement 2 is correct but does not explain statement 1: The following criterion was given by the CI for a region to qualify as biodiversity hotspot:
    o It must contain at least 1,500 species of vascular plants (> 0.5% of the world’s total) as endemic, implying that it must have a high percentage of plant life found nowhere else on the planet. A hotspot, in other words, is irreplaceable.
    o It has to have lost at least 70% of its original habitat or it must have 30% or less of its original natural vegetation. In other words, it must be threatened.

    Incorrect

    Answer. B
    Explanation:
    Statement 1 is correct: The Amazon rainforest may be a hotspot for animal and plant diversity, but Louisiana State University scientists report that new species form there less often than previously thought. Places such as deserts and mountaintops that do not have many species provide more opportunity for rapid diversification. This paradox of diversity — that new species form at a faster pace in “cold spots” than hotspots — was reported in the journal Science.
    The researchers who studied diversity in a major group of tropical birds found that although cold spots might be extreme, with dry, unstable environments, they are also relatively empty, giving new species the elbow room to evolve. In contrast, biodiversity hotspots such as the Amazon rainforest are the result of the gradual accumulation of species over time.
    Statement 2 is correct but does not explain statement 1: The following criterion was given by the CI for a region to qualify as biodiversity hotspot:
    o It must contain at least 1,500 species of vascular plants (> 0.5% of the world’s total) as endemic, implying that it must have a high percentage of plant life found nowhere else on the planet. A hotspot, in other words, is irreplaceable.
    o It has to have lost at least 70% of its original habitat or it must have 30% or less of its original natural vegetation. In other words, it must be threatened.

  4. Question 4 of 5
    4. Question

    4. Consider the following:
    1. Arid and dry conditions
    2. High water table
    3. Sea level rise
    4. Heavy rainfall
    5. Decomposition of organic matter
    Soil salinisation can be caused by how many of the above-mentioned factors?

    Correct

    Answer. B
    Explanation: Options 1, 2, and 3 are correct.
    Saline and alkaline soils are mostly found in arid and dry regions, where the rate of evaporation is high. Soluble salts from the lower layer of the soil come up and get accumulated due to the evaporation of moisture. These soils are not suitable for most crops as the crop yield is poor.
    When the water table rises (e.g. following irrigation in the absence of proper drainage), the salty groundwater may reach the upper soil layers and thus, supply salts to the rootzone.
    Causes of saline soil:
    ● Arid and dry conditions
    ● High water table
    ● Sloppy land that washes out salts in catchment areas
    ● Irrigation with saline water
    ● Poor drainage
    ● Sea level rise
    Acidic soils are mostly found in areas receiving high rainfall, which causes leaching of bases or salts. Note that the below-mentioned are the causes of soil acidity:
    ● Soils developed from acidic rocks like granite are acidic in nature.
    ● Heavy rains or irrigation leach down bases and lime deep within the soil, thereby, increasing soil acidity.
    ● Application of Ammonium sulphate and Ammonium chloride as fertilisers also causes increase in soil acidity.
    ● The decomposition of organic matter, present in the soil, by various microorganisms results in the production of organic acids, which may also increase soil acidity.
    ● Sloppy land washes out salts in catchment areas.

    Incorrect

    Answer. B
    Explanation: Options 1, 2, and 3 are correct.
    Saline and alkaline soils are mostly found in arid and dry regions, where the rate of evaporation is high. Soluble salts from the lower layer of the soil come up and get accumulated due to the evaporation of moisture. These soils are not suitable for most crops as the crop yield is poor.
    When the water table rises (e.g. following irrigation in the absence of proper drainage), the salty groundwater may reach the upper soil layers and thus, supply salts to the rootzone.
    Causes of saline soil:
    ● Arid and dry conditions
    ● High water table
    ● Sloppy land that washes out salts in catchment areas
    ● Irrigation with saline water
    ● Poor drainage
    ● Sea level rise
    Acidic soils are mostly found in areas receiving high rainfall, which causes leaching of bases or salts. Note that the below-mentioned are the causes of soil acidity:
    ● Soils developed from acidic rocks like granite are acidic in nature.
    ● Heavy rains or irrigation leach down bases and lime deep within the soil, thereby, increasing soil acidity.
    ● Application of Ammonium sulphate and Ammonium chloride as fertilisers also causes increase in soil acidity.
    ● The decomposition of organic matter, present in the soil, by various microorganisms results in the production of organic acids, which may also increase soil acidity.
    ● Sloppy land washes out salts in catchment areas.

  5. Question 5 of 5
    5. Question

    5. Consider the following statements about Pyrolysis:
    1. It is a waste treatment method which is applied under anaerobic conditions.
    2. Pyrolysis of cellulose releases a mixture of methane (CH4), carbon monoxide (CO) and moisture (H20).
    Which of the statements given above is/are incorrect?

    Correct

    Answer. D
    Explanation:
    Statement 1 is correct: Pyrolysis as a waste treatment process refers to destructive distillation of a solid carbonaceous material in the absence of stoichiometric oxygen and in the presence of heat. It needs a supply of heat from an external source.
    Statement 2 is correct: The ideal pyrolysis reaction in case of pure cellulose is described as under:

    Thus, it is seen that the product of pyrolysis of cellulose is a mixture of methane (CH4), carbon monoxide (CO) and moisture (H20).
    This process has been applied in industrial practices to produce charcoal from wood chips, coke and coke gas from coal, fuel gas and pitch from heavy hydrocarbons. It is also used as one of the important processes to dispose of the tyres which are not usable.

    Incorrect

    Answer. D
    Explanation:
    Statement 1 is correct: Pyrolysis as a waste treatment process refers to destructive distillation of a solid carbonaceous material in the absence of stoichiometric oxygen and in the presence of heat. It needs a supply of heat from an external source.
    Statement 2 is correct: The ideal pyrolysis reaction in case of pure cellulose is described as under:

    Thus, it is seen that the product of pyrolysis of cellulose is a mixture of methane (CH4), carbon monoxide (CO) and moisture (H20).
    This process has been applied in industrial practices to produce charcoal from wood chips, coke and coke gas from coal, fuel gas and pitch from heavy hydrocarbons. It is also used as one of the important processes to dispose of the tyres which are not usable.

window.wpAdvQuizInitList = window.wpAdvQuizInitList || []; window.wpAdvQuizInitList.push({ id: '#wpAdvQuiz_659', init: { quizId: 659, mode: 0, globalPoints: 10, timelimit: 0, resultsGrade: [0], bo: 0, qpp: 0, catPoints: [10], formPos: 0, lbn: "Finish quiz", json: {"3079":{"type":"single","id":3079,"catId":0,"points":2,"correct":[1,0,0,0]},"3080":{"type":"single","id":3080,"catId":0,"points":2,"correct":[0,0,0,1]},"3081":{"type":"single","id":3081,"catId":0,"points":2,"correct":[0,1,0,0]},"3082":{"type":"single","id":3082,"catId":0,"points":2,"correct":[0,1,0,0]},"3083":{"type":"single","id":3083,"catId":0,"points":2,"correct":[0,0,0,1]}} } });




Day-617 | Daily MCQs | UPSC Prelims | HISTORY

Day-617

Time limit: 0

Quiz-summary

0 of 5 questions completed

Questions:

  1. 1
  2. 2
  3. 3
  4. 4
  5. 5

Information

DAILY MCQ

You have already completed the quiz before. Hence you can not start it again.

Quiz is loading...

You must sign in or sign up to start the quiz.

You have to finish following quiz, to start this quiz:

Results

0 of 5 questions answered correctly

Your time:

Time has elapsed

You have reached 0 of 0 points, (0)

Categories

  1. Not categorized 0%
  1. 1
  2. 2
  3. 3
  4. 4
  5. 5
  1. Answered
  2. Review
  1. Question 1 of 5
    1. Question

    1. With reference to the history of India, the term ‘Ayyagar’ denotes:

    Correct

    Answer: D
    Explanation:
    The Ayagar system was an important feature of the village organization in the Vijayanagara Empire. In the Ayagar system, each village was organized as an independent unit for the operation of rural administration and 12 officers were appointed for their operation, which were collectively called Ayangars.

    Incorrect

    Answer: D
    Explanation:
    The Ayagar system was an important feature of the village organization in the Vijayanagara Empire. In the Ayagar system, each village was organized as an independent unit for the operation of rural administration and 12 officers were appointed for their operation, which were collectively called Ayangars.

  2. Question 2 of 5
    2. Question

    2. Consider the following statements:
    1. Mughal baluster columns were found only in the palaces of Akbar’s forts.
    2. A new form of Islamic architecture that featured double domes was introduced during the reign of Shah Jahan.
    Which of the statements given above is/are correct?

    Correct

    Answer: D
    Explanation:
    Statement 1 is incorrect: The examples of the Mughal baluster columns are found solely in Shah Jahan’s contributions to the fortress palaces. A baluster column consists of four parts – base, pot like element, shaft and capital. The different elements are separated by protruding rings and an additional concave contraction at the joint between the pot-like element and the shaft which forms its bulbs.
    Statement 2 is incorrect: The Lodi dynasty during their rule, introduced a new form of Islamic architecture that featured double domes. Double domes are built of two layers, with the inner layer providing a ceiling to the interior of the building and the outer layer crowning the building. This technique enabled the ceiling inside to be placed lower and in better relation to the interior space it covers. Example- the tomb of Sikandar Lodi, which is located in Lodhi Gardens in Delhi.

    Incorrect

    Answer: D
    Explanation:
    Statement 1 is incorrect: The examples of the Mughal baluster columns are found solely in Shah Jahan’s contributions to the fortress palaces. A baluster column consists of four parts – base, pot like element, shaft and capital. The different elements are separated by protruding rings and an additional concave contraction at the joint between the pot-like element and the shaft which forms its bulbs.
    Statement 2 is incorrect: The Lodi dynasty during their rule, introduced a new form of Islamic architecture that featured double domes. Double domes are built of two layers, with the inner layer providing a ceiling to the interior of the building and the outer layer crowning the building. This technique enabled the ceiling inside to be placed lower and in better relation to the interior space it covers. Example- the tomb of Sikandar Lodi, which is located in Lodhi Gardens in Delhi.

  3. Question 3 of 5
    3. Question

    3. With reference to the history of India, who was Ishwarakrishna?

    Correct

    Answer: B
    Explanation:
    ● Isvara Krishna was an Indian philosopher and sage. He was the author of Samkhyakarika, an account of the universe and its components according to the Samkhya school, one of the six schools of Hindu philosophy.
    ● Samkhyakarika is the earliest surviving authoritative text on classical Samkhya philosophy.
    ● Samkhya philosophy is said to have inspired some early Buddhist schools, and versions of the Samkhya system are still used in many schools of Buddhism and Jainism.
    ● Samkhya is also the basis for the Yoga school. Different versions of this system are also used in Vaishnavism and Shaivism.

    Incorrect

    Answer: B
    Explanation:
    ● Isvara Krishna was an Indian philosopher and sage. He was the author of Samkhyakarika, an account of the universe and its components according to the Samkhya school, one of the six schools of Hindu philosophy.
    ● Samkhyakarika is the earliest surviving authoritative text on classical Samkhya philosophy.
    ● Samkhya philosophy is said to have inspired some early Buddhist schools, and versions of the Samkhya system are still used in many schools of Buddhism and Jainism.
    ● Samkhya is also the basis for the Yoga school. Different versions of this system are also used in Vaishnavism and Shaivism.

  4. Question 4 of 5
    4. Question

    4. Consider the following:
    1. Tiger
    2. Elephant
    3. Rhinoceros
    4. Buffalo
    5. Deer
    How many of the animals given above are displayed on the proto-Shiva seal obtained from Mohenjodaro?

    Correct

    Answer: D
    Explanation:
    Proto-Shiva seal was discovered during the excavation of Mohenjodaro. The prominent figure in the seal is the man with the horned headdress. The animals depicted in the seal were a rhinoceros, an elephant, a buffalo, and a tiger. Two deers can also be seen right in front of the seated figure of the man.

    Additional information:
    ● The Indus Valley Civilization was established around 3300 BC. It flourished between 2600 BC and 1900 BC (Mature Indus Valley Civilization). It started declining around 1900 BC and disappeared around 1400 BC.
    ● This is also called Harappan Civilization after the first city to be excavated, Harappa (Punjab, Pakistan).
    ● Pre-Harappan civilization has been found in Mehrgarh, Pakistan, showing the first evidence of cotton cultivation.
    ● Geographically, this civilization covered Punjab, Sindh, Baluchistan, Rajasthan, Gujarat and Western Uttar Pradesh. It extended from Sutkagengor (in Baluchistan) in the West to Alamgirpur (Western UP) in the East; and from Mandu (Jammu) in the North to Daimabad (Ahmednagar, Maharashtra) in the South. Some Indus Valley sites have also been found as far away as Afghanistan and Turkmenistan.

    Incorrect

    Answer: D
    Explanation:
    Proto-Shiva seal was discovered during the excavation of Mohenjodaro. The prominent figure in the seal is the man with the horned headdress. The animals depicted in the seal were a rhinoceros, an elephant, a buffalo, and a tiger. Two deers can also be seen right in front of the seated figure of the man.

    Additional information:
    ● The Indus Valley Civilization was established around 3300 BC. It flourished between 2600 BC and 1900 BC (Mature Indus Valley Civilization). It started declining around 1900 BC and disappeared around 1400 BC.
    ● This is also called Harappan Civilization after the first city to be excavated, Harappa (Punjab, Pakistan).
    ● Pre-Harappan civilization has been found in Mehrgarh, Pakistan, showing the first evidence of cotton cultivation.
    ● Geographically, this civilization covered Punjab, Sindh, Baluchistan, Rajasthan, Gujarat and Western Uttar Pradesh. It extended from Sutkagengor (in Baluchistan) in the West to Alamgirpur (Western UP) in the East; and from Mandu (Jammu) in the North to Daimabad (Ahmednagar, Maharashtra) in the South. Some Indus Valley sites have also been found as far away as Afghanistan and Turkmenistan.

  5. Question 5 of 5
    5. Question

    5. Consider the following statements:
    1. Gandhi used Satyagraha and hunger strike for the first time during an industrial dispute between the owners and workers of a cotton mill in Ahmedabad.
    2. The Ahmedabad Mill strike was successful and the workers were granted a 50% wage hike they demanded.
    Which of the statements given above is/are correct?

    Correct

    Answer: A
    Explanation:
    Statement 1 is correct: Gandhi used Satyagraha and hunger strike for the first time during an industrial dispute between the owners and workers of a cotton mill in Ahmedabad in 1918.Statement 2 is incorrect: The Ahmedabad Mill strike was successful and the workers were granted the wage hike they wanted. In order to deal with wartime inflation brought on by Britain’s involvement in World War I, which doubled the prices of food grains, clothing, and other necessities, workers demanded a 50% wage increase. The tribunal in the case decided to end the strike and give the workers a 35% pay raise.
    Additional information:
    ● The textile mill workers in Ahmedabad went on strike in 1918 to demand economic justice after the mill owners stopped giving out plague bonuses.
    ● Gandhi stepped in to mediate this conflict between mill owners and workers in Ahmedabad. He started a fast in order to coerce a compromise.
    ● He also backed the Gujarati Khaira peasants in their fight against the seizure of land revenue following the failure of their crops. Sardar Vallabhbhai Patel left his lucrative legal practice at this time to support Gandhi.
    ● The mill owners wanted to take the bonus in the dispute between the workers and owners of the Ahmedabad mill.
    ● Anusuya Sarabhai was asked for help in the fight for justice. She was a social worker and the sister of Ambalal Sarabhai, one of the mill owners and president of the Ahmedabad Mill Owners Association, which was established in 1891 to develop the textile industry in Ahmedabad.
    ● Gandhi was respected by the mill owners and employees, so Anusuya Behn went to him and asked him to step in and help break the standoff between the employees and the employers.
    ● Despite being Ambalal’s friend, Gandhi championed the cause of the workers.

    Incorrect

    Answer: A
    Explanation:
    Statement 1 is correct: Gandhi used Satyagraha and hunger strike for the first time during an industrial dispute between the owners and workers of a cotton mill in Ahmedabad in 1918.Statement 2 is incorrect: The Ahmedabad Mill strike was successful and the workers were granted the wage hike they wanted. In order to deal with wartime inflation brought on by Britain’s involvement in World War I, which doubled the prices of food grains, clothing, and other necessities, workers demanded a 50% wage increase. The tribunal in the case decided to end the strike and give the workers a 35% pay raise.
    Additional information:
    ● The textile mill workers in Ahmedabad went on strike in 1918 to demand economic justice after the mill owners stopped giving out plague bonuses.
    ● Gandhi stepped in to mediate this conflict between mill owners and workers in Ahmedabad. He started a fast in order to coerce a compromise.
    ● He also backed the Gujarati Khaira peasants in their fight against the seizure of land revenue following the failure of their crops. Sardar Vallabhbhai Patel left his lucrative legal practice at this time to support Gandhi.
    ● The mill owners wanted to take the bonus in the dispute between the workers and owners of the Ahmedabad mill.
    ● Anusuya Sarabhai was asked for help in the fight for justice. She was a social worker and the sister of Ambalal Sarabhai, one of the mill owners and president of the Ahmedabad Mill Owners Association, which was established in 1891 to develop the textile industry in Ahmedabad.
    ● Gandhi was respected by the mill owners and employees, so Anusuya Behn went to him and asked him to step in and help break the standoff between the employees and the employers.
    ● Despite being Ambalal’s friend, Gandhi championed the cause of the workers.

window.wpAdvQuizInitList = window.wpAdvQuizInitList || []; window.wpAdvQuizInitList.push({ id: '#wpAdvQuiz_658', init: { quizId: 658, mode: 0, globalPoints: 10, timelimit: 0, resultsGrade: [0], bo: 0, qpp: 0, catPoints: [10], formPos: 0, lbn: "Finish quiz", json: {"3074":{"type":"single","id":3074,"catId":0,"points":2,"correct":[0,0,0,1]},"3075":{"type":"single","id":3075,"catId":0,"points":2,"correct":[0,0,0,1]},"3076":{"type":"single","id":3076,"catId":0,"points":2,"correct":[0,1,0,0]},"3077":{"type":"single","id":3077,"catId":0,"points":2,"correct":[0,0,0,1]},"3078":{"type":"single","id":3078,"catId":0,"points":2,"correct":[1,0,0,0]}} } });




Day-616 | Daily MCQs | UPSC Prelims | SCIENCE AND TECHNOLOGY

Day-616

Time limit: 0

Quiz-summary

0 of 5 questions completed

Questions:

  1. 1
  2. 2
  3. 3
  4. 4
  5. 5

Information

DAILY MCQ

You have already completed the quiz before. Hence you can not start it again.

Quiz is loading...

You must sign in or sign up to start the quiz.

You have to finish following quiz, to start this quiz:

Results

0 of 5 questions answered correctly

Your time:

Time has elapsed

You have reached 0 of 0 points, (0)

Categories

  1. Not categorized 0%
  1. 1
  2. 2
  3. 3
  4. 4
  5. 5
  1. Answered
  2. Review
  1. Question 1 of 5
    1. Question

    1. Consider the following statements:
    1. Thin coal seams endowed with low-sulphur coal makes rat-hole mining a viable option in the Northeastern India.
    2. Rat-hole mining in Meghalaya has enhanced the pH levels in the Kopili river.
    3. The National Green Tribunal (NGT) imposed a ban on rat-hole mining in 2014.
    How many of the above statements are correct?

    Correct

    Answer. A
    Explanation:
    Statement 1 is incorrect: Rat-hole mining involves digging of very small tunnels, usually only 3-4 feet deep, in which workers, more often children, enter and extract coal. It is practiced in the Northeastern States of India because the coal seams are thin and the coal quality is bad: having high sulphur content. These factors make open mining an unfeasible option in such regions.
    Statement 2 is incorrect: Since rat-hole mining is illegal, it is practiced behind closed doors. The water in the Kopili River (flows through Meghalaya and Assam) has turned acidic (i.e. the pH levels in the river have dropped). The entire roadsides in and around mining areas are for piling of coal. This is a major source of air, water and soil pollution.
    Statement 3 is correct: The National Green Tribunal (NGT) has banned rat-hole mining in 2014, and retained the ban in 2015. The ban was on grounds of the practice being unscientific and unsafe for workers.

    Incorrect

    Answer. A
    Explanation:
    Statement 1 is incorrect: Rat-hole mining involves digging of very small tunnels, usually only 3-4 feet deep, in which workers, more often children, enter and extract coal. It is practiced in the Northeastern States of India because the coal seams are thin and the coal quality is bad: having high sulphur content. These factors make open mining an unfeasible option in such regions.
    Statement 2 is incorrect: Since rat-hole mining is illegal, it is practiced behind closed doors. The water in the Kopili River (flows through Meghalaya and Assam) has turned acidic (i.e. the pH levels in the river have dropped). The entire roadsides in and around mining areas are for piling of coal. This is a major source of air, water and soil pollution.
    Statement 3 is correct: The National Green Tribunal (NGT) has banned rat-hole mining in 2014, and retained the ban in 2015. The ban was on grounds of the practice being unscientific and unsafe for workers.

  2. Question 2 of 5
    2. Question

    2. Consider the following pairs:
    Space Missions – Purpose
    1. Artemis – Creating a permanent base on Moon
    2. XRISM – Demonstration of precise lunar landing
    3. Euclid – Exploration of Dark Universe
    How many of the above pairs are correctly matched?

    Correct

    Answer: B
    Explanation
    Pair 1 is correctly matched: Artemis programme is a series of missions planned by NASA with an aim to establish a permanent base on the Moon. The lunar base will eventually be utilised as a launching pad for various manned Mars missions in future.
    Pair 2 is incorrectly matched: It is the SLIM mission, which is a small-scale exploration lander designed for pinpoint landings on the Moon’s surface. Both the missions (XRISM and SLIM) have been initiated by the Japanese Space Agency (JAXA). The X-Ray Imaging and Spectroscopy Mission (XRISM) is an X-ray space telescope mission to provide breakthroughs in the study of structure formation of the universe and outflows from galaxy nuclei.
    Pair 3 is correctly matched: Euclid mission is a space telescope designed to explore the composition and evolution of the dark Universe. It is a mission by the European Space Agency (ESA). Euclid will explore how the Universe has expanded and how structure has formed over cosmic history, revealing more about the role of gravity and the nature of dark energy and dark matter.

    Incorrect

    Answer: B
    Explanation
    Pair 1 is correctly matched: Artemis programme is a series of missions planned by NASA with an aim to establish a permanent base on the Moon. The lunar base will eventually be utilised as a launching pad for various manned Mars missions in future.
    Pair 2 is incorrectly matched: It is the SLIM mission, which is a small-scale exploration lander designed for pinpoint landings on the Moon’s surface. Both the missions (XRISM and SLIM) have been initiated by the Japanese Space Agency (JAXA). The X-Ray Imaging and Spectroscopy Mission (XRISM) is an X-ray space telescope mission to provide breakthroughs in the study of structure formation of the universe and outflows from galaxy nuclei.
    Pair 3 is correctly matched: Euclid mission is a space telescope designed to explore the composition and evolution of the dark Universe. It is a mission by the European Space Agency (ESA). Euclid will explore how the Universe has expanded and how structure has formed over cosmic history, revealing more about the role of gravity and the nature of dark energy and dark matter.

  3. Question 3 of 5
    3. Question

    3. With reference to different types of vitamins, consider the following statements:
    1. Vitamin B plays a pivotal role in the synthesis of collagen, which is essential for maintaining healthy skin.
    2. Vitamin H, also known as biotin, is essential for hair growth.
    3. Vitamin D is a fat-soluble vitamin.
    How many of the above statements are correct?

    Correct

    Answer: B
    Explanation
    Statement 1 is incorrect: It is Vitamin C which helps in the synthesis of collagen. Collagen is the primary building block of body’s skin, muscles, bones, tendons and ligaments, and other connective tissues. It’s also found in our organs, blood vessels and intestinal lining.
    Statement 2 is correct: Biotin is also known as Vitamin H. It is a water-soluble vitamin that plays a role in the growth of hair. Along with this it also helps in the metabolism of fats, carbohydrates, and amino acids. It was initially called “H” from the German word “Haar und Haut,” meaning hair and skin.
    Statement 3 is correct: Basking in the sun isn’t just for relaxation; it triggers the skin to produce a vital vitamin known as Vitamin D. The sun’s energy turns a chemical in the skin into vitamin D3, which is carried to the liver and then to kidneys to transform it to active vitamin D.

    Incorrect

    Answer: B
    Explanation
    Statement 1 is incorrect: It is Vitamin C which helps in the synthesis of collagen. Collagen is the primary building block of body’s skin, muscles, bones, tendons and ligaments, and other connective tissues. It’s also found in our organs, blood vessels and intestinal lining.
    Statement 2 is correct: Biotin is also known as Vitamin H. It is a water-soluble vitamin that plays a role in the growth of hair. Along with this it also helps in the metabolism of fats, carbohydrates, and amino acids. It was initially called “H” from the German word “Haar und Haut,” meaning hair and skin.
    Statement 3 is correct: Basking in the sun isn’t just for relaxation; it triggers the skin to produce a vital vitamin known as Vitamin D. The sun’s energy turns a chemical in the skin into vitamin D3, which is carried to the liver and then to kidneys to transform it to active vitamin D.

  4. Question 4 of 5
    4. Question

    4. Consider the following statements:
    1. Cores are the hardware-based processing units within a Central Processing Unit (CPU).
    2. Threads are the software-based instructions that can be processed by a CPU.
    Which of the statements given above is/are correct?

    Correct

    Answer: C
    Explanation
    Statement 1 is correct: A core is a physical component of the CPU that can execute instructions. Thus, it is a hardware-based processing unit within a CPU. A CPU performance will depend upon the number of cores on the machine and the speed at which the individual cores can execute instructions.
    Statement 2 is correct: Thread is a single sequential flow of control in a program that allows multiple activities within a single process. Thus, it is the software-based instruction. In other words, a single thread is like one command that runs at a time.

    Incorrect

    Answer: C
    Explanation
    Statement 1 is correct: A core is a physical component of the CPU that can execute instructions. Thus, it is a hardware-based processing unit within a CPU. A CPU performance will depend upon the number of cores on the machine and the speed at which the individual cores can execute instructions.
    Statement 2 is correct: Thread is a single sequential flow of control in a program that allows multiple activities within a single process. Thus, it is the software-based instruction. In other words, a single thread is like one command that runs at a time.

  5. Question 5 of 5
    5. Question

    5. Consider the following statements:
    1. Machine Learning models are suitable for solving simple problems, while Deep Learning models are used to solve complex problems.
    2. Machine Learning is a subset of Deep Learning.
    Which of the statements given above is/are correct?

    Correct

    Answer: A
    Explanation
    Machine Learning and Deep Learning are the two main concepts of Data Science and the subsets of Artificial Intelligence.
    Machine Learning allows the computers to learn from the experiences by its own, use statistical methods to improve the performance and predict the output without being explicitly programmed.
    Deep Learning is inspired by the functionality of human brain cells, which are called neurons, and leads to the concept of artificial neural networks. It is also called a deep neural network or deep neural learning.
    ● Statement 1 is CORRECT: Machine Learning algorithm is data driven. It follows a structured format for solving problems and thus is mainly employed for simpler tasks like predicting customer choice on e-commerce site. But, Deep Learning mimics complex neural schemas of human brain. It is adept at solving complex problems which require parallel processing.
    ● Statement 2 is INCORRECT: Deep Learning is a subset of Machine Learning. Deep Learning is a special case of machine learning in which each step is checked in a more elaborated manner.

    Incorrect

    Answer: A
    Explanation
    Machine Learning and Deep Learning are the two main concepts of Data Science and the subsets of Artificial Intelligence.
    Machine Learning allows the computers to learn from the experiences by its own, use statistical methods to improve the performance and predict the output without being explicitly programmed.
    Deep Learning is inspired by the functionality of human brain cells, which are called neurons, and leads to the concept of artificial neural networks. It is also called a deep neural network or deep neural learning.
    ● Statement 1 is CORRECT: Machine Learning algorithm is data driven. It follows a structured format for solving problems and thus is mainly employed for simpler tasks like predicting customer choice on e-commerce site. But, Deep Learning mimics complex neural schemas of human brain. It is adept at solving complex problems which require parallel processing.
    ● Statement 2 is INCORRECT: Deep Learning is a subset of Machine Learning. Deep Learning is a special case of machine learning in which each step is checked in a more elaborated manner.

window.wpAdvQuizInitList = window.wpAdvQuizInitList || []; window.wpAdvQuizInitList.push({ id: '#wpAdvQuiz_657', init: { quizId: 657, mode: 0, globalPoints: 10, timelimit: 0, resultsGrade: [0], bo: 0, qpp: 0, catPoints: [10], formPos: 0, lbn: "Finish quiz", json: {"3069":{"type":"single","id":3069,"catId":0,"points":2,"correct":[1,0,0,0]},"3070":{"type":"single","id":3070,"catId":0,"points":2,"correct":[0,1,0,0]},"3071":{"type":"single","id":3071,"catId":0,"points":2,"correct":[0,1,0,0]},"3072":{"type":"single","id":3072,"catId":0,"points":2,"correct":[0,0,1,0]},"3073":{"type":"single","id":3073,"catId":0,"points":2,"correct":[1,0,0,0]}} } });




Day-615 | Daily MCQs | UPSC Prelims | ENVIRONMENT

Day-615

Time limit: 0

Quiz-summary

0 of 5 questions completed

Questions:

  1. 1
  2. 2
  3. 3
  4. 4
  5. 5

Information

DAILY MCQ

You have already completed the quiz before. Hence you can not start it again.

Quiz is loading...

You must sign in or sign up to start the quiz.

You have to finish following quiz, to start this quiz:

Results

0 of 5 questions answered correctly

Your time:

Time has elapsed

You have reached 0 of 0 points, (0)

Categories

  1. Not categorized 0%
  1. 1
  2. 2
  3. 3
  4. 4
  5. 5
  1. Answered
  2. Review
  1. Question 1 of 5
    1. Question

    1. Consider the following statements:
    Statement I: Regenerative agriculture emphasizes on revitalising the soil health and biodiversity.
    Statement II: Intensive farming releases carbon dioxide (CO2) into the atmosphere.
    Which one of the following is correct in respect of the above statements?

    Correct

    Answer. A
    Explanation:
    Statement 1 is correct: The key to regenerative agriculture is that it not only “does no harm” to the land but actually improves it, using technologies that regenerate and revitalize the soil and the environment. Regenerative agriculture leads to healthy soil, capable of producing high quality, nutrient dense food while simultaneously improving, rather than degrading land, and ultimately leading to productive farms and healthy communities and economies. It is dynamic and holistic, incorporating permaculture and organic farming practices, including conservation tillage, cover crops, crop rotation, composting, mobile animal shelters and pasture cropping, to increase food production, farmers’ income and especially, topsoil.
    Statement 2 is correct and explains statement 1: When land is degraded due to intensive farming, soil carbon can be released into the atmosphere along with nitrous oxide, making land degradation one of the biggest contributors to climate change. An estimated two-thirds of all terrestrial carbon stored in soils and vegetation have been lost since the 19th century through land degradation.

    Incorrect

    Answer. A
    Explanation:
    Statement 1 is correct: The key to regenerative agriculture is that it not only “does no harm” to the land but actually improves it, using technologies that regenerate and revitalize the soil and the environment. Regenerative agriculture leads to healthy soil, capable of producing high quality, nutrient dense food while simultaneously improving, rather than degrading land, and ultimately leading to productive farms and healthy communities and economies. It is dynamic and holistic, incorporating permaculture and organic farming practices, including conservation tillage, cover crops, crop rotation, composting, mobile animal shelters and pasture cropping, to increase food production, farmers’ income and especially, topsoil.
    Statement 2 is correct and explains statement 1: When land is degraded due to intensive farming, soil carbon can be released into the atmosphere along with nitrous oxide, making land degradation one of the biggest contributors to climate change. An estimated two-thirds of all terrestrial carbon stored in soils and vegetation have been lost since the 19th century through land degradation.

  2. Question 2 of 5
    2. Question

    2. With reference to the Prevention of Cruelty to Animals Act, 1960, consider the following statements:
    1. It led to the establishment of ‘Animal Welfare Board of India’.
    2. It does not allow the dehorning of cattle by its owner.
    3. It legalises the clinical trials on animals in the country.
    How many of the above statements are correct?

    Correct

    Answer. B
    Explanation:
    Statement 1 is correct: The Animal Welfare Board of India is a statutory advisory body on animal welfare which was established in 1962 by the central government under Section 4 of the Prevention of Cruelty to Animals Act, 1960. From ensuring that animal welfare laws in the country are diligently followed, to provide grants to Animal Welfare Organizations and advising the Government of India on animal welfare issues, the Board has been the face of the animal welfare movement in the country for the last 60 years. It was started under the stewardship of Late Smt. Rukmini Devi Arundale, well known humanitarian.
    Headquarters of AWBI are located in Ballabhgarh, Haryana (since 2018). Previously, it was based in Chennai, Tamil Nadu.
    Statement 2 is incorrect but statement 3 is correct: According to the provisions of the Prevention of Cruelty to Animals Act, 1960-
    Experiments involving operations on animals for the purpose of advancement by new discovery of physiological knowledge or of knowledge which will be useful for saving or for prolonging life or alleviating suffering or for combating any disease, whether of human beings, animals or plants, shall not be unlawful. Thus, clinical trials on animals for medicinal advancements are allowed. (Chapter IV deals with experimentation on animals).
    The penalties and prohibitions under the said Act do not apply in the following cases:
    ● the dehorning of cattle, or the castration or branding or nose-roping of any animal, in the prescribed manner; or
    ● the destruction of stray dogs in lethal chambers or [by such other methods as may be prescribed]; or
    ● the extermination or destruction of any animal under the authority of any law for the time being in force; or
    ● any matter dealt with in Chapter IV.
    Therefore, the above-mentioned activities are permitted under the Act.

    Incorrect

    Answer. B
    Explanation:
    Statement 1 is correct: The Animal Welfare Board of India is a statutory advisory body on animal welfare which was established in 1962 by the central government under Section 4 of the Prevention of Cruelty to Animals Act, 1960. From ensuring that animal welfare laws in the country are diligently followed, to provide grants to Animal Welfare Organizations and advising the Government of India on animal welfare issues, the Board has been the face of the animal welfare movement in the country for the last 60 years. It was started under the stewardship of Late Smt. Rukmini Devi Arundale, well known humanitarian.
    Headquarters of AWBI are located in Ballabhgarh, Haryana (since 2018). Previously, it was based in Chennai, Tamil Nadu.
    Statement 2 is incorrect but statement 3 is correct: According to the provisions of the Prevention of Cruelty to Animals Act, 1960-
    Experiments involving operations on animals for the purpose of advancement by new discovery of physiological knowledge or of knowledge which will be useful for saving or for prolonging life or alleviating suffering or for combating any disease, whether of human beings, animals or plants, shall not be unlawful. Thus, clinical trials on animals for medicinal advancements are allowed. (Chapter IV deals with experimentation on animals).
    The penalties and prohibitions under the said Act do not apply in the following cases:
    ● the dehorning of cattle, or the castration or branding or nose-roping of any animal, in the prescribed manner; or
    ● the destruction of stray dogs in lethal chambers or [by such other methods as may be prescribed]; or
    ● the extermination or destruction of any animal under the authority of any law for the time being in force; or
    ● any matter dealt with in Chapter IV.
    Therefore, the above-mentioned activities are permitted under the Act.

  3. Question 3 of 5
    3. Question

    3. Consider the following statements about ‘Open Natural Ecosystems (ONEs)’:
    1. They are clearly defined under the Environment (Protection) Act, 1986.
    2. Wastelands and grasslands are part of the ONEs.
    3. They play a crucial role in carbon sequestration.
    4. They are facing a growing threat from renewable energy.
    How many of the above statements are correct?

    Correct

    Answer. C
    Explanation:
    Statement 1 is incorrect: The term ‘Open Natural Ecosystems (ONEs)’ is not mentioned or defined in the Environment (Protection) Act, 1986. A clear definition of ONEs is lacking in India.
    Statement 2 is correct: Across the world, open natural ecosystems (also called ONEs) include cold and hot deserts, rock outcrops, boulder and rubble fields, wetlands and marshes, and diverse grasslands and savanna ecosystems. By some estimates, they occupy about two-thirds of land on Earth. Nearly 70% of the areas with open natural ecosystems overlap with those the government calls ‘wastelands’. This included ravines, grasslands, shrublands, waterlogged and marshy areas, pastures and even coastal areas.
    Statement 3 is correct: Grasslands, part of open natural ecosystems, sequester 146 tonnes of carbon per hectare per year and support 500 million livestock and over 20 nomadic tribes in India.
    Statement 4 is correct: Land-use change such as intensive agriculture, afforestation drives and renewable energy projects threaten open natural ecosystems (ONEs).

    Incorrect

    Answer. C
    Explanation:
    Statement 1 is incorrect: The term ‘Open Natural Ecosystems (ONEs)’ is not mentioned or defined in the Environment (Protection) Act, 1986. A clear definition of ONEs is lacking in India.
    Statement 2 is correct: Across the world, open natural ecosystems (also called ONEs) include cold and hot deserts, rock outcrops, boulder and rubble fields, wetlands and marshes, and diverse grasslands and savanna ecosystems. By some estimates, they occupy about two-thirds of land on Earth. Nearly 70% of the areas with open natural ecosystems overlap with those the government calls ‘wastelands’. This included ravines, grasslands, shrublands, waterlogged and marshy areas, pastures and even coastal areas.
    Statement 3 is correct: Grasslands, part of open natural ecosystems, sequester 146 tonnes of carbon per hectare per year and support 500 million livestock and over 20 nomadic tribes in India.
    Statement 4 is correct: Land-use change such as intensive agriculture, afforestation drives and renewable energy projects threaten open natural ecosystems (ONEs).

  4. Question 4 of 5
    4. Question

    4. Consider the following statements:
    1. Ethylene-vinyl acetate (EVA) and polyvinyl chloride (PVC) are commonly used in the footwear industry.
    2. The recycled plastic bottles and rubber tyres can make the footwear industry more eco-friendly.
    Which of the statements given above is/are incorrect?

    Correct

    Answer. D
    Explanation:
    Statement 1 is correct: The dominance of petrochemical-based plastics in the footwear segment is a major challenge in approaching footwear with circularity. The main byproducts of petrochemicals used in shoes for soles are polyurethane (PU), ethylene-vinyl acetate (EVA) and polyvinyl chloride (PVC), all known for value, durability and resistance. Be it EVA or PVC, these are non-biodegradable toxic materials, which after disintegration produce harmful material that goes into the soil.
    Statement 2 is correct: In recent years, footwear made of vegan material, recycled plastic bottles or upcycled rubber tyres with cork soles, has introduced a new vocabulary for footwear design among conscious fashion-goers.

    Incorrect

    Answer. D
    Explanation:
    Statement 1 is correct: The dominance of petrochemical-based plastics in the footwear segment is a major challenge in approaching footwear with circularity. The main byproducts of petrochemicals used in shoes for soles are polyurethane (PU), ethylene-vinyl acetate (EVA) and polyvinyl chloride (PVC), all known for value, durability and resistance. Be it EVA or PVC, these are non-biodegradable toxic materials, which after disintegration produce harmful material that goes into the soil.
    Statement 2 is correct: In recent years, footwear made of vegan material, recycled plastic bottles or upcycled rubber tyres with cork soles, has introduced a new vocabulary for footwear design among conscious fashion-goers.

  5. Question 5 of 5
    5. Question

    5. With reference to the Wetlands (Conservation and Management) Rules of 2010, consider the following statements:
    1. It is compulsory for the state governments to identify, demarcate and officially designate all wetlands across the state within a stipulated period of two years.
    2. These rules led to the establishment of Wetland Authority at national and state levels for the conservation and management of wetlands.
    Which of the statements given above is/are correct?

    Correct

    Answer. A
    Explanation:
    Statement 1 is correct: The Wetlands (Conservation and Management) Rules of 2010, enacted under the Environment Protection Act of 1986, make it compulsory for the state government to identify, demarcate and officially designate all wetlands across the state within a stipulated period of two years.
    Statement 2 is incorrect: The Wetlands (Conservation and Management) Rules amended in 2017 shifted the emphasis of wetland management from a central authority to state-level bodies. These rules provided for the establishment of National Wetland Authority and State Wetland Authorities to ensure more effective conservation and management of wetlands in the state.

    Incorrect

    Answer. A
    Explanation:
    Statement 1 is correct: The Wetlands (Conservation and Management) Rules of 2010, enacted under the Environment Protection Act of 1986, make it compulsory for the state government to identify, demarcate and officially designate all wetlands across the state within a stipulated period of two years.
    Statement 2 is incorrect: The Wetlands (Conservation and Management) Rules amended in 2017 shifted the emphasis of wetland management from a central authority to state-level bodies. These rules provided for the establishment of National Wetland Authority and State Wetland Authorities to ensure more effective conservation and management of wetlands in the state.

window.wpAdvQuizInitList = window.wpAdvQuizInitList || []; window.wpAdvQuizInitList.push({ id: '#wpAdvQuiz_656', init: { quizId: 656, mode: 0, globalPoints: 10, timelimit: 0, resultsGrade: [0], bo: 0, qpp: 0, catPoints: [10], formPos: 0, lbn: "Finish quiz", json: {"3064":{"type":"single","id":3064,"catId":0,"points":2,"correct":[1,0,0,0]},"3065":{"type":"single","id":3065,"catId":0,"points":2,"correct":[0,1,0,0]},"3066":{"type":"single","id":3066,"catId":0,"points":2,"correct":[0,0,1,0]},"3067":{"type":"single","id":3067,"catId":0,"points":2,"correct":[0,0,0,1]},"3068":{"type":"single","id":3068,"catId":0,"points":2,"correct":[1,0,0,0]}} } });




Day-614 | Daily MCQs | UPSC Prelims | CURRENT DEVELOPMENTS

Day-614

Time limit: 0

Quiz-summary

0 of 5 questions completed

Questions:

  1. 1
  2. 2
  3. 3
  4. 4
  5. 5

Information

DAILY MCQ

You have already completed the quiz before. Hence you can not start it again.

Quiz is loading...

You must sign in or sign up to start the quiz.

You have to finish following quiz, to start this quiz:

Results

0 of 5 questions answered correctly

Your time:

Time has elapsed

You have reached 0 of 0 points, (0)

Categories

  1. Not categorized 0%
  1. 1
  2. 2
  3. 3
  4. 4
  5. 5
  1. Answered
  2. Review
  1. Question 1 of 5
    1. Question

    1. Recently, the Pran Pratishtha ceremony of the Ram temple in Ayodhya was done by the Prime Minister of India, which is constructed in the Nagara style. Consider the following statements about the Nagara style of the temple architecture:
    1. The Nagara style of temple architecture emerged during the late Maurya period.
    2. Shikhara is the most distinguishable aspect of Nagara style temples.
    3. In these temples, circumambulatory passage around the garbha griha is absent.
    4. All north Indian temples are constructed in nagara style architecture.
    How many of the statements given above are incorrect?

    Correct

    Answer: C
    Context: The Ram Temple in Ayodhya was inaugurated on 22th Jan 2024 Chandrakant Sompura, and his son Ashish created the structure in the Nagara style of temple construction.
    Statement 1 is incorrect: The Nagara style of temple architecture emerged some time in the 5th century CE, during the late Gupta period, in northern India.
    Statement 2 is correct: Nagara temples are built on a raised plinth, with the garbha griha (sanctum sanctorum) where the idol of the deity rests on the most sacred part of the temple. Towering over the garbha griha is the shikhara (literally ‘mountain peak’), the most distinguishable aspect of Nagara style temples.
    Basics of Nagara style. (Express Sketch)
    Statement 3 is incorrect: A typical Nagara style temple also comprises a circumambulatory passage around the garbha griha, and one or more mandapas (halls) on the same axis.
    Statement 4 is incorrect: All north Indian temples constructed in Nagara style architecture are not entirely accurate. While Nagara is indeed a prominent and historically significant style in the north, it’s not the only one present.

    Incorrect

    Answer: C
    Context: The Ram Temple in Ayodhya was inaugurated on 22th Jan 2024 Chandrakant Sompura, and his son Ashish created the structure in the Nagara style of temple construction.
    Statement 1 is incorrect: The Nagara style of temple architecture emerged some time in the 5th century CE, during the late Gupta period, in northern India.
    Statement 2 is correct: Nagara temples are built on a raised plinth, with the garbha griha (sanctum sanctorum) where the idol of the deity rests on the most sacred part of the temple. Towering over the garbha griha is the shikhara (literally ‘mountain peak’), the most distinguishable aspect of Nagara style temples.
    Basics of Nagara style. (Express Sketch)
    Statement 3 is incorrect: A typical Nagara style temple also comprises a circumambulatory passage around the garbha griha, and one or more mandapas (halls) on the same axis.
    Statement 4 is incorrect: All north Indian temples constructed in Nagara style architecture are not entirely accurate. While Nagara is indeed a prominent and historically significant style in the north, it’s not the only one present.

  2. Question 2 of 5
    2. Question

    2. Which of the following organizations publishes the ‘Enabling Trade Report’?

    Correct

    Answer: B
    Context: Recently, the annual meeting of the World Economic Forum was held in Davos from 15-19 January 2024. They have issued a biennially published report, the Enabling Trade Report. The report estimates that world economic growth will slow from 3% in 2022 to 2.4% in 2023, with some signs of recovery next year. The report is based on a survey of businesses and experts from 136 countries.
    India’s overall exports (Merchandise and Services combined) in November 2023 is estimated to be USD 62.58 Billion, exhibiting a positive growth of 1.23 per cent.
    Hence, Option (b) is the correct answer.
    Additional Information
    Reports published by WEF
    ● Global Gender Gap Report,
    ● Global Risk Report,
    ● Global Travel and Tourism,
    ● Global Competitiveness Report,
    ● Energy Transition Index,
    ● Global IT Report,

    Incorrect

    Answer: B
    Context: Recently, the annual meeting of the World Economic Forum was held in Davos from 15-19 January 2024. They have issued a biennially published report, the Enabling Trade Report. The report estimates that world economic growth will slow from 3% in 2022 to 2.4% in 2023, with some signs of recovery next year. The report is based on a survey of businesses and experts from 136 countries.
    India’s overall exports (Merchandise and Services combined) in November 2023 is estimated to be USD 62.58 Billion, exhibiting a positive growth of 1.23 per cent.
    Hence, Option (b) is the correct answer.
    Additional Information
    Reports published by WEF
    ● Global Gender Gap Report,
    ● Global Risk Report,
    ● Global Travel and Tourism,
    ● Global Competitiveness Report,
    ● Energy Transition Index,
    ● Global IT Report,

  3. Question 3 of 5
    3. Question

    3. With reference to the ‘Archaeological Survey of India’ (ASI), consider the following statements:
    1. The prime concern of the ASI is the maintenance of ancient monuments and archaeological sites and remains of national importance.
    2. It does not oversee the implementation of the Antiquities and Art Treasure Act, of 1972.
    3. Alexander Cunningham was appointed as the first Director-General of ASI.
    How many of the statements given above are correct?

    Correct

    Answer: B
    Context: Recently, the Varanasi district court allowed the Hindu side to offer prayers in the southern basement, also known as ‘Vyas ka Tahkhana’, inside the Gyanvapi Mosque. Here is the ASI survey report about the Gyanvapi Mosque. The Archaeological Survey of India (ASI), which is part of the Ministry of Culture, is the premier organization for archaeological research and the preservation of the nation’s cultural heritage.
    Explanation
    Statement 1 is correct: The prime concern of the ASI is maintenance of ancient monuments and archaeological sites and remains of national importance.
    Statement 2 is incorrect: Furthermore, it governs all archaeological activities in the country in accordance with the provisions of the Ancient Monuments and Archaeological Sites and Remains Act of 1958. It also oversees the Antiquities and Art Treasure Act of 1972.
    Statement 3 is correct: By 1851, the British government in India saw the need for a systematic archaeological survey of the country. Consequently, the ASI was established in 1861. Alexander Cunningham, a protege of Princep, was appointed as the first director-general of ASI.

    Incorrect

    Answer: B
    Context: Recently, the Varanasi district court allowed the Hindu side to offer prayers in the southern basement, also known as ‘Vyas ka Tahkhana’, inside the Gyanvapi Mosque. Here is the ASI survey report about the Gyanvapi Mosque. The Archaeological Survey of India (ASI), which is part of the Ministry of Culture, is the premier organization for archaeological research and the preservation of the nation’s cultural heritage.
    Explanation
    Statement 1 is correct: The prime concern of the ASI is maintenance of ancient monuments and archaeological sites and remains of national importance.
    Statement 2 is incorrect: Furthermore, it governs all archaeological activities in the country in accordance with the provisions of the Ancient Monuments and Archaeological Sites and Remains Act of 1958. It also oversees the Antiquities and Art Treasure Act of 1972.
    Statement 3 is correct: By 1851, the British government in India saw the need for a systematic archaeological survey of the country. Consequently, the ASI was established in 1861. Alexander Cunningham, a protege of Princep, was appointed as the first director-general of ASI.

  4. Question 4 of 5
    4. Question

    4. Consider the following statements with reference to the One Stop Centre Scheme:
    1. It is a central sector scheme.
    2. It ensures women’s security in any public place.
    3. It supports women affected by domestic violence.
    How many of the statements given above are correct?

    Correct

    Answer: A
    Context: The Finance Minister of India, said on 31st Jan 2024 that one-stop centers have been set up in more than 700 districts across the country to help women facing violence and about 8.3 lakh women have got help through them.
    Explanation:
    Statement 1 is incorrect: Ministry of Women and Child Development (MWCD) has formulated a Centrally Sponsored Scheme for setting up One Stop Centres (OSC), to be funded from the Nirbhaya Fund.
    Statement 2 is incorrect: one stop centers (OSCs) have been set up across the country to provide integrated support and assistance under one roof to women affected by violence. Also, universalization of women helpline (181). The target group includes all women, including girls under 18 years of age, affected by violence, regardless of class, religion, region, sexual orientation or marital status. It is not mentioned about absolute women security in public places.
    Statement 3 is correct: One-Stop Centre scheme is a sub-scheme of the National Mission for Empowerment of Women which also includes the Indira Gandhi Matritva Sahayog Yojana. The scheme is funded by the Nirbhaya fund. Under this scheme, complete financial assistance will be provided by the Centre to the States and the Union territories. The intention of the One-Stop Scheme (OSC) is to support women affected by the violence that they may face within the family or at the workplace or within the community, in private or public places.

    Incorrect

    Answer: A
    Context: The Finance Minister of India, said on 31st Jan 2024 that one-stop centers have been set up in more than 700 districts across the country to help women facing violence and about 8.3 lakh women have got help through them.
    Explanation:
    Statement 1 is incorrect: Ministry of Women and Child Development (MWCD) has formulated a Centrally Sponsored Scheme for setting up One Stop Centres (OSC), to be funded from the Nirbhaya Fund.
    Statement 2 is incorrect: one stop centers (OSCs) have been set up across the country to provide integrated support and assistance under one roof to women affected by violence. Also, universalization of women helpline (181). The target group includes all women, including girls under 18 years of age, affected by violence, regardless of class, religion, region, sexual orientation or marital status. It is not mentioned about absolute women security in public places.
    Statement 3 is correct: One-Stop Centre scheme is a sub-scheme of the National Mission for Empowerment of Women which also includes the Indira Gandhi Matritva Sahayog Yojana. The scheme is funded by the Nirbhaya fund. Under this scheme, complete financial assistance will be provided by the Centre to the States and the Union territories. The intention of the One-Stop Scheme (OSC) is to support women affected by the violence that they may face within the family or at the workplace or within the community, in private or public places.

  5. Question 5 of 5
    5. Question

    5. With reference to the “Invasive Alien Species”, consider the following statements:
    1. It is the major factor behind the global animals and plant extinctions.
    2. The majority of the impacts of biological invasions on native species are positive.
    3. The water hyacinth is the world’s most widespread invasive alien species on land.
    How many of the statements given above are correct?

    Correct

    Answer: B
    Context: This assessment report on invasive alien species and their control, recently published by the Intergovernmental Platform on Biodiversity and Ecosystem Services (IPBES), is the most comprehensive study to date, covering 37,000 introduced alien species, including plants and animals, and 3,500 Covers regions and biomes around the world, including more invasive alien species.
    Explanation:
    Statement 1 is correct: Invasive alien species have been a major factor in 60% and the only driver in 16% of global animal and plant extinctions that we have recorded. About 85% of the impacts of biological invasions on native species are negative,” said co-chair of the Assessment.
    Statement 2 is incorrect: About 6% of alien plants; 22% of alien invertebrates; 14% of alien vertebrates; and 11% of alien microbes are known to be invasive, posing major risks to nature and to people, the IPBES has said. Nearly 80% of the documented impacts of invasive species on nature’s contribution to people are negative.
    Statement 3 is correct: The water hyacinth is the world’s most widespread invasive alien species on land. Lantana, a flowering shrub, and the black rat are the second and third most widespread globally. The brown rat and the house mouse are also widespread invasive alien species.

    Incorrect

    Answer: B
    Context: This assessment report on invasive alien species and their control, recently published by the Intergovernmental Platform on Biodiversity and Ecosystem Services (IPBES), is the most comprehensive study to date, covering 37,000 introduced alien species, including plants and animals, and 3,500 Covers regions and biomes around the world, including more invasive alien species.
    Explanation:
    Statement 1 is correct: Invasive alien species have been a major factor in 60% and the only driver in 16% of global animal and plant extinctions that we have recorded. About 85% of the impacts of biological invasions on native species are negative,” said co-chair of the Assessment.
    Statement 2 is incorrect: About 6% of alien plants; 22% of alien invertebrates; 14% of alien vertebrates; and 11% of alien microbes are known to be invasive, posing major risks to nature and to people, the IPBES has said. Nearly 80% of the documented impacts of invasive species on nature’s contribution to people are negative.
    Statement 3 is correct: The water hyacinth is the world’s most widespread invasive alien species on land. Lantana, a flowering shrub, and the black rat are the second and third most widespread globally. The brown rat and the house mouse are also widespread invasive alien species.

window.wpAdvQuizInitList = window.wpAdvQuizInitList || []; window.wpAdvQuizInitList.push({ id: '#wpAdvQuiz_655', init: { quizId: 655, mode: 0, globalPoints: 10, timelimit: 0, resultsGrade: [0], bo: 0, qpp: 0, catPoints: [10], formPos: 0, lbn: "Finish quiz", json: {"3059":{"type":"single","id":3059,"catId":0,"points":2,"correct":[0,0,1,0]},"3060":{"type":"single","id":3060,"catId":0,"points":2,"correct":[0,1,0,0]},"3061":{"type":"single","id":3061,"catId":0,"points":2,"correct":[0,1,0,0]},"3062":{"type":"single","id":3062,"catId":0,"points":2,"correct":[1,0,0,0]},"3063":{"type":"single","id":3063,"catId":0,"points":2,"correct":[0,1,0,0]}} } });




Day-613 | Daily MCQs | UPSC Prelims | ECONOMY

Day-613

Time limit: 0

Quiz-summary

0 of 5 questions completed

Questions:

  1. 1
  2. 2
  3. 3
  4. 4
  5. 5

Information

DAILY MCQ

You have already completed the quiz before. Hence you can not start it again.

Quiz is loading...

You must sign in or sign up to start the quiz.

You have to finish following quiz, to start this quiz:

Results

0 of 5 questions answered correctly

Your time:

Time has elapsed

You have reached 0 of 0 points, (0)

Categories

  1. Not categorized 0%
  1. 1
  2. 2
  3. 3
  4. 4
  5. 5
  1. Answered
  2. Review
  1. Question 1 of 5
    1. Question

    1. Consider the following statements with reference to ‘side-pocketing’ in financial markets:
    1. It is an accounting technique used in debt mutual funds to segregate the bad assets within the fund.
    2. The Securities and Exchange Board of India (SEBI) has not permitted this practice.
    Which of the statements given above is/are correct?

    Correct

    Answer: A
    Explanation:
    Statement 1 is correct: Side pocketing is a technique to safeguard investors in instruments that have exposure to risky assets. It is basically an accounting method that is used to separate illiquid investments from liquid and quality investments, in a debt portfolio.
    ● When side pocketing is implemented, and illiquid assets are pushed in a separate pocket, the fund’s NAV (net asset value) reflects the value of liquid assets only — a different NAV assigned to the side pocket assets based on an estimate of the realizable value of investors.
    ● With the help of side pocketing, risky bets can be segregated from safer and liquid investments that may get impacted due to changing credit profiles of risky assets.
    Statement 2 is incorrect: India’s financial market regulator, SEBI, allowed side pocketing in India in 2018. In 2018, many debt schemes saw a sharp fall in the NAV. After these schemes, investments in Infrastructure Leasing & Financial Services Ltd (IL&FS) and some of its subsidiaries saw credit rating downgrades.

    Incorrect

    Answer: A
    Explanation:
    Statement 1 is correct: Side pocketing is a technique to safeguard investors in instruments that have exposure to risky assets. It is basically an accounting method that is used to separate illiquid investments from liquid and quality investments, in a debt portfolio.
    ● When side pocketing is implemented, and illiquid assets are pushed in a separate pocket, the fund’s NAV (net asset value) reflects the value of liquid assets only — a different NAV assigned to the side pocket assets based on an estimate of the realizable value of investors.
    ● With the help of side pocketing, risky bets can be segregated from safer and liquid investments that may get impacted due to changing credit profiles of risky assets.
    Statement 2 is incorrect: India’s financial market regulator, SEBI, allowed side pocketing in India in 2018. In 2018, many debt schemes saw a sharp fall in the NAV. After these schemes, investments in Infrastructure Leasing & Financial Services Ltd (IL&FS) and some of its subsidiaries saw credit rating downgrades.

  2. Question 2 of 5
    2. Question

    2. Which of the following best describes the term ‘Tobin Tax’?

    Correct

    Answer: A
    Explanation:
    ● Tobin tax is a duty proposed on spot currency trades to penalize short-term currency trading to stabilize markets and disincentive speculation.
    ● It can be used to generate revenue streams for countries that see a great deal of short-term currency movement.
    ● It is sometimes referred to as the Robin Hood tax, as many see it as a way for governments to take small amounts of money from the people making large, short-term currency exchanges.
    ● It was proposed by James Tobin in 1972, when the fixed exchange rates under the Bretton Woods system were replaced with flexible exchange rates in 1971, and there was a massive movement of funds between different currencies that threatened to destabilize the economy.

    Incorrect

    Answer: A
    Explanation:
    ● Tobin tax is a duty proposed on spot currency trades to penalize short-term currency trading to stabilize markets and disincentive speculation.
    ● It can be used to generate revenue streams for countries that see a great deal of short-term currency movement.
    ● It is sometimes referred to as the Robin Hood tax, as many see it as a way for governments to take small amounts of money from the people making large, short-term currency exchanges.
    ● It was proposed by James Tobin in 1972, when the fixed exchange rates under the Bretton Woods system were replaced with flexible exchange rates in 1971, and there was a massive movement of funds between different currencies that threatened to destabilize the economy.

  3. Question 3 of 5
    3. Question

    3. With respect to the foreign banks in India, consider the following statements:
    1. There are more foreign banks than public sector banks.
    2. Foreign banks are not subject to priority sector lending targets.
    3. They are subject to Basel capital framework norms.
    How many of the above statements are correct?

    Correct

    Answer: B
    Explanation:
    Statement 1 is correct: In the Indian banking system, foreign banks are more than public sector banks. At present, while there are 44 foreign banks operating in India, there are only 12 public sector banks.
    Statement 2 is incorrect: Foreign banks, with more than 20 branches, are also subject to priority sector lending targets.
    ● Reserve Bank of India (RBI) regulations require banks to allocate 40% of their adjusted net bank credit (ANBC), for the priority sector, comprising agriculture, small and medium enterprises, exports, and economically vulnerable groups such as small farmers, micro-enterprises, and disadvantaged segments.
    ● Other than lending, these norms also prescribe banks to ensure that the loans extended under PSL are for approved purposes and the end use is continuously monitored. For this, banks are advised to put in place a proper internal control system.
    Statement 3 is correct: In India, all the scheduled commercial banks, comprising public sector banks, private sector banks and foreign banks, but excluding regional rural banks, fall under the purview of Basel III regulations.
    ● The global financial crisis of 2008, triggered by the Lehman Brothers’ collapse, set alarm bells ringing for financial institutions.
    ● The Basel III norms are a comprehensive set of reform measures which are designed & developed by the Basel Committee on Banking Supervision with an aim to strengthen the regulation, supervision and risk management of the banking sector (BCBS). The norms call for improvement of the quantity and quality of capital of banks, stronger supervision, and more stringent risk management and disclosure standards.

    Incorrect

    Answer: B
    Explanation:
    Statement 1 is correct: In the Indian banking system, foreign banks are more than public sector banks. At present, while there are 44 foreign banks operating in India, there are only 12 public sector banks.
    Statement 2 is incorrect: Foreign banks, with more than 20 branches, are also subject to priority sector lending targets.
    ● Reserve Bank of India (RBI) regulations require banks to allocate 40% of their adjusted net bank credit (ANBC), for the priority sector, comprising agriculture, small and medium enterprises, exports, and economically vulnerable groups such as small farmers, micro-enterprises, and disadvantaged segments.
    ● Other than lending, these norms also prescribe banks to ensure that the loans extended under PSL are for approved purposes and the end use is continuously monitored. For this, banks are advised to put in place a proper internal control system.
    Statement 3 is correct: In India, all the scheduled commercial banks, comprising public sector banks, private sector banks and foreign banks, but excluding regional rural banks, fall under the purview of Basel III regulations.
    ● The global financial crisis of 2008, triggered by the Lehman Brothers’ collapse, set alarm bells ringing for financial institutions.
    ● The Basel III norms are a comprehensive set of reform measures which are designed & developed by the Basel Committee on Banking Supervision with an aim to strengthen the regulation, supervision and risk management of the banking sector (BCBS). The norms call for improvement of the quantity and quality of capital of banks, stronger supervision, and more stringent risk management and disclosure standards.

  4. Question 4 of 5
    4. Question

    4. With reference to the Micro, Small and Medium Enterprises (MSMEs) sector in India, consider the following statements:
    1. It contributes less than 40% to India’s overall exports.
    2. It contributes more than 40% to India’s overall manufacturing output.
    Which of the statements given above is/are correct?

    Correct

    Answer: D
    Explanation:
    The Micro, Small and Medium Enterprises (MSMEs) sector plays an important role in the growth of the Indian economy.
    Statement 1 is incorrect. The share of MSME-specified goods in overall exports stood at 43.6% in FY23. It was 45.03% in FY22, 49.35 per cent in FY21 and 49.77 per cent in FY20.
    As can be seen, the share of MSMEs in overall exports has been declining. The decline in export share is contrary to the government’s target. Former MSME Minister in 2020 had set a target of increasing MSMEs’ share in exports to 60% in five years.
    Statement 2 is incorrect: The share of MSME manufacturing output in all India manufacturing output during the year 2019-20, 2020-21 and 2021-22 was 36.6%, 36.9% and 36.2% respectively. Hence, it has been less than 40%.

    Incorrect

    Answer: D
    Explanation:
    The Micro, Small and Medium Enterprises (MSMEs) sector plays an important role in the growth of the Indian economy.
    Statement 1 is incorrect. The share of MSME-specified goods in overall exports stood at 43.6% in FY23. It was 45.03% in FY22, 49.35 per cent in FY21 and 49.77 per cent in FY20.
    As can be seen, the share of MSMEs in overall exports has been declining. The decline in export share is contrary to the government’s target. Former MSME Minister in 2020 had set a target of increasing MSMEs’ share in exports to 60% in five years.
    Statement 2 is incorrect: The share of MSME manufacturing output in all India manufacturing output during the year 2019-20, 2020-21 and 2021-22 was 36.6%, 36.9% and 36.2% respectively. Hence, it has been less than 40%.

  5. Question 5 of 5
    5. Question

    5. Consider the following statements:
    Statement-I: Effective revenue deficit excludes the grants in aid for the creation of assets in states.
    Statement-II: It was suggested by the Rangarajan Committee on public expenditure.
    Which one of the following is correct in respect of the above statements?

    Correct

    Answer: B
    Explanation:
    Both Statement-I and Statement-II are correct and Statement-II is not the correct explanation for Statement-I
    Statement 1 is correct: Revenue deficit arises when the government’s revenue expenditure exceeds the total revenue receipts. Revenue deficit includes those transactions that have a direct impact on a government’s current income and expenditure.
    Effective revenue deficit is the difference between revenue deficit and grants for creation of capital assets.
    Statement 2 is correct: The concept of effective revenue deficit has been suggested by the Rangarajan Committee on Public Expenditure.
    The concept was introduced to ascertain the actual deficit in the revenue account after adjusting for expenditure of capital nature.

    Incorrect

    Answer: B
    Explanation:
    Both Statement-I and Statement-II are correct and Statement-II is not the correct explanation for Statement-I
    Statement 1 is correct: Revenue deficit arises when the government’s revenue expenditure exceeds the total revenue receipts. Revenue deficit includes those transactions that have a direct impact on a government’s current income and expenditure.
    Effective revenue deficit is the difference between revenue deficit and grants for creation of capital assets.
    Statement 2 is correct: The concept of effective revenue deficit has been suggested by the Rangarajan Committee on Public Expenditure.
    The concept was introduced to ascertain the actual deficit in the revenue account after adjusting for expenditure of capital nature.

window.wpAdvQuizInitList = window.wpAdvQuizInitList || []; window.wpAdvQuizInitList.push({ id: '#wpAdvQuiz_654', init: { quizId: 654, mode: 0, globalPoints: 10, timelimit: 0, resultsGrade: [0], bo: 0, qpp: 0, catPoints: [10], formPos: 0, lbn: "Finish quiz", json: {"3054":{"type":"single","id":3054,"catId":0,"points":2,"correct":[1,0,0,0]},"3055":{"type":"single","id":3055,"catId":0,"points":2,"correct":[1,0,0,0]},"3056":{"type":"single","id":3056,"catId":0,"points":2,"correct":[0,1,0,0]},"3057":{"type":"single","id":3057,"catId":0,"points":2,"correct":[0,0,0,1]},"3058":{"type":"single","id":3058,"catId":0,"points":2,"correct":[0,1,0,0]}} } });




Day-612 | Daily MCQs | UPSC Prelims | GEOGRAPHY

Day-612

Time limit: 0

Quiz-summary

0 of 5 questions completed

Questions:

  1. 1
  2. 2
  3. 3
  4. 4
  5. 5

Information

DAILY MCQ

You have already completed the quiz before. Hence you can not start it again.

Quiz is loading...

You must sign in or sign up to start the quiz.

You have to finish following quiz, to start this quiz:

Results

0 of 5 questions answered correctly

Your time:

Time has elapsed

You have reached 0 of 0 points, (0)

Categories

  1. Not categorized 0%
  1. 1
  2. 2
  3. 3
  4. 4
  5. 5
  1. Answered
  2. Review
  1. Question 1 of 5
    1. Question

    1. Consider the following pairs about water bodies around the world:
    Water bodies in news – Countries
    1. Lake Retba – Senegal
    2. Mahmudia Wetlands – Yemen
    3. Lake Titicaca – Chile
    How many of the above pairs are correctly matched?

    Correct

    Answer: A
    Explanation:
    Pair 1 is matched correctly:
    ● Lake Retba, better known as Lac Rose (the Pink Lake), is located around 35km from the city of Dakar, Senegal. It sits in a depression with a shoreline 6.5 metres below sea level.
    ● The lake’s waters are virtually devoid of life, with the exception of a few microscopic algae and bacteria.
    ● It’s losing its appeal for a number of reasons. Apart from tourism, artisanal salt mining has been one of the dominant activities around the lake.
    Why does the water turn pink?
    ● The pink coloration is due to the proliferation of halophilic green algae (living in a salty environment), Dunaliella salina, which contain red pigments. The alga is associated with halophilic bacteria of the genus Halobacterium. This microscopic alga’s resistance to salt comes from its high concentration of carotenoid pigments, which protect it from light, and its high glycerol content.
    ● In fact, Dunaliella salina contains at least four antioxidant pigments (beta-carotene, astaxanthin, lutein and zeaxanthin), which are rich in vitamins and trace elements. When salinity is high, algae with red pigments thrive, and when salinity is low, they give way to other algae rich in green pigments.

    Pair 2 is matched incorrectly:
    ● Wildlife conservation organisation World Wildlife Fund (WWF) has urged the Romanian government to classify Mahmudia wetland as a ‘national interest ecological restoration area’ to protect its natural progress and foster community prosperity.
    ● The Mahmudia wetland is situated in Romania’s Danube Delta.
    ● It is facing a critical threat as agricultural leaseholders seek to revert a significant portion of the naturally restored wetlands to cropland.

    ● This move poses a risk to the biodiversity, livelihoods, and climate resilience that the wetland currently provides.
    Pair 3 is matched incorrectly:
    ● Lake Titicaca is facing a serious threat from climate change and drought.
    ● It is the largest freshwater lake in South America and the highest of the world’s largest lakes.
    ● The lake, which lies on the border between Bolivia and Peru, has seen its water levels drop to near-record lows.
    ● The lack of rainfall and the increased evaporation due to rising temperatures have reduced the inflow and volume of the lake.
    ● This has resulted in stranded boats, exposed shorelines, and diminished fish populations.
    ● The lake is also home to more than 500 species of plants and animals, some of which are endemic and endangered.

    Incorrect

    Answer: A
    Explanation:
    Pair 1 is matched correctly:
    ● Lake Retba, better known as Lac Rose (the Pink Lake), is located around 35km from the city of Dakar, Senegal. It sits in a depression with a shoreline 6.5 metres below sea level.
    ● The lake’s waters are virtually devoid of life, with the exception of a few microscopic algae and bacteria.
    ● It’s losing its appeal for a number of reasons. Apart from tourism, artisanal salt mining has been one of the dominant activities around the lake.
    Why does the water turn pink?
    ● The pink coloration is due to the proliferation of halophilic green algae (living in a salty environment), Dunaliella salina, which contain red pigments. The alga is associated with halophilic bacteria of the genus Halobacterium. This microscopic alga’s resistance to salt comes from its high concentration of carotenoid pigments, which protect it from light, and its high glycerol content.
    ● In fact, Dunaliella salina contains at least four antioxidant pigments (beta-carotene, astaxanthin, lutein and zeaxanthin), which are rich in vitamins and trace elements. When salinity is high, algae with red pigments thrive, and when salinity is low, they give way to other algae rich in green pigments.

    Pair 2 is matched incorrectly:
    ● Wildlife conservation organisation World Wildlife Fund (WWF) has urged the Romanian government to classify Mahmudia wetland as a ‘national interest ecological restoration area’ to protect its natural progress and foster community prosperity.
    ● The Mahmudia wetland is situated in Romania’s Danube Delta.
    ● It is facing a critical threat as agricultural leaseholders seek to revert a significant portion of the naturally restored wetlands to cropland.

    ● This move poses a risk to the biodiversity, livelihoods, and climate resilience that the wetland currently provides.
    Pair 3 is matched incorrectly:
    ● Lake Titicaca is facing a serious threat from climate change and drought.
    ● It is the largest freshwater lake in South America and the highest of the world’s largest lakes.
    ● The lake, which lies on the border between Bolivia and Peru, has seen its water levels drop to near-record lows.
    ● The lack of rainfall and the increased evaporation due to rising temperatures have reduced the inflow and volume of the lake.
    ● This has resulted in stranded boats, exposed shorelines, and diminished fish populations.
    ● The lake is also home to more than 500 species of plants and animals, some of which are endemic and endangered.

  2. Question 2 of 5
    2. Question

    2. Consider the following statements regarding the rivers originating from Hindu Kush Himalayan (HKH) region:
    1. The Amu Darya River flows into the Aral Sea.
    2. The Tarim River flows into the Taklamakan Desert.
    3. The Yellow River flows into the Gulf of Bohai.
    4. The Mekong River flows into the Gulf of Thailand.
    How many of the above statements are correct?

    Correct

    Answer: C
    Explanation:
    Statement 1 is correct:
    ● The Amu Darya, historically known as the Oxus River, is a major river in Central Asia and Afghanistan.
    ● Rising in the Pamir Mountains, north of the Hindu Kush, the Amu Darya is formed by the confluence of the Vakhsh and Panj rivers, in the Tigrovaya Balka Nature Reserve on the border between Afghanistan and Tajikistan, and flows from there north-westwards into the southern remnants of the Aral Sea.

    Statement 2 is correct:
    ● The Tarim River is a river in Xinjiang, China.
    ● It is the longest inland river in China.
    ● It is the principal river of the Tarim Basin, a desert region of Central Asia between the Tian Shan and Kunlun Mountains.
    ● The river historically terminated at Lop Nur, but today reaches no further than Taitema Lake before drying out in Taklamakan Desert.

    Statement 3 is correct:
    ● The Yellow River (Huang He) is the second longest river in China (after the Yangtze).
    ● It’s the fifth-longest river in the world.
    ● Source: The Bayankala Mountains on the Plateau of Tibet in western central China.
    ● Mouth: Southern Bohai Sea
    ● Claims to fame: world’s muddiest major river, “China’s cradle (of civilization)”
    ● The name “Yellow River” comes from the huge amounts of “yellow” loess sediment it carries, which are eroded when it flows through the Loess Plateau.

    Statement 4 is incorrect:
    ● The Mekong River is a trans-boundary river in East Asia and Southeast Asia.
    ● It is the world’s twelfth-longest river and the third-longest in Asia with an estimated length of 4,909 km.
    ● From its headwaters in the Tibetan Plateau, the river runs through Southwest China (where it is officially called the Lancang River), Myanmar, Laos, Thailand, Cambodia, and southern Vietnam.
    ● Even so, the river is a major trade route between Tibet and Southeast Asia.
    ● Mouth of the river: South China Sea

    Incorrect

    Answer: C
    Explanation:
    Statement 1 is correct:
    ● The Amu Darya, historically known as the Oxus River, is a major river in Central Asia and Afghanistan.
    ● Rising in the Pamir Mountains, north of the Hindu Kush, the Amu Darya is formed by the confluence of the Vakhsh and Panj rivers, in the Tigrovaya Balka Nature Reserve on the border between Afghanistan and Tajikistan, and flows from there north-westwards into the southern remnants of the Aral Sea.

    Statement 2 is correct:
    ● The Tarim River is a river in Xinjiang, China.
    ● It is the longest inland river in China.
    ● It is the principal river of the Tarim Basin, a desert region of Central Asia between the Tian Shan and Kunlun Mountains.
    ● The river historically terminated at Lop Nur, but today reaches no further than Taitema Lake before drying out in Taklamakan Desert.

    Statement 3 is correct:
    ● The Yellow River (Huang He) is the second longest river in China (after the Yangtze).
    ● It’s the fifth-longest river in the world.
    ● Source: The Bayankala Mountains on the Plateau of Tibet in western central China.
    ● Mouth: Southern Bohai Sea
    ● Claims to fame: world’s muddiest major river, “China’s cradle (of civilization)”
    ● The name “Yellow River” comes from the huge amounts of “yellow” loess sediment it carries, which are eroded when it flows through the Loess Plateau.

    Statement 4 is incorrect:
    ● The Mekong River is a trans-boundary river in East Asia and Southeast Asia.
    ● It is the world’s twelfth-longest river and the third-longest in Asia with an estimated length of 4,909 km.
    ● From its headwaters in the Tibetan Plateau, the river runs through Southwest China (where it is officially called the Lancang River), Myanmar, Laos, Thailand, Cambodia, and southern Vietnam.
    ● Even so, the river is a major trade route between Tibet and Southeast Asia.
    ● Mouth of the river: South China Sea

  3. Question 3 of 5
    3. Question

    3. Consider the following statements about traditional cultivation systems:
    1. Akkadi Saalu is an intercropping system that has been in traditional practice in Kerala.
    2. Khandagari is a traditional shifting cultivation system in North East India.
    3. The Kuttanad farming system is the practice of rice cultivation below sea level in coastal Karnataka.
    How many of the above statements are correct?

    Correct

    Answer: D
    Explanation:
    Statement 1 is incorrect:
    ● AKKADI SAALU (Diversified Farming Systems) is a traditional culture of farming by growing diversified crops in a given plot under rain-fed conditions.
    ● This traditional cultivation practice is largely practiced in interior Karnataka.
    ● It is also connecting monsoon and rabi cropping seasons, where the rainfall is effectively utilized to secure varieties of food – fodder – fiber crops in a given land.
    ● Securing nutritional food security for soil microbes – plants – domestic animals and farming communities is the goal of AKKADI SAALU.
    ● AKKADI SAALU METHOD ensures one or the other crops above the soil for more than eight months in a year to conserve the soil moisture, enhance the nutrients status of soil, control the erosion losses, suppress the weeds in crop plants, and remove the residual effects of pesticides, fertilizers, and heavy metals from earlier use.
    Statement 2 is incorrect:
    Kandhagari traditional cultivation system:
    ● Wayanad is a place in Kerala which is characterized by its traditional shifting cultivation system called “Kandhagari”, which involves the rotation of crops like rice, millets and tubers.
    ● This system has helped maintain biodiversity and is deeply rooted in the culture of the indigenous communities.
    Statement 3 is incorrect:
    Kuttanad below sea level farming system:
    ● The only system in India that has been practicing rice cultivation below sea level since the past 2 centuries.
    ● Kuttanad is a delta region of about 900 sq. km situated on the west coast of Kerala State, India. The area is a larger mosaic of fragmented landscape patches and varied ecosystems such as coastal backwaters, rivers, vast stretches of paddy fields, marshes, ponds, garden lands, edges, corridors and remarkably networked waterways.
    ● The Kuttanad Below Sea-level Farming System (KBSFS) is unique, as it is the only system in India that practices rice cultivation below sea level. The major land use structure of KBSFS is flat stretches of rice fields in about 50,000 ha of mostly reclaimed delta swamps. The rice fields, which are popularly known as “Puncha Vayals’ ‘ exist in three landscape elements: Karapadam (upland rice fields), Kayal (wetland rice fields) and Kari (land buried with black coal-like materials).

    Incorrect

    Answer: D
    Explanation:
    Statement 1 is incorrect:
    ● AKKADI SAALU (Diversified Farming Systems) is a traditional culture of farming by growing diversified crops in a given plot under rain-fed conditions.
    ● This traditional cultivation practice is largely practiced in interior Karnataka.
    ● It is also connecting monsoon and rabi cropping seasons, where the rainfall is effectively utilized to secure varieties of food – fodder – fiber crops in a given land.
    ● Securing nutritional food security for soil microbes – plants – domestic animals and farming communities is the goal of AKKADI SAALU.
    ● AKKADI SAALU METHOD ensures one or the other crops above the soil for more than eight months in a year to conserve the soil moisture, enhance the nutrients status of soil, control the erosion losses, suppress the weeds in crop plants, and remove the residual effects of pesticides, fertilizers, and heavy metals from earlier use.
    Statement 2 is incorrect:
    Kandhagari traditional cultivation system:
    ● Wayanad is a place in Kerala which is characterized by its traditional shifting cultivation system called “Kandhagari”, which involves the rotation of crops like rice, millets and tubers.
    ● This system has helped maintain biodiversity and is deeply rooted in the culture of the indigenous communities.
    Statement 3 is incorrect:
    Kuttanad below sea level farming system:
    ● The only system in India that has been practicing rice cultivation below sea level since the past 2 centuries.
    ● Kuttanad is a delta region of about 900 sq. km situated on the west coast of Kerala State, India. The area is a larger mosaic of fragmented landscape patches and varied ecosystems such as coastal backwaters, rivers, vast stretches of paddy fields, marshes, ponds, garden lands, edges, corridors and remarkably networked waterways.
    ● The Kuttanad Below Sea-level Farming System (KBSFS) is unique, as it is the only system in India that practices rice cultivation below sea level. The major land use structure of KBSFS is flat stretches of rice fields in about 50,000 ha of mostly reclaimed delta swamps. The rice fields, which are popularly known as “Puncha Vayals’ ‘ exist in three landscape elements: Karapadam (upland rice fields), Kayal (wetland rice fields) and Kari (land buried with black coal-like materials).

  4. Question 4 of 5
    4. Question

    4. Consider the following pairs:
    Indian tribes in news – States
    1. Hakki Pikki tribes – Karnataka
    2. Hattee tribes – Himachal Pradesh
    3. Soligas tribes – Meghalaya
    How many of the above pairs are correctly matched?

    Correct

    Answer: B
    Explanation:
    Pair 1 is matched correctly:
    ● The Hakki Pikkis are traditionally a semi-nomadic tribe of bird catchers and hunters, who settled down in several parts of Karnataka.
    ● In the last 20 years the community left bird catching and hunting, and have started to prepare traditional medicines, especially hair oils.
    ● Their settlement is mainly on the edge of Bannerghatta National Park.
    Pair 2 is matched correctly:
    ● Recently, the Central Government granted Tribal Status to the Hatti Community of the Tans-Giri region of Himachal Pradesh’s Sirmaur district.
    Who are the Hattis?
    ● The Hattis are a close-knit community that got their name from their tradition of selling homegrown vegetables, crops, meat, and wool, etc. at small markets called ‘haat’ in towns.
    ● Hatti men traditionally don a distinctive white headgear on ceremonial occasions.
    Pair 3 is matched incorrectly:
    ● Indigenous groups Soligas have been living in the Cauvery Basin and the surrounding hills of peninsular India for thousands of years.
    ● Soligas, one of the oldest indigenous communities in the country, are the original inhabitants of Karnataka and live mostly in the Chamarajanagar and Mandya districts.
    ● Honey is an important part of the diet for the Soliga people, who still forage large parts of their food from the biodiversity-rich Ghats.
    ● They are the first tribal community living inside the core area of a tiger reserve in India to get their forest rights officially recognised by a court of law.
    ● Also the scientific community has named a new genus (Soliga ecarinata) of wasp after this community.

    Incorrect

    Answer: B
    Explanation:
    Pair 1 is matched correctly:
    ● The Hakki Pikkis are traditionally a semi-nomadic tribe of bird catchers and hunters, who settled down in several parts of Karnataka.
    ● In the last 20 years the community left bird catching and hunting, and have started to prepare traditional medicines, especially hair oils.
    ● Their settlement is mainly on the edge of Bannerghatta National Park.
    Pair 2 is matched correctly:
    ● Recently, the Central Government granted Tribal Status to the Hatti Community of the Tans-Giri region of Himachal Pradesh’s Sirmaur district.
    Who are the Hattis?
    ● The Hattis are a close-knit community that got their name from their tradition of selling homegrown vegetables, crops, meat, and wool, etc. at small markets called ‘haat’ in towns.
    ● Hatti men traditionally don a distinctive white headgear on ceremonial occasions.
    Pair 3 is matched incorrectly:
    ● Indigenous groups Soligas have been living in the Cauvery Basin and the surrounding hills of peninsular India for thousands of years.
    ● Soligas, one of the oldest indigenous communities in the country, are the original inhabitants of Karnataka and live mostly in the Chamarajanagar and Mandya districts.
    ● Honey is an important part of the diet for the Soliga people, who still forage large parts of their food from the biodiversity-rich Ghats.
    ● They are the first tribal community living inside the core area of a tiger reserve in India to get their forest rights officially recognised by a court of law.
    ● Also the scientific community has named a new genus (Soliga ecarinata) of wasp after this community.

  5. Question 5 of 5
    5. Question

    5. Consider the following rocks:
    1. Gabbro
    2. Shale
    3. Phyllite
    4. Marble
    5. Granite
    6. Diorite
    How many of the above rocks are ‘Igneous rocks’?

    Correct

    Answer: A
    Explanation: Gabbro, Granite and Diorite are the igneous rocks here.
    Igneous rocks:
    ● Igneous rocks are classified based on texture. Texture depends upon size and arrangement of grains or other physical conditions of the materials. If molten material is cooled slowly at great depths, mineral grains may be very large.
    ● Sudden cooling (at the surface) results in small and smooth grains.
    ● Intermediate conditions of cooling would result in intermediate sizes of grains making up igneous rocks.
    ● Granite, gabbro, pegmatite, basalt, volcanic breccia and tuff are some of the examples of igneous rocks.
    Additional Information
    Other types of rocks are:
    Sedimentary Rocks –
    ● The word ‘sedimentary’ is derived from the Latin word ‘sedimentum’, which means settling.
    ● Rocks (igneous, sedimentary and metamorphic) of the earth’s surface are exposed to denudational agents, and are broken up into various sizes of fragments. Such fragments are transported by different exogenous agencies and deposited. These deposits through compaction turn into rocks. This process is called lithification. In many sedimentary rocks, the layers of deposits retain their characteristics even after lithification.
    ● Depending upon the mode of formation, sedimentary rocks are classified into three major groups:
    ✔ Mechanically formed — sandstone, conglomerate, limestone, shale, loess etc.
    ✔ Organically formed — geyserite, chalk, limestone, coal etc.
    ✔ Chemically formed — chert, limestone, halite, potash etc.
    Metamorphic Rocks:
    ● The word metamorphic means ‘change of form’. These rocks form under the action of pressure, volume and temperature (PVT) changes.
    ● Metamorphism occurs when rocks are forced down to lower levels by tectonic processes or when molten magma rising through the crust comes in contact with the crustal rocks or the underlying rocks are subjected to great amounts of pressure by overlying rocks. Metamorphism is a process by which already consolidated rocks undergo recrystallisation and reorganisation of materials within original rocks.
    ● Types of metamorphic rocks depend upon original rocks that were subjected to metamorphism.
    ● Metamorphic rocks are classified into two major groups — foliated rocks and non-foliated rocks.
    ✔ Gneissoid, syenite, slate, schist, marble, quartzite etc. are some examples of metamorphic rocks.

    Incorrect

    Answer: A
    Explanation: Gabbro, Granite and Diorite are the igneous rocks here.
    Igneous rocks:
    ● Igneous rocks are classified based on texture. Texture depends upon size and arrangement of grains or other physical conditions of the materials. If molten material is cooled slowly at great depths, mineral grains may be very large.
    ● Sudden cooling (at the surface) results in small and smooth grains.
    ● Intermediate conditions of cooling would result in intermediate sizes of grains making up igneous rocks.
    ● Granite, gabbro, pegmatite, basalt, volcanic breccia and tuff are some of the examples of igneous rocks.
    Additional Information
    Other types of rocks are:
    Sedimentary Rocks –
    ● The word ‘sedimentary’ is derived from the Latin word ‘sedimentum’, which means settling.
    ● Rocks (igneous, sedimentary and metamorphic) of the earth’s surface are exposed to denudational agents, and are broken up into various sizes of fragments. Such fragments are transported by different exogenous agencies and deposited. These deposits through compaction turn into rocks. This process is called lithification. In many sedimentary rocks, the layers of deposits retain their characteristics even after lithification.
    ● Depending upon the mode of formation, sedimentary rocks are classified into three major groups:
    ✔ Mechanically formed — sandstone, conglomerate, limestone, shale, loess etc.
    ✔ Organically formed — geyserite, chalk, limestone, coal etc.
    ✔ Chemically formed — chert, limestone, halite, potash etc.
    Metamorphic Rocks:
    ● The word metamorphic means ‘change of form’. These rocks form under the action of pressure, volume and temperature (PVT) changes.
    ● Metamorphism occurs when rocks are forced down to lower levels by tectonic processes or when molten magma rising through the crust comes in contact with the crustal rocks or the underlying rocks are subjected to great amounts of pressure by overlying rocks. Metamorphism is a process by which already consolidated rocks undergo recrystallisation and reorganisation of materials within original rocks.
    ● Types of metamorphic rocks depend upon original rocks that were subjected to metamorphism.
    ● Metamorphic rocks are classified into two major groups — foliated rocks and non-foliated rocks.
    ✔ Gneissoid, syenite, slate, schist, marble, quartzite etc. are some examples of metamorphic rocks.

window.wpAdvQuizInitList = window.wpAdvQuizInitList || []; window.wpAdvQuizInitList.push({ id: '#wpAdvQuiz_653', init: { quizId: 653, mode: 0, globalPoints: 10, timelimit: 0, resultsGrade: [0], bo: 0, qpp: 0, catPoints: [10], formPos: 0, lbn: "Finish quiz", json: {"3049":{"type":"single","id":3049,"catId":0,"points":2,"correct":[1,0,0,0]},"3050":{"type":"single","id":3050,"catId":0,"points":2,"correct":[0,0,1,0]},"3051":{"type":"single","id":3051,"catId":0,"points":2,"correct":[0,0,0,1]},"3052":{"type":"single","id":3052,"catId":0,"points":2,"correct":[0,1,0,0]},"3053":{"type":"single","id":3053,"catId":0,"points":2,"correct":[1,0,0,0]}} } });




Day-611 | Daily MCQs | UPSC Prelims | POLITY

Day-611

Time limit: 0

Quiz-summary

0 of 5 questions completed

Questions:

  1. 1
  2. 2
  3. 3
  4. 4
  5. 5

Information

DAILY MCQ

You have already completed the quiz before. Hence you can not start it again.

Quiz is loading...

You must sign in or sign up to start the quiz.

You have to finish following quiz, to start this quiz:

Results

0 of 5 questions answered correctly

Your time:

Time has elapsed

You have reached 0 of 0 points, (0)

Categories

  1. Not categorized 0%
  1. 1
  2. 2
  3. 3
  4. 4
  5. 5
  1. Answered
  2. Review
  1. Question 1 of 5
    1. Question

    1. Consider the following:
    1. Article 367
    2. The General Clauses Act, 1897
    3. The Constituent Assembly Debates
    4. Judicial Precedents
    How many of the above are used by the judiciary in India for the interpretation of the Constitution?

    Correct

    Answer: D
    Explanation:
    The Constitution of India is interpreted by the judiciary by using all the above tools and instruments. Art 367 explains the meanings of words used in the constitution. It also says that the General Clauses Act, 1897 will also be used for interpretation. The General Clauses Act of 1897 aims to consolidate various provisions for the interpretation of words and legal principles into a single statute. These provisions would otherwise have to be specified separately in many different Acts and regulations. The Act also includes definitions of some words and some general principles of interpretation. It aims to provide general definitions that apply to all Central Acts and Regulations. Another method to know the meaning of words used in the Constitution is to look into the Constituent Assembly debates. Constituent Assembly debates are frequently cited before courts to canvas a particular interpretation of the Constitution. The Supreme Court has referred to the Constituent Assembly debates in important constitutional cases such as A K Gopalan vs. State of Madras, Golak Nath vs. State of Punjab and Kesavananda Bharati vs. State of Kerala.
    The courts also use the judicial precedents to understand the meaning of words and phrases employed in the Constitution. The most commonly cited source of constitutional meaning is the Supreme Court’s prior decisions on questions of constitutional law. For most, if not all Justices, judicial precedent provides possible principles, rules, or standards to govern judicial decisions in future cases with arguably similar facts.

    Incorrect

    Answer: D
    Explanation:
    The Constitution of India is interpreted by the judiciary by using all the above tools and instruments. Art 367 explains the meanings of words used in the constitution. It also says that the General Clauses Act, 1897 will also be used for interpretation. The General Clauses Act of 1897 aims to consolidate various provisions for the interpretation of words and legal principles into a single statute. These provisions would otherwise have to be specified separately in many different Acts and regulations. The Act also includes definitions of some words and some general principles of interpretation. It aims to provide general definitions that apply to all Central Acts and Regulations. Another method to know the meaning of words used in the Constitution is to look into the Constituent Assembly debates. Constituent Assembly debates are frequently cited before courts to canvas a particular interpretation of the Constitution. The Supreme Court has referred to the Constituent Assembly debates in important constitutional cases such as A K Gopalan vs. State of Madras, Golak Nath vs. State of Punjab and Kesavananda Bharati vs. State of Kerala.
    The courts also use the judicial precedents to understand the meaning of words and phrases employed in the Constitution. The most commonly cited source of constitutional meaning is the Supreme Court’s prior decisions on questions of constitutional law. For most, if not all Justices, judicial precedent provides possible principles, rules, or standards to govern judicial decisions in future cases with arguably similar facts.

  2. Question 2 of 5
    2. Question

    2. Consider the following statements about the Goods and Services Tax (GST) Council:
    1. It is a constitutional body established by the Parliament as per the provisions of Article 279A of the Constitution.
    2. The Central Goods and Services Act, 2017 provides for a mechanism to address the disputes between the Union and the states arising out of the GST council recommendations.
    Which of the above given statements is/are correct?

    Correct

    Answer: D
    Explanation:
    Statement 1 is incorrect: The GST Council is a constitutional body responsible for making recommendations on issues related to the implementation of the Goods and Services Tax in India. According to Article 279A, it is the President by order who will constitute the GST Council within 60 days from the commencement of 101st amendment. The GST Council was constituted on 15.09.2016 under Article 279A of the Constitution by the President. The GST Council consists of the Union Finance Minister, the Union Minister of State in charge of Revenue or Finance and the Minister in charge of Finance or Taxation or any other Minister nominated by each State Government.
    Statement 2 is incorrect: The dispute resolution mechanism provided under the CGST Act is of an Appellate Tribunal called GSTAT which decides appeals from the decisions of the GST implementation authorities. This is different from the mechanism provided in Art 279A.
    As per Art 279A (11),
    The Goods and Services Tax Council shall establish a mechanism to
    adjudicate any dispute—
    (a) between the Government of India and one or more States; or
    (b) between the Government of India and any State or States on one side and one or more other States on the other side; or
    (c) between two or more States,
    arising out of the recommendations of the Council or implementation thereof.
    So, both the statements are incorrect.
    Hence, option D is the correct answer.

    Incorrect

    Answer: D
    Explanation:
    Statement 1 is incorrect: The GST Council is a constitutional body responsible for making recommendations on issues related to the implementation of the Goods and Services Tax in India. According to Article 279A, it is the President by order who will constitute the GST Council within 60 days from the commencement of 101st amendment. The GST Council was constituted on 15.09.2016 under Article 279A of the Constitution by the President. The GST Council consists of the Union Finance Minister, the Union Minister of State in charge of Revenue or Finance and the Minister in charge of Finance or Taxation or any other Minister nominated by each State Government.
    Statement 2 is incorrect: The dispute resolution mechanism provided under the CGST Act is of an Appellate Tribunal called GSTAT which decides appeals from the decisions of the GST implementation authorities. This is different from the mechanism provided in Art 279A.
    As per Art 279A (11),
    The Goods and Services Tax Council shall establish a mechanism to
    adjudicate any dispute—
    (a) between the Government of India and one or more States; or
    (b) between the Government of India and any State or States on one side and one or more other States on the other side; or
    (c) between two or more States,
    arising out of the recommendations of the Council or implementation thereof.
    So, both the statements are incorrect.
    Hence, option D is the correct answer.

  3. Question 3 of 5
    3. Question

    3. In the context of Indian polity, which of the following is/are mandatory before carving out a union territory from a state?

    Correct

    Answer: A
    Explanation:
    Article 3 of the Constitution of India deals with the formation of new States and alteration of areas, boundaries or names of existing States. It provides that no Bill for such purpose shall be introduced in either House of Parliament except on the recommendation of the President. It also provides for the reference of the Bill by the President to the Legislature of the concerned state for expressing its views within such period. It means, along with Presidential recommendation, the reference to the concerned assembly, ideally should be mandatory. But in the Article 370 judgment, the SC held that this reference is not mandatory while upholding the carving out of the UT of Ladakh from the then state of J and K. Hence, as the law stands today, reorganization of states inter se, does not require mandatory reference to the concerned state assembly.
    Hence, option A is the correct answer.

    Incorrect

    Answer: A
    Explanation:
    Article 3 of the Constitution of India deals with the formation of new States and alteration of areas, boundaries or names of existing States. It provides that no Bill for such purpose shall be introduced in either House of Parliament except on the recommendation of the President. It also provides for the reference of the Bill by the President to the Legislature of the concerned state for expressing its views within such period. It means, along with Presidential recommendation, the reference to the concerned assembly, ideally should be mandatory. But in the Article 370 judgment, the SC held that this reference is not mandatory while upholding the carving out of the UT of Ladakh from the then state of J and K. Hence, as the law stands today, reorganization of states inter se, does not require mandatory reference to the concerned state assembly.
    Hence, option A is the correct answer.

  4. Question 4 of 5
    4. Question

    4. With which of the following Union Ministries, the Department of Public Enterprises (DPE) is associated?

    Correct

    Answer: B
    Explanation:
    The Department of Public Enterprises (DPE) is located in the Ministry of Finance. On July 7, 2021, the DPE was moved from the Ministry of Heavy Industries to the Ministry of Finance. The shift of DPE to the Finance Ministry will help in efficient monitoring of the capital expenditure, asset monetisation and financial health of the Central Public Sector Enterprises. The DPE plays an important role in developing policies and guidelines for the functioning and performance of PSUs in India. It oversees the functions of the GoI’s Public Sector Undertakings and advises on their efficient management and control in the functional areas, such as finance, production, organisation and marketing.
    The DPE’s functions include:
    Monitoring the performance of state-run firms
    Monetizing their assets better through divestment and strategic sales
    Directing their procurements policies
    Coordinating matters of general policy affecting all Public Sector Enterprises
    Hence, the answer is B

    Incorrect

    Answer: B
    Explanation:
    The Department of Public Enterprises (DPE) is located in the Ministry of Finance. On July 7, 2021, the DPE was moved from the Ministry of Heavy Industries to the Ministry of Finance. The shift of DPE to the Finance Ministry will help in efficient monitoring of the capital expenditure, asset monetisation and financial health of the Central Public Sector Enterprises. The DPE plays an important role in developing policies and guidelines for the functioning and performance of PSUs in India. It oversees the functions of the GoI’s Public Sector Undertakings and advises on their efficient management and control in the functional areas, such as finance, production, organisation and marketing.
    The DPE’s functions include:
    Monitoring the performance of state-run firms
    Monetizing their assets better through divestment and strategic sales
    Directing their procurements policies
    Coordinating matters of general policy affecting all Public Sector Enterprises
    Hence, the answer is B

  5. Question 5 of 5
    5. Question

    5. Consider the following statements:
    Statement-I: The fundamental duties added into the Constitution by the 42nd Amendment have not been made self-executory.
    Statement-II: The Constitution makes it obligatory for the state to make laws for the enforceability of the fundamental duties.
    Which one of the following is correct in respect of the above statements?

    Correct

    Answer: C
    Explanation:
    The fundamental duties were incorporated in Part IV-A of the Constitution by the Constitution (42nd Amendment) Act, 1976, during emergency. Article 51(A) describes 11 fundamental duties — 10 came with the 42nd Amendment; the 11th was added by the 86th Amendment in 2002. They were made a part of the Constitution to emphasise the obligation of the citizen in return for the fundamental rights that he or she enjoys. The Russian Constitution has the concept of fundamental duties. The 42nd Constitutional Amendment was made in light of the recommendations of the Swaran Singh Committee.
    The fundamental duties are not made self-executory unlike most of the fundamental rights. The enforceability of fundamental duties has posed a persistent legal dilemma. The constitution itself does not prescribe a legal sanction against the breach or the non-performance of a fundamental duty. It doesn’t even ask the state to make laws in conformity with the fundamental duties. Under the Constitution, neither is there a provision that enforces fundamental duties nor is there any specific prohibition to their enforcement. Nevertheless, the legislature has incorporated some fundamental duties in certain statutes. These statutes lay down frameworks for enforcement of duties and set sanctions in the event of a failure to adhere with the prescription of the law.

    Incorrect

    Answer: C
    Explanation:
    The fundamental duties were incorporated in Part IV-A of the Constitution by the Constitution (42nd Amendment) Act, 1976, during emergency. Article 51(A) describes 11 fundamental duties — 10 came with the 42nd Amendment; the 11th was added by the 86th Amendment in 2002. They were made a part of the Constitution to emphasise the obligation of the citizen in return for the fundamental rights that he or she enjoys. The Russian Constitution has the concept of fundamental duties. The 42nd Constitutional Amendment was made in light of the recommendations of the Swaran Singh Committee.
    The fundamental duties are not made self-executory unlike most of the fundamental rights. The enforceability of fundamental duties has posed a persistent legal dilemma. The constitution itself does not prescribe a legal sanction against the breach or the non-performance of a fundamental duty. It doesn’t even ask the state to make laws in conformity with the fundamental duties. Under the Constitution, neither is there a provision that enforces fundamental duties nor is there any specific prohibition to their enforcement. Nevertheless, the legislature has incorporated some fundamental duties in certain statutes. These statutes lay down frameworks for enforcement of duties and set sanctions in the event of a failure to adhere with the prescription of the law.

window.wpAdvQuizInitList = window.wpAdvQuizInitList || []; window.wpAdvQuizInitList.push({ id: '#wpAdvQuiz_652', init: { quizId: 652, mode: 0, globalPoints: 10, timelimit: 0, resultsGrade: [0], bo: 0, qpp: 0, catPoints: [10], formPos: 0, lbn: "Finish quiz", json: {"3044":{"type":"single","id":3044,"catId":0,"points":2,"correct":[0,0,0,1]},"3045":{"type":"single","id":3045,"catId":0,"points":2,"correct":[0,0,0,1]},"3046":{"type":"single","id":3046,"catId":0,"points":2,"correct":[1,0,0,0]},"3047":{"type":"single","id":3047,"catId":0,"points":2,"correct":[0,1,0,0]},"3048":{"type":"single","id":3048,"catId":0,"points":2,"correct":[0,0,1,0]}} } });




Day-610 | Daily MCQs | UPSC Prelims | HISTORY

Day-610

Time limit: 0

Quiz-summary

0 of 5 questions completed

Questions:

  1. 1
  2. 2
  3. 3
  4. 4
  5. 5

Information

DAILY MCQ

You have already completed the quiz before. Hence you can not start it again.

Quiz is loading...

You must sign in or sign up to start the quiz.

You have to finish following quiz, to start this quiz:

Results

0 of 5 questions answered correctly

Your time:

Time has elapsed

You have reached 0 of 0 points, (0)

Categories

  1. Not categorized 0%
  1. 1
  2. 2
  3. 3
  4. 4
  5. 5
  1. Answered
  2. Review
  1. Question 1 of 5
    1. Question

    1. Consider the following statements regarding Subika painting:
    1. It is linked to the Meitei community’s cultural history through its six surviving manuscripts.
    2. It is mostly created on glass and board instead of cloth and vellum.
    Which of the statements given above is/are correct?

    Correct

    Answer: A
    Explanation:
    Statement 1 is correct: The Subika painting style is intricately linked to the Meitei community’s cultural history through its six surviving manuscripts: Subika, Subika Achouba, Subika Laishaba, Subika Choudit, Subika Cheithil and Thengrakhel Subika.
    Statement 2 is incorrect: It is done on handmade paper, and the materials for manuscripts, such as handmade paper or tree bark, are prepared locally.

    Incorrect

    Answer: A
    Explanation:
    Statement 1 is correct: The Subika painting style is intricately linked to the Meitei community’s cultural history through its six surviving manuscripts: Subika, Subika Achouba, Subika Laishaba, Subika Choudit, Subika Cheithil and Thengrakhel Subika.
    Statement 2 is incorrect: It is done on handmade paper, and the materials for manuscripts, such as handmade paper or tree bark, are prepared locally.

  2. Question 2 of 5
    2. Question

    2. With reference to the Government of India Act, 1919, consider the following statements:
    1. The Act introduced a bicameral legislature, the lower House or Central Legislative Assembly and the Upper House or Council of State.
    2. The Act introduced dyarchy for the executive at the level of the provincial government.
    3. The Act introduced the concept of separate electorate for the first time.
    How many of the statements given above are correct?

    Correct

    Answer: B
    Explanation:
    Statement 1 is correct: The Act introduced bicameral legislature i.e the Lower House or Central Legislative Assembly and the Upper House or Council of State.
    Statement 2 is correct: The Act introduced dyarchy (rule of two individuals/parties) for the executive at the level of the provincial government. The diarchy was implemented in eight provinces:
    Assam, Bengal, Bihar and Orissa, Central Provinces, United Provinces, Bombay, Madras and Punjab.
    The provincial governments were given more powers under the system of Dyarchy.
    The governor was to be the executive head in the province.
    Statement 3 is incorrect: The Indian Council Act 1909 introduced the concept of separate electorate. Under separate electorates, the Muslim members were to be elected only by Muslim voters. Some constituencies were earmarked for Muslims and only Muslims could vote their representatives.
    Additional information:
    In 1918, Edwin Montagu, the Secretary of State, and Lord Chelmsford, the Viceroy, produced their scheme of constitutional reforms, known as the Montagu-Chelmsford (or Mont-Ford) Reforms, which led to the enactment of the Government of India Act of 1919.
    Montagu-Chelmsford Reforms which came into force in 1921.
    The sole purpose of this Act was to ensure Indians of their representation in the Government.
    The Act introduced reforms at the Central as well as Provincial levels of Government.
    They introduced significant acts, commissions, and policies that shaped the modern view of education, still persisting in India.

    Incorrect

    Answer: B
    Explanation:
    Statement 1 is correct: The Act introduced bicameral legislature i.e the Lower House or Central Legislative Assembly and the Upper House or Council of State.
    Statement 2 is correct: The Act introduced dyarchy (rule of two individuals/parties) for the executive at the level of the provincial government. The diarchy was implemented in eight provinces:
    Assam, Bengal, Bihar and Orissa, Central Provinces, United Provinces, Bombay, Madras and Punjab.
    The provincial governments were given more powers under the system of Dyarchy.
    The governor was to be the executive head in the province.
    Statement 3 is incorrect: The Indian Council Act 1909 introduced the concept of separate electorate. Under separate electorates, the Muslim members were to be elected only by Muslim voters. Some constituencies were earmarked for Muslims and only Muslims could vote their representatives.
    Additional information:
    In 1918, Edwin Montagu, the Secretary of State, and Lord Chelmsford, the Viceroy, produced their scheme of constitutional reforms, known as the Montagu-Chelmsford (or Mont-Ford) Reforms, which led to the enactment of the Government of India Act of 1919.
    Montagu-Chelmsford Reforms which came into force in 1921.
    The sole purpose of this Act was to ensure Indians of their representation in the Government.
    The Act introduced reforms at the Central as well as Provincial levels of Government.
    They introduced significant acts, commissions, and policies that shaped the modern view of education, still persisting in India.

  3. Question 3 of 5
    3. Question

    3. Consider the following statements:
    Statement I: The Gupta empire controlled rich seaports and overseas trade.
    Statement II: The Gupta empire controlled the entire Ganga valley.
    Which one of the following is correct in respect of the above statements?

    Correct

    Answer: A
    Explanation:
    Statement 1 is correct: The Gupta empire controlled rich sea ports and overseas trade.
    Statement 2 is correct: The Gupta empire was able to bring the entire Ganga valley under their control.
    The Ganga valley with its populations and resources was the basis for expansion of their control over Gujrat and they were able to control sea ports and overseas trade.
    Additional information:
    Around the last decade of the 3rd century CE (about 275 CE), the dynasty of the Guptas came to power.
    The Gupta empire established its control over a good part of the former dominions of both the Kushanas and the Satavahanas. The Guptas kept northern India politically united for more than a century (335 CE- 455 CE).
    The Guptas are believed to have been feudatories of the Kushanas.
    The original kingdom of the Guptas comprised Uttar Pradesh and Bihar with their centre of power at Prayag (U.P).
    The Guptas set up their rule over the fertile plains of the Madhyadesha, also known as Anuganga (the middle Gangetic basin), Saketa (U.P Ayodhya), Prayag (U.P) and Magadha (mostly Bihar).
    The Guptas made good use of the iron ore reserves in central India and south Bihar and also took advantage of their proximity to the areas in north India which carried on silk trade with the Byzantine empire (eastern Roman empire).
    The Gupta period in ancient India is referred to as the “Golden Age” because of the numerous achievements in the field of arts, literature, science and technology. It also brought about the political unification of the subcontinent.

    Incorrect

    Answer: A
    Explanation:
    Statement 1 is correct: The Gupta empire controlled rich sea ports and overseas trade.
    Statement 2 is correct: The Gupta empire was able to bring the entire Ganga valley under their control.
    The Ganga valley with its populations and resources was the basis for expansion of their control over Gujrat and they were able to control sea ports and overseas trade.
    Additional information:
    Around the last decade of the 3rd century CE (about 275 CE), the dynasty of the Guptas came to power.
    The Gupta empire established its control over a good part of the former dominions of both the Kushanas and the Satavahanas. The Guptas kept northern India politically united for more than a century (335 CE- 455 CE).
    The Guptas are believed to have been feudatories of the Kushanas.
    The original kingdom of the Guptas comprised Uttar Pradesh and Bihar with their centre of power at Prayag (U.P).
    The Guptas set up their rule over the fertile plains of the Madhyadesha, also known as Anuganga (the middle Gangetic basin), Saketa (U.P Ayodhya), Prayag (U.P) and Magadha (mostly Bihar).
    The Guptas made good use of the iron ore reserves in central India and south Bihar and also took advantage of their proximity to the areas in north India which carried on silk trade with the Byzantine empire (eastern Roman empire).
    The Gupta period in ancient India is referred to as the “Golden Age” because of the numerous achievements in the field of arts, literature, science and technology. It also brought about the political unification of the subcontinent.

  4. Question 4 of 5
    4. Question

    4. Consider the following statements:
    1. A Sanskrit College was started in Varanasi by Jonathan Duncan for the study of Hindu philosophy and laws in 1791.
    2. Lord Wellesley established the first educational institution, Calcutta Madrasa for Islamic Law Studies in 1781.
    3. William Jones founded the Asiatic Society of Bengal to understand and study the culture and history of India in 1784.
    How many of the statements given above are correct?

    Correct

    Answer: B
    Explanation:
    Statement 1 is correct: A Sanskrit College was started in Varanasi by Jonathan Duncan for the study of Hindu philosophy and laws In 1791.
    Statement 2 is incorrect: Warren Hastings established the first educational institution, Calcutta Madrasa for Islamic Law Studies in 1781.
    Statement 3 is correct: William Jones founded the Asiatic Society of Bengal to understand and study the culture and history of India in 1784.
    Additional information:
    The British Education System in India introduced the concept of practical learning and modern education in India. Initially, the British established educational institutions for learning about the local customs, traditions, and laws to understand the country better.
    In education under British rule, there were three agents of modern education, i.e., Indian intellectuals and reformers, Christian Missionaries, and the East India Company.
    They introduced significant acts, commissions, and policies that shaped the modern view of education, still persisting in India.
    However, after they started ruling India, they abolished the ancient gurukul system. They established certain educational institutions to educate a small section of Indians and introduced them to English.

    Incorrect

    Answer: B
    Explanation:
    Statement 1 is correct: A Sanskrit College was started in Varanasi by Jonathan Duncan for the study of Hindu philosophy and laws In 1791.
    Statement 2 is incorrect: Warren Hastings established the first educational institution, Calcutta Madrasa for Islamic Law Studies in 1781.
    Statement 3 is correct: William Jones founded the Asiatic Society of Bengal to understand and study the culture and history of India in 1784.
    Additional information:
    The British Education System in India introduced the concept of practical learning and modern education in India. Initially, the British established educational institutions for learning about the local customs, traditions, and laws to understand the country better.
    In education under British rule, there were three agents of modern education, i.e., Indian intellectuals and reformers, Christian Missionaries, and the East India Company.
    They introduced significant acts, commissions, and policies that shaped the modern view of education, still persisting in India.
    However, after they started ruling India, they abolished the ancient gurukul system. They established certain educational institutions to educate a small section of Indians and introduced them to English.

  5. Question 5 of 5
    5. Question

    5. Consider the following statements regarding the Satnami revolt:
    1. The Satnami revolt was a major peasant uprising that took place during the reign of Mughal Emperor Shah Jahan.
    2. The Satnamis were subjected to oppressive taxes and discriminatory policies which led to widespread discontent among the sect.
    3. The rebellion was successful in overcoming the Mughal rule, and found their own regional kingdom.
    How many of the statements given above are correct?

    Correct

    Answer: A
    Explanation:
    Statement 1 is incorrect: The Satnami revolt was a major peasant uprising that took place in 1672 during the reign of Mughal Emperor Aurangzeb.
    Statement 2 is correct: The Satnamis were subjected to oppressive taxes and discriminatory policies under Aurangzeb’s rule, which led to widespread discontent among the sects.
    Statement 3 is incorrect: The rebellion was crushed when Aurangzeb himself took personal command and sent 10,000 troops with artillery to crush the Satnamis.
    Additional information:
    The Satnami sect was founded by a saint named “Birbhan” in 1657 in Narnaul in Haryana. The major religious activity of this sect is to chant and meditate on the true names {Sat-Nam} of God specially Rama and Krishna.
    This sect is thought to be an offshoot of Ravidasi sect and comprised people from different castes and communities of Hindu society. The followers of this sect kept their heads shaved {thus called Mundiyas}. Today, there are at least 1.5 million followers of the Satnami sect spread in Rajasthan, UP, MP, Bihar, Gujarat and Maharashtra.
    During the rule of Puritan Mughal Aurangzeb, there was resentment among Hindus for revival of Jaziya and general destruction of temples under imperial orders. The revolt triggered when a Mughal soldier killed a Satnami. The Satnamis killed the soldier in revenge and in turn Mughal soldiers were sent to teach them a lesson. Some 5,000 Satnamis stood up in arms and routed the Mughal troops in the town, drove away the Mughal administrators and set up their own administration under their leader Birbhan.

    Incorrect

    Answer: A
    Explanation:
    Statement 1 is incorrect: The Satnami revolt was a major peasant uprising that took place in 1672 during the reign of Mughal Emperor Aurangzeb.
    Statement 2 is correct: The Satnamis were subjected to oppressive taxes and discriminatory policies under Aurangzeb’s rule, which led to widespread discontent among the sects.
    Statement 3 is incorrect: The rebellion was crushed when Aurangzeb himself took personal command and sent 10,000 troops with artillery to crush the Satnamis.
    Additional information:
    The Satnami sect was founded by a saint named “Birbhan” in 1657 in Narnaul in Haryana. The major religious activity of this sect is to chant and meditate on the true names {Sat-Nam} of God specially Rama and Krishna.
    This sect is thought to be an offshoot of Ravidasi sect and comprised people from different castes and communities of Hindu society. The followers of this sect kept their heads shaved {thus called Mundiyas}. Today, there are at least 1.5 million followers of the Satnami sect spread in Rajasthan, UP, MP, Bihar, Gujarat and Maharashtra.
    During the rule of Puritan Mughal Aurangzeb, there was resentment among Hindus for revival of Jaziya and general destruction of temples under imperial orders. The revolt triggered when a Mughal soldier killed a Satnami. The Satnamis killed the soldier in revenge and in turn Mughal soldiers were sent to teach them a lesson. Some 5,000 Satnamis stood up in arms and routed the Mughal troops in the town, drove away the Mughal administrators and set up their own administration under their leader Birbhan.

window.wpAdvQuizInitList = window.wpAdvQuizInitList || []; window.wpAdvQuizInitList.push({ id: '#wpAdvQuiz_651', init: { quizId: 651, mode: 0, globalPoints: 10, timelimit: 0, resultsGrade: [0], bo: 0, qpp: 0, catPoints: [10], formPos: 0, lbn: "Finish quiz", json: {"3039":{"type":"single","id":3039,"catId":0,"points":2,"correct":[1,0,0,0]},"3040":{"type":"single","id":3040,"catId":0,"points":2,"correct":[0,1,0,0]},"3041":{"type":"single","id":3041,"catId":0,"points":2,"correct":[1,0,0,0]},"3042":{"type":"single","id":3042,"catId":0,"points":2,"correct":[0,1,0,0]},"3043":{"type":"single","id":3043,"catId":0,"points":2,"correct":[1,0,0,0]}} } });




Day-609 | Daily MCQs | UPSC Prelims | GENERAL SCIENCE AND TECHNOLOGICAL DEVELOPMENTS

Day-609

Time limit: 0

Quiz-summary

0 of 5 questions completed

Questions:

  1. 1
  2. 2
  3. 3
  4. 4
  5. 5

Information

DAILY MCQ

You have already completed the quiz before. Hence you can not start it again.

Quiz is loading...

You must sign in or sign up to start the quiz.

You have to finish following quiz, to start this quiz:

Results

0 of 5 questions answered correctly

Your time:

Time has elapsed

You have reached 0 of 0 points, (0)

Categories

  1. Not categorized 0%
  1. 1
  2. 2
  3. 3
  4. 4
  5. 5
  1. Answered
  2. Review
  1. Question 1 of 5
    1. Question

    1. Consider the following statements:
    1. Solar wind produces auroras.
    2. Solar flare damages satellite communication.
    3. Coronal mass ejection causes turbulence in earth’s magnetic field.
    How many of the above statements are correct?

    Correct

    Answer: C
    Explanation
    Although solar wind, solar flare and coronal mass ejection are interrelated they all describe the sun’s activity. Only the frequency of occurrence and intensity differs.
    Statement 1 is correct: Solar winds are a stream of plasma particles that are ejected from the Sun’s atmosphere. They are caused by the interaction of plasma with gases present in the earth’s ionosphere. This produces a brilliant display of colours in the night sky in polar regions generally known as Auroras. The solar wind is a continual stream of protons and electrons from the sun’s outermost atmosphere — the corona. These charged particles breeze through the solar system at speeds ranging between 400km per sec and 800km per sec.
    Statement 2 is correct: Solar flares are an intense burst of radiation that is caused by the release of magnetic energy from sunspots. They can be damaging to satellites and electronics on Earth.
    Statement 3 is correct: Coronal Mass Ejections (CME) are sudden, terribly violent ejections of billions of tons of plasma from the Corona or the Sun’s outer atmosphere. They can cause geomagnetic turbulence and damage the Earth’s magnetic field.

    Incorrect

    Answer: C
    Explanation
    Although solar wind, solar flare and coronal mass ejection are interrelated they all describe the sun’s activity. Only the frequency of occurrence and intensity differs.
    Statement 1 is correct: Solar winds are a stream of plasma particles that are ejected from the Sun’s atmosphere. They are caused by the interaction of plasma with gases present in the earth’s ionosphere. This produces a brilliant display of colours in the night sky in polar regions generally known as Auroras. The solar wind is a continual stream of protons and electrons from the sun’s outermost atmosphere — the corona. These charged particles breeze through the solar system at speeds ranging between 400km per sec and 800km per sec.
    Statement 2 is correct: Solar flares are an intense burst of radiation that is caused by the release of magnetic energy from sunspots. They can be damaging to satellites and electronics on Earth.
    Statement 3 is correct: Coronal Mass Ejections (CME) are sudden, terribly violent ejections of billions of tons of plasma from the Corona or the Sun’s outer atmosphere. They can cause geomagnetic turbulence and damage the Earth’s magnetic field.

  2. Question 2 of 5
    2. Question

    2. Consider the following statements:
    Statement I: The energy released by the nuclear fusion process is safe as compared to that released by the nuclear fission.
    Statement II: Nuclear fusion requires a very high starting temperature which can occur only under strict operating conditions.
    Which one of the following is correct in respect of the above statements?

    Correct

    Answer: A
    Explanation:
    Context: The International Thermonuclear Experimental Reactor (ITER) is already billions of dollars over budget and decades behind schedule. Not even its leaders can say how much more money and time it will take to complete. It run the risk of turning into one of the largest scientific disaster in the history of mankind. ITER is supposed to help humanity achieve the dream of a world powered not by fossil fuels but by fusion energy, the same process that makes the stars shine.
    Explanation
    Statement 1 is CORRECT: Fusion reactions is intrinsically safe and is not expected to produce high activity or long-lived nuclear waste. Furthermore, as the fusion process is difficult to start and maintain, there is no risk of a runaway reaction and meltdown.
    Statement 2 is CORRECT and is CORRECT explanation of Statement 1: Fusion can only occur under strict operational conditions, outside of which (in the case of an accident or system failure, for example), the plasma will naturally terminate, lose its energy very quickly and extinguish before any sustained damage is done to the reactor. Importantly, nuclear fusion — just like fission — does not emit carbon dioxide or other greenhouse gases into the atmosphere, so it could be a long-term source of low-carbon electricity in future.

    Incorrect

    Answer: A
    Explanation:
    Context: The International Thermonuclear Experimental Reactor (ITER) is already billions of dollars over budget and decades behind schedule. Not even its leaders can say how much more money and time it will take to complete. It run the risk of turning into one of the largest scientific disaster in the history of mankind. ITER is supposed to help humanity achieve the dream of a world powered not by fossil fuels but by fusion energy, the same process that makes the stars shine.
    Explanation
    Statement 1 is CORRECT: Fusion reactions is intrinsically safe and is not expected to produce high activity or long-lived nuclear waste. Furthermore, as the fusion process is difficult to start and maintain, there is no risk of a runaway reaction and meltdown.
    Statement 2 is CORRECT and is CORRECT explanation of Statement 1: Fusion can only occur under strict operational conditions, outside of which (in the case of an accident or system failure, for example), the plasma will naturally terminate, lose its energy very quickly and extinguish before any sustained damage is done to the reactor. Importantly, nuclear fusion — just like fission — does not emit carbon dioxide or other greenhouse gases into the atmosphere, so it could be a long-term source of low-carbon electricity in future.

  3. Question 3 of 5
    3. Question

    3. With reference to the properties of photons, which of the following statements is correct?

    Correct

    Answer: B
    Explanation:
    The Dual Nature of matter and the dual nature of radiation were ground-breaking physics notions. Scientists discovered wave-particle duality, or the dual nature of matter and radiation. Everything is made up of waves and particles.
    What is a Photon?
    A photon is the smallest discrete amount of electromagnetic energy, also known as a quantum. It’s the fundamental unit of all light.
    Photons have the following basic properties:
    The quantity of photons crossing an area per unit time increases as light intensity increases. It has no effect on the radiation’s energy.
    Electric and magnetic fields have no effect on a photon. It has no electrical charge.
    A photon is massless.
    It’s a sturdily constructed particle.
    When radiation is emitted or absorbed, photons can be generated or destroyed.
    During a photon-electron collision, the whole energy and momentum are conserved.
    A photon is incapable of decay on its own.
    A photon’s energy can be transferred when it interacts with other particles.

    Incorrect

    Answer: B
    Explanation:
    The Dual Nature of matter and the dual nature of radiation were ground-breaking physics notions. Scientists discovered wave-particle duality, or the dual nature of matter and radiation. Everything is made up of waves and particles.
    What is a Photon?
    A photon is the smallest discrete amount of electromagnetic energy, also known as a quantum. It’s the fundamental unit of all light.
    Photons have the following basic properties:
    The quantity of photons crossing an area per unit time increases as light intensity increases. It has no effect on the radiation’s energy.
    Electric and magnetic fields have no effect on a photon. It has no electrical charge.
    A photon is massless.
    It’s a sturdily constructed particle.
    When radiation is emitted or absorbed, photons can be generated or destroyed.
    During a photon-electron collision, the whole energy and momentum are conserved.
    A photon is incapable of decay on its own.
    A photon’s energy can be transferred when it interacts with other particles.

  4. Question 4 of 5
    4. Question

    4. Which of the following statements about RADAR and LiDAR technology is incorrect?

    Correct

    Answer: D
    Explanation:
    The basic purpose of LiDAR and RADAR is the same – detecting the presence and volume of distant objects – it is essential to understand the difference between these two technologies.
    Light Detection and Ranging (LiDAR) is a light-based remote sensing technology. By having a LiDAR with a 360° viewing angle it is possible to obtain a point cloud of the environment. Then, a specific software makes a 3D image that reproduces the shape around the LiDAR with a precise position in space.
    The RADAR system works in much the same way as LiDAR, with the big difference that it uses radio waves instead of laser or LED light. It transmits radio waves from a rotating or fixed antenna and measures the time of flight of the reflected signal.
    With its wavelength, the RADAR can detect objects at long distance and through fog or clouds. But its lateral resolution is limited by the size of the antenna. The resolution of standard RADAR is several meters at 100 meters.
    LiDAR is a compact solution that enables a high level of accuracy for 3D mapping. At 100 meters, LiDAR systems have a resolution of a few centimetres.
    This is why LiDAR is used for laser altimetry and contour mapping. Radar, on the other hand, is used for aircraft anti-collision systems, air traffic control or radar astronomy.

    Incorrect

    Answer: D
    Explanation:
    The basic purpose of LiDAR and RADAR is the same – detecting the presence and volume of distant objects – it is essential to understand the difference between these two technologies.
    Light Detection and Ranging (LiDAR) is a light-based remote sensing technology. By having a LiDAR with a 360° viewing angle it is possible to obtain a point cloud of the environment. Then, a specific software makes a 3D image that reproduces the shape around the LiDAR with a precise position in space.
    The RADAR system works in much the same way as LiDAR, with the big difference that it uses radio waves instead of laser or LED light. It transmits radio waves from a rotating or fixed antenna and measures the time of flight of the reflected signal.
    With its wavelength, the RADAR can detect objects at long distance and through fog or clouds. But its lateral resolution is limited by the size of the antenna. The resolution of standard RADAR is several meters at 100 meters.
    LiDAR is a compact solution that enables a high level of accuracy for 3D mapping. At 100 meters, LiDAR systems have a resolution of a few centimetres.
    This is why LiDAR is used for laser altimetry and contour mapping. Radar, on the other hand, is used for aircraft anti-collision systems, air traffic control or radar astronomy.

  5. Question 5 of 5
    5. Question

    5. Consider the following statements regarding Gravity:
    1. Anything that has mass, has gravity.
    2. Objects with less mass have more gravity.
    Which of the above given statements is/ are incorrect?

    Correct

    Answer: B
    Explanation:
    Gravity is the force by which a planet or other body draws objects toward its centre. The force of gravity keeps all the planets in orbit around the sun.
    Anything that has mass also has gravity. Objects with more mass have more gravity. Gravity also gets weaker with distance. So, the closer objects are to each other, the stronger their gravitational pull is.
    Earth’s gravity comes from all its mass. All its mass makes a combined gravitational pull on all the mass in your body. That’s what gives you weight. And if you were on a planet with less mass than Earth, you would weigh less than you do here.
    Gravity is the force by which a planet or other body draws objects toward its centre. The force of gravity keeps all the planets in orbit around the sun.

    Gravity in our universe
    Gravity is what holds the planets in orbit around the sun and what keeps the moon in orbit around Earth. The gravitational pull of the moon pulls the seas towards it, causing the ocean tides. Gravity creates stars and planets by pulling together the material from which they are made. It not only pulls on mass but also on light. Albert Einstein discovered this principle. If you shine a flashlight upwards, the light will grow imperceptibly redder as gravity pulls it. You can’t see the change with your eyes, but scientists can measure it.

    Incorrect

    Answer: B
    Explanation:
    Gravity is the force by which a planet or other body draws objects toward its centre. The force of gravity keeps all the planets in orbit around the sun.
    Anything that has mass also has gravity. Objects with more mass have more gravity. Gravity also gets weaker with distance. So, the closer objects are to each other, the stronger their gravitational pull is.
    Earth’s gravity comes from all its mass. All its mass makes a combined gravitational pull on all the mass in your body. That’s what gives you weight. And if you were on a planet with less mass than Earth, you would weigh less than you do here.
    Gravity is the force by which a planet or other body draws objects toward its centre. The force of gravity keeps all the planets in orbit around the sun.

    Gravity in our universe
    Gravity is what holds the planets in orbit around the sun and what keeps the moon in orbit around Earth. The gravitational pull of the moon pulls the seas towards it, causing the ocean tides. Gravity creates stars and planets by pulling together the material from which they are made. It not only pulls on mass but also on light. Albert Einstein discovered this principle. If you shine a flashlight upwards, the light will grow imperceptibly redder as gravity pulls it. You can’t see the change with your eyes, but scientists can measure it.

window.wpAdvQuizInitList = window.wpAdvQuizInitList || []; window.wpAdvQuizInitList.push({ id: '#wpAdvQuiz_650', init: { quizId: 650, mode: 0, globalPoints: 10, timelimit: 0, resultsGrade: [0], bo: 0, qpp: 0, catPoints: [10], formPos: 0, lbn: "Finish quiz", json: {"3034":{"type":"single","id":3034,"catId":0,"points":2,"correct":[0,0,1,0]},"3035":{"type":"single","id":3035,"catId":0,"points":2,"correct":[1,0,0,0]},"3036":{"type":"single","id":3036,"catId":0,"points":2,"correct":[0,1,0,0]},"3037":{"type":"single","id":3037,"catId":0,"points":2,"correct":[0,0,0,1]},"3038":{"type":"single","id":3038,"catId":0,"points":2,"correct":[0,1,0,0]}} } });




Day-608 | Daily MCQs | UPSC Prelims | ENVIRONMENT

Day-608

Time limit: 0

Quiz-summary

0 of 5 questions completed

Questions:

  1. 1
  2. 2
  3. 3
  4. 4
  5. 5

Information

DAILY MCQ

You have already completed the quiz before. Hence you can not start it again.

Quiz is loading...

You must sign in or sign up to start the quiz.

You have to finish following quiz, to start this quiz:

Results

0 of 5 questions answered correctly

Your time:

Time has elapsed

You have reached 0 of 0 points, (0)

Categories

  1. Not categorized 0%
  1. 1
  2. 2
  3. 3
  4. 4
  5. 5
  1. Answered
  2. Review
  1. Question 1 of 5
    1. Question

    1. Consider the following statements:
    1. Sharks are often targeted by hunters for their fins which are highly priced as delicacies.
    2. Pangolins are poached for their scales that have therapeutic value.
    3. Sangai deer is often poached for its horns which have medicinal uses.
    How many of the above statements are correct?

    Correct

    Answer. B
    Explanation:
    Statement 1 is correct: Sharks and their relatives are some of the most imperilled animals on Earth: More than one-third of all known species are threatened with extinction. Losing marine predators can destabilise entire food webs and the ecosystems that these food webs depend on. Shark finning involves cutting off a shark’s fin, often while the shark is still alive, and dumping the animal back into the sea to die slowly. The fins are used for shark fin soup, an East Asian dish associated with wealth and festivity.
    Statement 2 is correct: Pangolins are among the most trafficked wild mammals globally. They are poached for their scales, used as an ingredient in traditional medicine in China and Southeast Asia.
    The Indian Pangolin has been classified as ‘Endangered’ and the Chinese Pangolin as ‘Critically Endangered’ by the International Union for Conservation of Nature’s Red List of Threatened Species.
    Statement 3 is incorrect: Poaching and hunting of the Sangai deer (endemic to Loktak lake, Manipur) for its meat and antlers (not horns) has been a significant threat to its survival. The antlers of the male Sangai deer are highly valued for their medicinal properties in traditional Chinese medicine, which has created its demand for them in the illegal wildlife trade.

    Incorrect

    Answer. B
    Explanation:
    Statement 1 is correct: Sharks and their relatives are some of the most imperilled animals on Earth: More than one-third of all known species are threatened with extinction. Losing marine predators can destabilise entire food webs and the ecosystems that these food webs depend on. Shark finning involves cutting off a shark’s fin, often while the shark is still alive, and dumping the animal back into the sea to die slowly. The fins are used for shark fin soup, an East Asian dish associated with wealth and festivity.
    Statement 2 is correct: Pangolins are among the most trafficked wild mammals globally. They are poached for their scales, used as an ingredient in traditional medicine in China and Southeast Asia.
    The Indian Pangolin has been classified as ‘Endangered’ and the Chinese Pangolin as ‘Critically Endangered’ by the International Union for Conservation of Nature’s Red List of Threatened Species.
    Statement 3 is incorrect: Poaching and hunting of the Sangai deer (endemic to Loktak lake, Manipur) for its meat and antlers (not horns) has been a significant threat to its survival. The antlers of the male Sangai deer are highly valued for their medicinal properties in traditional Chinese medicine, which has created its demand for them in the illegal wildlife trade.

  2. Question 2 of 5
    2. Question

    2. Once any area around a National Park is notified by the government as an Eco-Sensitive Zone (ESZ), then:
    1. the people living in the villages within the ESZ must evacuate the region and relocate elsewhere.
    2. a blanket ban is imposed on further widening of roads and commercial mining.
    3. the local people are allowed to undertake any construction on their land.
    How many of the above statements are correct?

    Correct

    Answer. A
    Explanation:
    Statements 1 and 2 are incorrect: The Eco-Sensitive Zone (ESZ) Notification does not involve displacement and evacuation of farmers/people living in the villages.
    The activities in the ESZ are generally regulated and not prohibitory in nature barring a few such as: (i) commercial mining, stone quarrying and crushing units;
    (ii) major hydroelectric project;
    (iii) handling of hazardous substances;
    (iv) discharge of untreated effluents;
    (v) setting up of brick kilns;
    (vi) setting up of polluting industries, which have high potential for environmental damage.
    As such, there is no prohibition on ongoing agriculture and horticulture practices by local communities, dairy farming, aquaculture, fisheries, poultry farm, goat farm, food related units etc. Further, the activities like infrastructure augmentation including civic amenities, widening of roads, non-polluting industries etc. are also under regulated category.
    Statement 3 is correct: No new commercial construction of any kind is permitted within 1 Kilometre from the boundary of the Protected Area or up to extent of the Eco-Sensitive Zone whichever is nearer. However, there is no restriction for local people, they may undertake construction in their land for their use.

    Incorrect

    Answer. A
    Explanation:
    Statements 1 and 2 are incorrect: The Eco-Sensitive Zone (ESZ) Notification does not involve displacement and evacuation of farmers/people living in the villages.
    The activities in the ESZ are generally regulated and not prohibitory in nature barring a few such as: (i) commercial mining, stone quarrying and crushing units;
    (ii) major hydroelectric project;
    (iii) handling of hazardous substances;
    (iv) discharge of untreated effluents;
    (v) setting up of brick kilns;
    (vi) setting up of polluting industries, which have high potential for environmental damage.
    As such, there is no prohibition on ongoing agriculture and horticulture practices by local communities, dairy farming, aquaculture, fisheries, poultry farm, goat farm, food related units etc. Further, the activities like infrastructure augmentation including civic amenities, widening of roads, non-polluting industries etc. are also under regulated category.
    Statement 3 is correct: No new commercial construction of any kind is permitted within 1 Kilometre from the boundary of the Protected Area or up to extent of the Eco-Sensitive Zone whichever is nearer. However, there is no restriction for local people, they may undertake construction in their land for their use.

  3. Question 3 of 5
    3. Question

    3. Consider the following statements:
    1. India currently has the fourth highest wind installed capacity in the world.
    2. Gujarat leads in the installed capacity of wind energy in the country.
    Which of the statements given above is/are incorrect?

    Correct

    Answer: B
    Explanation:
    Statement 1 is correct: India’s wind energy sector is led by indigenous wind power industry and has shown consistent progress. The expansion of the wind industry has resulted in a strong ecosystem, project operation capabilities and manufacturing base of around 12,000 MW per annum. The country currently has the fourth highest wind installed capacity in the world with total installed capacity of 41.93 GW (as on 31st December, 2022) of which 1.85 GW was added during January to December, 2022.
    Statement 2 is incorrect: According to the Ministry of New and Renewable Energy, the states leading in wind power installed capacity, in descending order are:
    Tamil Nadu-Gujarat-Karnataka-Maharashtra

    Incorrect

    Answer: B
    Explanation:
    Statement 1 is correct: India’s wind energy sector is led by indigenous wind power industry and has shown consistent progress. The expansion of the wind industry has resulted in a strong ecosystem, project operation capabilities and manufacturing base of around 12,000 MW per annum. The country currently has the fourth highest wind installed capacity in the world with total installed capacity of 41.93 GW (as on 31st December, 2022) of which 1.85 GW was added during January to December, 2022.
    Statement 2 is incorrect: According to the Ministry of New and Renewable Energy, the states leading in wind power installed capacity, in descending order are:
    Tamil Nadu-Gujarat-Karnataka-Maharashtra

  4. Question 4 of 5
    4. Question

    4. With reference to the ‘Cool Coalition’, consider the following statements:
    1. It connects government and non-governmental actors such as finance and academia to facilitate knowledge exchange, advocacy and joint action towards a rapid global transition to efficient and climate-friendly cooling.
    2. It was launched by Clean Cooling Collaborative (K-CEP) in the year 2019.
    Which of the statements given above is/are correct?

    Correct

    Answer: A
    Explanation:
    Statement 1 is correct but statement 2 is incorrect: Launched at the first Global Conference on Synergies between the 2030 Agenda and Paris Agreement at Copenhagen, Denmark in April 2019, the Cool Coalition was assembled by United Nations Environment Programme (UNEP) with the belief that a unified effort of governments, cities, businesses, and civil society that share the objective of advancing, sustainable cooling will be more effective in bringing about change than acting alone.
    It is a global multi-stakeholder network that connects a wide range of key actors from government, cities, international organizations, businesses, finance, academia, and civil society groups to facilitate knowledge exchange, advocacy and joint action towards a rapid global transition to efficient and climate-friendly cooling. The Cool Coalition is now working with over 100 partners, including 23 countries. India is also a member of the Cool Coalition.
    Additional information
    The Cool Coalition and COP28 UAE Presidency launched the Global Cooling Pledge at COP28 of the UNFCCC, with nearly 70 government signatories and more than 50 supporters from international organizations, international finance institutions and industry. The Global Cooling Pledge marks the world’s first collective effort to improve energy efficiency and reduce emissions from the cooling sector.
    The Pledge aims to reduce global cooling-related emissions by 68 per cent by 2050, improve energy efficiency of cooling technologies by 50 per cent by 2030 and to increase access to sustainable cooling for the most vulnerable – all of which is needed to keep the 1.5°C goal in reach.

    Incorrect

    Answer: A
    Explanation:
    Statement 1 is correct but statement 2 is incorrect: Launched at the first Global Conference on Synergies between the 2030 Agenda and Paris Agreement at Copenhagen, Denmark in April 2019, the Cool Coalition was assembled by United Nations Environment Programme (UNEP) with the belief that a unified effort of governments, cities, businesses, and civil society that share the objective of advancing, sustainable cooling will be more effective in bringing about change than acting alone.
    It is a global multi-stakeholder network that connects a wide range of key actors from government, cities, international organizations, businesses, finance, academia, and civil society groups to facilitate knowledge exchange, advocacy and joint action towards a rapid global transition to efficient and climate-friendly cooling. The Cool Coalition is now working with over 100 partners, including 23 countries. India is also a member of the Cool Coalition.
    Additional information
    The Cool Coalition and COP28 UAE Presidency launched the Global Cooling Pledge at COP28 of the UNFCCC, with nearly 70 government signatories and more than 50 supporters from international organizations, international finance institutions and industry. The Global Cooling Pledge marks the world’s first collective effort to improve energy efficiency and reduce emissions from the cooling sector.
    The Pledge aims to reduce global cooling-related emissions by 68 per cent by 2050, improve energy efficiency of cooling technologies by 50 per cent by 2030 and to increase access to sustainable cooling for the most vulnerable – all of which is needed to keep the 1.5°C goal in reach.

  5. Question 5 of 5
    5. Question

    5. With reference to the ‘Tapirs’, consider the following statements:
    1. Tapirs are guaranteed the same level of protection as tigers in India.
    2. Malayan Tapir is the only tapir species found in South-East Asia.
    3. Mountain Tapir is naturally found in the Western Ghats in India.
    How many of the above statements are correct?

    Correct

    Answer: B
    Explanation:
    Tapir can best be described as resembling a pig but with a long snout, it actually belongs to the family of horses and rhinoceroses. They inhabit parts of Latin America and in Asia are found in Malaysia, Thailand and may wander into India along the border shared with Myanmar.
    Statement 1 is correct: By virtue of being placed in the Schedule I of the Indian Wildlife (Protection) Act, 1972, Tapirs enjoy the highest degree of protection in India just like other endangered species such as tigers and elephants.
    Statement 2 is correct: Malayan Tapir is the largest of the world’s tapirs and the only one found in Asia, predominantly in South East Asia. The other 3 tapir species are known to live in Central and South America. The Malay tapir is listed as ‘Endangered’ by the IUCN due to threats such as habitat loss and fragmentation, hunting, road-kills and by-catches by snare hunters.
    Statement 3 is incorrect: Mountain tapir is not found in the Western Ghats. Also known as woolly tapir, it lives high in the Andes Mountains. Woolly tapirs, named for their warm and protective coat, are the smallest of all tapirs. There are no tapirs in India currently.

    Incorrect

    Answer: B
    Explanation:
    Tapir can best be described as resembling a pig but with a long snout, it actually belongs to the family of horses and rhinoceroses. They inhabit parts of Latin America and in Asia are found in Malaysia, Thailand and may wander into India along the border shared with Myanmar.
    Statement 1 is correct: By virtue of being placed in the Schedule I of the Indian Wildlife (Protection) Act, 1972, Tapirs enjoy the highest degree of protection in India just like other endangered species such as tigers and elephants.
    Statement 2 is correct: Malayan Tapir is the largest of the world’s tapirs and the only one found in Asia, predominantly in South East Asia. The other 3 tapir species are known to live in Central and South America. The Malay tapir is listed as ‘Endangered’ by the IUCN due to threats such as habitat loss and fragmentation, hunting, road-kills and by-catches by snare hunters.
    Statement 3 is incorrect: Mountain tapir is not found in the Western Ghats. Also known as woolly tapir, it lives high in the Andes Mountains. Woolly tapirs, named for their warm and protective coat, are the smallest of all tapirs. There are no tapirs in India currently.

window.wpAdvQuizInitList = window.wpAdvQuizInitList || []; window.wpAdvQuizInitList.push({ id: '#wpAdvQuiz_649', init: { quizId: 649, mode: 0, globalPoints: 10, timelimit: 0, resultsGrade: [0], bo: 0, qpp: 0, catPoints: [10], formPos: 0, lbn: "Finish quiz", json: {"3029":{"type":"single","id":3029,"catId":0,"points":2,"correct":[0,1,0,0]},"3030":{"type":"single","id":3030,"catId":0,"points":2,"correct":[1,0,0,0]},"3031":{"type":"single","id":3031,"catId":0,"points":2,"correct":[0,1,0,0]},"3032":{"type":"single","id":3032,"catId":0,"points":2,"correct":[1,0,0,0]},"3033":{"type":"single","id":3033,"catId":0,"points":2,"correct":[0,1,0,0]}} } });




Day-607 | Daily MCQs | UPSC Prelims | CURRENT AFFAIRS

Day-607

Time limit: 0

Quiz-summary

0 of 5 questions completed

Questions:

  1. 1
  2. 2
  3. 3
  4. 4
  5. 5

Information

DAILY MCQ

You have already completed the quiz before. Hence you can not start it again.

Quiz is loading...

You must sign in or sign up to start the quiz.

You have to finish following quiz, to start this quiz:

Results

0 of 5 questions answered correctly

Your time:

Time has elapsed

You have reached 0 of 0 points, (0)

Categories

  1. Not categorized 0%
  1. 1
  2. 2
  3. 3
  4. 4
  5. 5
  1. Answered
  2. Review
  1. Question 1 of 5
    1. Question

    1. It is now a widely accepted view that the traditional architecture of houses should inspire the architecture of modern times, thanks to their resilience against disasters and extreme weather events. In this context, consider the following pairs:
    Traditional architectures – States associated
    1. Kathkuni – Gujarat
    2. Dhajji-Dewari – Meghalaya
    3. Bhonga houses – Himachal Pradesh
    How many of the above pairs are correctly matched?

    Correct

    Answer. D
    Explanation:
    1. KATHKUNI – Tirthan valley, Himachal Pradesh
    With many typical ancient structures having survived tectonic tremors, the resilience of the kathkuni structures lie in the flexibility of dry masonry and alternate layers of wood beams without any cementing material. The stone plinth rises above the ground level giving strength to the superstructure and also preventing it from snow and ground water.
    The double-skinned walls with the air gap between two parallel wooden beams filled with loose, small stone pieces insulates the structure, keeping it warm in cold weather and cool in summers.

    2. DHAJJI DEWARI, – Kashmir
    It is referred to in the Indian Standard Codes as brick nogged timber frame construction. It is a timber frame into which one layer of masonry is tightly packed to form a wall, resulting in a continuous wall membrane of wood and masonry. The wooden members used here can be imparted horizontally, vertically or inclined into the masonry wall. These wooden members divide the masonry wall, so that the crack does not propagate to the whole wall and ultimately imparts strength to the masonry wall.

    3. BHONGA HOUSES – Kutch region, Gujarat
    A Bhonga consists of a single cylindrically shaped room. The Bhonga has a conical roof supported by cylindrical walls. This type of house is quite durable and appropriate for prevalent desert conditions. Due to its robustness against natural hazards such as earthquakes as well as its pleasant aesthetics, this housing is also known as “Architecture without Architects.”

    Incorrect

    Answer. D
    Explanation:
    1. KATHKUNI – Tirthan valley, Himachal Pradesh
    With many typical ancient structures having survived tectonic tremors, the resilience of the kathkuni structures lie in the flexibility of dry masonry and alternate layers of wood beams without any cementing material. The stone plinth rises above the ground level giving strength to the superstructure and also preventing it from snow and ground water.
    The double-skinned walls with the air gap between two parallel wooden beams filled with loose, small stone pieces insulates the structure, keeping it warm in cold weather and cool in summers.

    2. DHAJJI DEWARI, – Kashmir
    It is referred to in the Indian Standard Codes as brick nogged timber frame construction. It is a timber frame into which one layer of masonry is tightly packed to form a wall, resulting in a continuous wall membrane of wood and masonry. The wooden members used here can be imparted horizontally, vertically or inclined into the masonry wall. These wooden members divide the masonry wall, so that the crack does not propagate to the whole wall and ultimately imparts strength to the masonry wall.

    3. BHONGA HOUSES – Kutch region, Gujarat
    A Bhonga consists of a single cylindrically shaped room. The Bhonga has a conical roof supported by cylindrical walls. This type of house is quite durable and appropriate for prevalent desert conditions. Due to its robustness against natural hazards such as earthquakes as well as its pleasant aesthetics, this housing is also known as “Architecture without Architects.”

  2. Question 2 of 5
    2. Question

    2. “Turning off The Tap” is talked about in which one of the following contexts?

    Correct

    Answer. C
    Explanation: Brought out by the United Nations Environment Programme (UNEP), the Report “Turning off the Tap” proposes systems change to address the causes of plastic pollution, starting by reducing problematic and unnecessary plastic use, redesigning the system, products and their packaging and combining these with a market transformation towards circularity in plastics. This can be achieved by accelerating three key shifts – reorient and diversify, reuse, and recycle, – and actions to deal with the legacy of plastic pollution.

    Incorrect

    Answer. C
    Explanation: Brought out by the United Nations Environment Programme (UNEP), the Report “Turning off the Tap” proposes systems change to address the causes of plastic pollution, starting by reducing problematic and unnecessary plastic use, redesigning the system, products and their packaging and combining these with a market transformation towards circularity in plastics. This can be achieved by accelerating three key shifts – reorient and diversify, reuse, and recycle, – and actions to deal with the legacy of plastic pollution.

  3. Question 3 of 5
    3. Question

    3. With reference to the calculation of Minimum Support Price (MSP), consider the following statements:
    1. The A2+FL cost includes both the actual paid-out costs and imputed value of family labour.
    2. The C2 cost includes only the expenses incurred by farmers on seeds, fertilizers, pesticides, labour, irrigation, and other inputs.
    3. The Swaminathan Commission recommended that the MSP should at least be 50% more than the weighted average of A2+FL cost.
    How many of the above statements are correct?

    Correct

    Answer: A
    Explanation:
    Statement 1 is correct: MSP is the minimum price at which the government procures crops from farmers. The government bases its announcement on the recommendations given by the Commission for Agricultural Costs & Prices (CACP), which details three major formulae to arrive at MSP.
    A2: Costs incurred by the farmer in production of a particular crop. It includes several inputs such as expenditure on seeds, fertilisers, pesticides, leased-in land, hired labour, machinery and fuel.
    A2+FL: Costs incurred by the farmer (actual paid-out costs) and the value of family labour.
    Statement 2 is incorrect:
    C2 is a comprehensive cost, which is A2+FL cost plus imputed rental value of owned land plus interest on fixed capital, rent paid for leased-in land.
    Statement 3 is incorrect: The National Commission of Farmers also known as the Swaminathan Commission recommended that the MSP should at least be 50 per cent more than the weighted average CoP, which it refers to as the C2 cost.

    Incorrect

    Answer: A
    Explanation:
    Statement 1 is correct: MSP is the minimum price at which the government procures crops from farmers. The government bases its announcement on the recommendations given by the Commission for Agricultural Costs & Prices (CACP), which details three major formulae to arrive at MSP.
    A2: Costs incurred by the farmer in production of a particular crop. It includes several inputs such as expenditure on seeds, fertilisers, pesticides, leased-in land, hired labour, machinery and fuel.
    A2+FL: Costs incurred by the farmer (actual paid-out costs) and the value of family labour.
    Statement 2 is incorrect:
    C2 is a comprehensive cost, which is A2+FL cost plus imputed rental value of owned land plus interest on fixed capital, rent paid for leased-in land.
    Statement 3 is incorrect: The National Commission of Farmers also known as the Swaminathan Commission recommended that the MSP should at least be 50 per cent more than the weighted average CoP, which it refers to as the C2 cost.

  4. Question 4 of 5
    4. Question

    4. The term ‘Alpenglow’, often seen in the news, is related to:

    Correct

    Answer-B
    Context: Astronaut Lora O’Hara on the International Space Station captured Alpenglow near the Hindu Kush. This natural phenomenon occurs during sunrise or sunset, casting rosy, reddish, or orange hues on mountain slopes. Alpenglow’s colors vary due to the sun’s angle and atmospheric conditions. Vibrancy depends on sun location, light angle, clouds, humidity, and particulates. The phenomenon’s rapid changes make it a captivating spectacle during the twilight hours.
    Hence, option (b) is the correct answer.

    Incorrect

    Answer-B
    Context: Astronaut Lora O’Hara on the International Space Station captured Alpenglow near the Hindu Kush. This natural phenomenon occurs during sunrise or sunset, casting rosy, reddish, or orange hues on mountain slopes. Alpenglow’s colors vary due to the sun’s angle and atmospheric conditions. Vibrancy depends on sun location, light angle, clouds, humidity, and particulates. The phenomenon’s rapid changes make it a captivating spectacle during the twilight hours.
    Hence, option (b) is the correct answer.

  5. Question 5 of 5
    5. Question

    5. With reference to the United Nations Statistical Commission, consider the following statements:
    1. It is the highest decision-making body for international statistical activities.
    2. It is responsible for setting statistical standards and implementation at international levels.
    3. India has been elected to the UN Statistical Commission for a two-year term in 2024.
    How many of the statements given above are correct?

    Correct

    Answer-B
    Context: The 55th session of the United Nations Statistical Commission is scheduled to be held in New York from 27 February – 1 March 2024. The Statistical Commission consists of 24 UN member states, elected on a consular basis by the UN Economic and Social Council.
    ● The United Nations Statistical Commission, established in 1947, is the highest body of the global statistical system bringing together the Chief Statisticians from member states from around the world.
    The Statistical Commission oversees the work of the United Nations Statistics Division (UNSD), a functional commission of the UN Economic and Social Council.It is the highest decision-making body for international statistical activities, responsible for setting statistical standards and developing concepts and methods, including their implementation at the national and international levels. Hence, statement 1 is correct and statement 2 is correct.
    Statement 3 is incorrect: India has been elected to the UN Statistical Commission for a four-year term beginning on January 1, 2024.

    Incorrect

    Answer-B
    Context: The 55th session of the United Nations Statistical Commission is scheduled to be held in New York from 27 February – 1 March 2024. The Statistical Commission consists of 24 UN member states, elected on a consular basis by the UN Economic and Social Council.
    ● The United Nations Statistical Commission, established in 1947, is the highest body of the global statistical system bringing together the Chief Statisticians from member states from around the world.
    The Statistical Commission oversees the work of the United Nations Statistics Division (UNSD), a functional commission of the UN Economic and Social Council.It is the highest decision-making body for international statistical activities, responsible for setting statistical standards and developing concepts and methods, including their implementation at the national and international levels. Hence, statement 1 is correct and statement 2 is correct.
    Statement 3 is incorrect: India has been elected to the UN Statistical Commission for a four-year term beginning on January 1, 2024.

window.wpAdvQuizInitList = window.wpAdvQuizInitList || []; window.wpAdvQuizInitList.push({ id: '#wpAdvQuiz_648', init: { quizId: 648, mode: 0, globalPoints: 10, timelimit: 0, resultsGrade: [0], bo: 0, qpp: 0, catPoints: [10], formPos: 0, lbn: "Finish quiz", json: {"3024":{"type":"single","id":3024,"catId":0,"points":2,"correct":[0,0,0,1]},"3025":{"type":"single","id":3025,"catId":0,"points":2,"correct":[0,0,1,0]},"3026":{"type":"single","id":3026,"catId":0,"points":2,"correct":[1,0,0,0]},"3027":{"type":"single","id":3027,"catId":0,"points":2,"correct":[0,1,0,0]},"3028":{"type":"single","id":3028,"catId":0,"points":2,"correct":[0,1,0,0]}} } });




Day-606 | Daily MCQs | UPSC Prelims | ECONOMY

Day-606

Time limit: 0

Quiz-summary

0 of 5 questions completed

Questions:

  1. 1
  2. 2
  3. 3
  4. 4
  5. 5

Information

DAILY MCQ

You have already completed the quiz before. Hence you can not start it again.

Quiz is loading...

You must sign in or sign up to start the quiz.

You have to finish following quiz, to start this quiz:

Results

0 of 5 questions answered correctly

Your time:

Time has elapsed

You have reached 0 of 0 points, (0)

Categories

  1. Not categorized 0%
  1. 1
  2. 2
  3. 3
  4. 4
  5. 5
  1. Answered
  2. Review
  1. Question 1 of 5
    1. Question

    1. Consider the following statements:
    Statement-I: ‘Crowding in’ occurs when higher government spending leads to higher private investment.
    Statement-II: Higher government spending can stimulate aggregate demand.
    Which one of the following is correct in respect of the above statements?

    Correct

    Answer: A
    Explanation:
    Both Statement-I and Statement-II are correct and Statement-II is the correct explanation for Statement-I
    Crowding-in refers to the situation where an increase in government spending leads to an increase in private sector investment.
    The mechanism behind it works like this:
    ● A rise in government spending can boost aggregate demand (C+I+G+X-M) which can lead to increased real economic growth, higher employment and higher real incomes.
    ● This, in turn, can increase consumer and business confidence (an improvement in animal spirits) leading to increased private investment in new projects and businesses.
    ● The phenomenon is more likely to occur when the government expenditure is in the form of capital expenditure, targeted at infrastructure projects, etc, that increases the productivity of the economy.
    ● An opposite of this phenomenon is ‘crowding-out’. It occurs when higher government spending results in driving out the private sector from borrowing, thus effectively reducing private investment.
    ● This occurs because higher spending by the government requires the government to borrow from the market. This leaves less funds available for the private sector to borrow. A high demand for funds increases the interest rate in the market. An interest rate can be defined as the cost of the funds. Thus, crowding out of private investment happens.

    Incorrect

    Answer: A
    Explanation:
    Both Statement-I and Statement-II are correct and Statement-II is the correct explanation for Statement-I
    Crowding-in refers to the situation where an increase in government spending leads to an increase in private sector investment.
    The mechanism behind it works like this:
    ● A rise in government spending can boost aggregate demand (C+I+G+X-M) which can lead to increased real economic growth, higher employment and higher real incomes.
    ● This, in turn, can increase consumer and business confidence (an improvement in animal spirits) leading to increased private investment in new projects and businesses.
    ● The phenomenon is more likely to occur when the government expenditure is in the form of capital expenditure, targeted at infrastructure projects, etc, that increases the productivity of the economy.
    ● An opposite of this phenomenon is ‘crowding-out’. It occurs when higher government spending results in driving out the private sector from borrowing, thus effectively reducing private investment.
    ● This occurs because higher spending by the government requires the government to borrow from the market. This leaves less funds available for the private sector to borrow. A high demand for funds increases the interest rate in the market. An interest rate can be defined as the cost of the funds. Thus, crowding out of private investment happens.

  2. Question 2 of 5
    2. Question

    2. With reference to ‘Debentures’, consider the following statements:
    1. An individual’s investment in debentures represents the proportional stake in the company’s ownership.
    2. They are entitled to periodic interest payments at a predetermined rate.
    Which of the statements given above is/are correct?

    Correct

    Answer: B
    Explanation:
    Statement 1 is incorrect: A debenture is a type of a long-term debt instrument, and not equity representing a stake in the company, issued by a company or organisation to raise funds from the public or institutional investors. It is a loan that investors provide to the issuer, typically a corporation or a government entity.
    Statement 2 is correct: A debenture holder is entitled to interest at the fixed rate.
    There exist different types of debentures:
    ● Convertible debentures: They are bonds that can convert into equity shares of the issuing corporation after a specific period. Convertible debentures are hybrid financial products with the benefits of both debt and equity.
    ● Non-convertible debentures: They are traditional debentures that cannot be converted into equity of the issuing corporation. To compensate for the lack of convertibility investors are rewarded with a higher interest rate when compared to convertible debentures.
    ● Redeemable debentures: They are debentures that come with a specific maturity date. The issuer is obligated to repurchase them from debenture holders at face value upon maturity.
    ● Irredeemable debentures: These do not have a fixed maturity rate. They continue indefinitely and the issuer has no obligation to repurchase them.
    ● Registered debenture: It is a debenture for which the issuer maintains a register of debenture holders. These debentures are linked to specific investors.
    ● Unregistered debenture: These debentures do not have a specific record of individual debenture holders.

    Incorrect

    Answer: B
    Explanation:
    Statement 1 is incorrect: A debenture is a type of a long-term debt instrument, and not equity representing a stake in the company, issued by a company or organisation to raise funds from the public or institutional investors. It is a loan that investors provide to the issuer, typically a corporation or a government entity.
    Statement 2 is correct: A debenture holder is entitled to interest at the fixed rate.
    There exist different types of debentures:
    ● Convertible debentures: They are bonds that can convert into equity shares of the issuing corporation after a specific period. Convertible debentures are hybrid financial products with the benefits of both debt and equity.
    ● Non-convertible debentures: They are traditional debentures that cannot be converted into equity of the issuing corporation. To compensate for the lack of convertibility investors are rewarded with a higher interest rate when compared to convertible debentures.
    ● Redeemable debentures: They are debentures that come with a specific maturity date. The issuer is obligated to repurchase them from debenture holders at face value upon maturity.
    ● Irredeemable debentures: These do not have a fixed maturity rate. They continue indefinitely and the issuer has no obligation to repurchase them.
    ● Registered debenture: It is a debenture for which the issuer maintains a register of debenture holders. These debentures are linked to specific investors.
    ● Unregistered debenture: These debentures do not have a specific record of individual debenture holders.

  3. Question 3 of 5
    3. Question

    3. Consider the following statements:
    Statement-I: Direct monetisation of fiscal deficit leads to inflationary pressure in the economy.
    Statement-II: The central bank purchases government securities in the secondary market.
    Which one of the following is correct in respect of the above statements?

    Correct

    Answer: C
    Explanation:
    Statement-I is correct but Statement-II is incorrect
    Direct monetization of deficit refers to a scenario where a central bank prints currency to the tune of accommodating massive deficit spending by the government. The central bank, the RBI in India, does so by purchasing government securities directly in the primary market and not secondary market. Hence, statement 2 is incorrect.
    Direct monetisation of the deficit leads to inflationary pressure in the economy as there is increase in money supply in the economy. Hence, statement 1 is correct.
    Direct monetisation process used to be automatic in India only until 1997, when it was later decided to end this practice by entrusting RBI to conduct such OMOs (Open Market Operations) only in the secondary market.

    Incorrect

    Answer: C
    Explanation:
    Statement-I is correct but Statement-II is incorrect
    Direct monetization of deficit refers to a scenario where a central bank prints currency to the tune of accommodating massive deficit spending by the government. The central bank, the RBI in India, does so by purchasing government securities directly in the primary market and not secondary market. Hence, statement 2 is incorrect.
    Direct monetisation of the deficit leads to inflationary pressure in the economy as there is increase in money supply in the economy. Hence, statement 1 is correct.
    Direct monetisation process used to be automatic in India only until 1997, when it was later decided to end this practice by entrusting RBI to conduct such OMOs (Open Market Operations) only in the secondary market.

  4. Question 4 of 5
    4. Question

    4. With reference to the Female Labour Force Participation Rate (FLFPR) in India in the last five years, consider the following statements:
    1. It has been falling in the urban areas, while it has been rising in the rural areas.
    2. Rise in rural FLFPR is marked by a rise in the share of self-employment and agriculture among working women.
    3. Overall, it has risen to more than 35%.
    How many of the above statements are correct?

    Correct

    Answer: B
    Explanation:
    Statement 1 is incorrect: The government has taken various steps to improve women’s participation in the labour force and quality of their employment. Female Labour Force Participation Rate (FLFPR) has been rising for at least six years now. While urban FLFPR has also been rising, the rural FLFPR has seen a sharp growth. Hence, both the urban as well as rural areas have registered an increase in participation.
    Statement 2 is correct: The rise in rural female LFPR has been accompanied by a rise in the share of self-employment and agriculture among working women. The rise in rural female employment has been contributed by both own account worker/employer category (share rising from 19 per cent in 2017-18 to 27.9 percent in 2022-23) and the unpaid helper category (share rising from 38.7 percent to 43.1 per cent, which is a relatively smaller rise), indicating a rising contribution of females to rural production.
    Statement 3 is correct: Overall, FLFPR rose from 23.3 per cent in 2017-18 to 37 per cent in 2022-23.

    Incorrect

    Answer: B
    Explanation:
    Statement 1 is incorrect: The government has taken various steps to improve women’s participation in the labour force and quality of their employment. Female Labour Force Participation Rate (FLFPR) has been rising for at least six years now. While urban FLFPR has also been rising, the rural FLFPR has seen a sharp growth. Hence, both the urban as well as rural areas have registered an increase in participation.
    Statement 2 is correct: The rise in rural female LFPR has been accompanied by a rise in the share of self-employment and agriculture among working women. The rise in rural female employment has been contributed by both own account worker/employer category (share rising from 19 per cent in 2017-18 to 27.9 percent in 2022-23) and the unpaid helper category (share rising from 38.7 percent to 43.1 per cent, which is a relatively smaller rise), indicating a rising contribution of females to rural production.
    Statement 3 is correct: Overall, FLFPR rose from 23.3 per cent in 2017-18 to 37 per cent in 2022-23.

  5. Question 5 of 5
    5. Question

    5. Consider the following statements:
    Statement-I: India is less vulnerable to the sovereign debt crisis.
    Statement-II: US dollar-denominated debt remained the largest component of India’s external debt.
    Which one of the following is correct in respect of the above statements?

    Correct

    Answer: B
    Explanation:
    Both Statement-I and Statement-II are correct and Statement-II is not the correct explanation for Statement-I
    Statement 1 is correct: India is less vulnerable to the sovereign debt crisis.
    ● Sovereign default or debt crisis occurs when a country fails to repay its debt, or its debt-to-GDP ratio becomes unsustainably high.
    ● As stated by the Finance Ministry, India’s public debt-to-GDP ratio has barely increased from 81% in 2005-06 to 84% in 2021-22, and is back to 81% in 2022-23.
    ● According to a recent IMF report (International Monetary Fund), India’s general government debt, including the Centre and States, could be 100% of GDP under adverse circumstances by fiscal 2028. This created a controversy, with the government refuting the IMF projections as “a worst-case scenario and is not fait accompli”.
    ● According to the Reserve Bank of India’s (RBI) monthly bulletin, India’s debt-to-GDP ratio could decline to 73.4% by 2030-31 from an estimated 81.6% in 2023-24.
    ● On the external debt front, at end-September 2023, India’s external debt was placed at $635.3 billion, recording an increase of $ 6.4 billion over its level at end-June 2023.
    ● The external debt to GDP ratio stood at 18.61% as at end-September 2023, as against 18.58% as at end-June 2023. It was 20% in 2022.
    ● In a 2020 research paper, Reserve Bank of India (RBI) economists noted that India’s external vulnerability remained minimal. In a 2022 note, RBI economists said the threshold level of external debt is 23-24% of GDP, implying India could raise growth maximizing external debt.

    Statement 2 is correct: The US dollar denominated debt remained the largest component of India’s external debt with a share of 54.7% at end-September 2023, followed by Indian Rupee (30.5%), SDR (5.7%), Japanese Yen (5.6%), and Euro (2.9%).

    Incorrect

    Answer: B
    Explanation:
    Both Statement-I and Statement-II are correct and Statement-II is not the correct explanation for Statement-I
    Statement 1 is correct: India is less vulnerable to the sovereign debt crisis.
    ● Sovereign default or debt crisis occurs when a country fails to repay its debt, or its debt-to-GDP ratio becomes unsustainably high.
    ● As stated by the Finance Ministry, India’s public debt-to-GDP ratio has barely increased from 81% in 2005-06 to 84% in 2021-22, and is back to 81% in 2022-23.
    ● According to a recent IMF report (International Monetary Fund), India’s general government debt, including the Centre and States, could be 100% of GDP under adverse circumstances by fiscal 2028. This created a controversy, with the government refuting the IMF projections as “a worst-case scenario and is not fait accompli”.
    ● According to the Reserve Bank of India’s (RBI) monthly bulletin, India’s debt-to-GDP ratio could decline to 73.4% by 2030-31 from an estimated 81.6% in 2023-24.
    ● On the external debt front, at end-September 2023, India’s external debt was placed at $635.3 billion, recording an increase of $ 6.4 billion over its level at end-June 2023.
    ● The external debt to GDP ratio stood at 18.61% as at end-September 2023, as against 18.58% as at end-June 2023. It was 20% in 2022.
    ● In a 2020 research paper, Reserve Bank of India (RBI) economists noted that India’s external vulnerability remained minimal. In a 2022 note, RBI economists said the threshold level of external debt is 23-24% of GDP, implying India could raise growth maximizing external debt.

    Statement 2 is correct: The US dollar denominated debt remained the largest component of India’s external debt with a share of 54.7% at end-September 2023, followed by Indian Rupee (30.5%), SDR (5.7%), Japanese Yen (5.6%), and Euro (2.9%).

window.wpAdvQuizInitList = window.wpAdvQuizInitList || []; window.wpAdvQuizInitList.push({ id: '#wpAdvQuiz_647', init: { quizId: 647, mode: 0, globalPoints: 10, timelimit: 0, resultsGrade: [0], bo: 0, qpp: 0, catPoints: [10], formPos: 0, lbn: "Finish quiz", json: {"3019":{"type":"single","id":3019,"catId":0,"points":2,"correct":[1,0,0,0]},"3020":{"type":"single","id":3020,"catId":0,"points":2,"correct":[0,1,0,0]},"3021":{"type":"single","id":3021,"catId":0,"points":2,"correct":[0,0,1,0]},"3022":{"type":"single","id":3022,"catId":0,"points":2,"correct":[0,1,0,0]},"3023":{"type":"single","id":3023,"catId":0,"points":2,"correct":[0,1,0,0]}} } });




Day-605 | Daily MCQs | UPSC Prelims | GEOGRAPHY

Day-605

Time limit: 0

Quiz-summary

0 of 5 questions completed

Questions:

  1. 1
  2. 2
  3. 3
  4. 4
  5. 5

Information

DAILY MCQ

You have already completed the quiz before. Hence you can not start it again.

Quiz is loading...

You must sign in or sign up to start the quiz.

You have to finish following quiz, to start this quiz:

Results

0 of 5 questions answered correctly

Your time:

Time has elapsed

You have reached 0 of 0 points, (0)

Categories

  1. Not categorized 0%
  1. 1
  2. 2
  3. 3
  4. 4
  5. 5
  1. Answered
  2. Review
  1. Question 1 of 5
    1. Question

    1. Consider the following pairs:
    International borders – Countries sharing these borders
    1. 49th Parallel – North Korea and South Korea
    2. Mannerheim Line – Russia and Ukraine
    3. Blue Line – Israel and Lebanon
    4. Green Line – Republic of Cyprus and Turkish Cyprus
    How many of the above pairs are correctly matched?

    Correct

    Answer: B
    Explanation:
    Pair 1 is matched incorrectly:
    The 49th Parallel is the border shared between the USA and Canada.
    This open border between two countries has been in news due to the influx of illegal refugees into the USA through Canada.
    The border between South Korea and North Korea is known as the 38th Parallel.
    Pair 2 is matched incorrectly:
    The Mannerheim Line is the border between Russia and Finland.
    The border has been in the news since Finland joined the North Atlantic Treaty Organisation (NATO).
    Pair 3 is matched correctly:
    The Blue Line is the border between Israel and Lebanon.
    It has been in the news due to ongoing conflict between Hezbollah and Israel.
    Pair 4 is matched correctly:
    Border between the Republic of Cyprus and Turkish Cyprus is known as the Green Line.

    Incorrect

    Answer: B
    Explanation:
    Pair 1 is matched incorrectly:
    The 49th Parallel is the border shared between the USA and Canada.
    This open border between two countries has been in news due to the influx of illegal refugees into the USA through Canada.
    The border between South Korea and North Korea is known as the 38th Parallel.
    Pair 2 is matched incorrectly:
    The Mannerheim Line is the border between Russia and Finland.
    The border has been in the news since Finland joined the North Atlantic Treaty Organisation (NATO).
    Pair 3 is matched correctly:
    The Blue Line is the border between Israel and Lebanon.
    It has been in the news due to ongoing conflict between Hezbollah and Israel.
    Pair 4 is matched correctly:
    Border between the Republic of Cyprus and Turkish Cyprus is known as the Green Line.

  2. Question 2 of 5
    2. Question

    2. Consider the following countries:
    1. Myanmar
    2. Laos
    3. Thailand
    4. Cambodia
    5. Vietnam
    6. China
    How many of the above countries are part of the Lancang-Mekong Cooperation?

    Correct

    Answer: D
    Explanation:
    Lancang-Mekong Cooperation:
    Lancang-Mekong Cooperation is a multilateral format established in 2016 for cooperation between the riparian states of the Lancang River and Mekong River.
    The Lancang is the part of the Mekong that flows through China.
    Cambodia, Laos, Myanmar, Vietnam and Thailand are five downstream countries of the Mekong River.
    The central purpose of the format is for China to manage water flow from its hydropower dams with the other riparian states.
    LMC Special Fund was created in 2016 to aid in small and medium-sized projects by the Lancang-Mekong countries.

    Incorrect

    Answer: D
    Explanation:
    Lancang-Mekong Cooperation:
    Lancang-Mekong Cooperation is a multilateral format established in 2016 for cooperation between the riparian states of the Lancang River and Mekong River.
    The Lancang is the part of the Mekong that flows through China.
    Cambodia, Laos, Myanmar, Vietnam and Thailand are five downstream countries of the Mekong River.
    The central purpose of the format is for China to manage water flow from its hydropower dams with the other riparian states.
    LMC Special Fund was created in 2016 to aid in small and medium-sized projects by the Lancang-Mekong countries.

  3. Question 3 of 5
    3. Question

    3. Consider the following pairs:
    Islands in news – Countries
    1. Canary Islands – Portugal
    2. Snake Island – Ukraine
    3. Macquarie Islands – Australia
    4. Orkney Islands – United Kingdom
    How many of the above pairs are correctly matched?

    Correct

    Answer: C
    Explanation:
    Pair 1 is matched incorrectly:
    The Canary Islands are part of Spain in the Atlantic Ocean, situated 115 km west of the Moroccan coast.
    It is a volcanic island.
    It has been in the news due to the influx of illegal refugees from conflict-ridden areas of Africa and Asia.
    Pair 2 is matched correctly:
    Snake or Serpent Island, also known as Zmiinyi Island is a small piece of rock less than 700 metres from end to end, that has been described as being X-shaped.
    Location:
    It is located 35 km from the coast in the Black Sea, to the east of the mouth of the Danube and roughly southwest of the port city of Odessa.
    The island is marked on the map by the tiny village of Bile that is located on it, belongs to Ukraine.

    Pair 3 is matched correctly:
    Macquarie Island is located around 1,500km southeast of Tasmania which belongs to Australia.
    It is the exposed crest of the 1,600 km-long undersea Macquarie Ridge.
    This is the only piece of land in the world formed entirely of oceanic crust.
    The area is also designated as a World Heritage Area.
    It has been in the news because the Federal government of Australia proposed the expansion of the marine park covering Macquarie Island’s Economic Exclusion Zone.
    Pair 4 is matched correctly:
    It is an archipelago consisting of 70 individual islands, of which only 20 are inhabited.
    The Orkney Islands can be found roughly 10 miles off the north coast of Scotland (which is part of the United Kingdom).
    The islands have been inhabited since prehistoric times and are home to numerous archaeological sites, including Neolithic stone circles, chambered tombs (such as Maeshowe).
    The four monuments that make up the Heart of Neolithic Orkney are unquestionably among the most important Neolithic sites in Western Europe.
    These are the Ring of Brodgar, Stones of Stenness, Maeshowe and Skara Brae.
    The Heart of Neolithic Orkney is designated as a UNESCO world Heritage site.
    It has been in news because Orkney Islands is looking at ways to split off from the U.K. and potentially become a self-governing territory of Norway.

    Incorrect

    Answer: C
    Explanation:
    Pair 1 is matched incorrectly:
    The Canary Islands are part of Spain in the Atlantic Ocean, situated 115 km west of the Moroccan coast.
    It is a volcanic island.
    It has been in the news due to the influx of illegal refugees from conflict-ridden areas of Africa and Asia.
    Pair 2 is matched correctly:
    Snake or Serpent Island, also known as Zmiinyi Island is a small piece of rock less than 700 metres from end to end, that has been described as being X-shaped.
    Location:
    It is located 35 km from the coast in the Black Sea, to the east of the mouth of the Danube and roughly southwest of the port city of Odessa.
    The island is marked on the map by the tiny village of Bile that is located on it, belongs to Ukraine.

    Pair 3 is matched correctly:
    Macquarie Island is located around 1,500km southeast of Tasmania which belongs to Australia.
    It is the exposed crest of the 1,600 km-long undersea Macquarie Ridge.
    This is the only piece of land in the world formed entirely of oceanic crust.
    The area is also designated as a World Heritage Area.
    It has been in the news because the Federal government of Australia proposed the expansion of the marine park covering Macquarie Island’s Economic Exclusion Zone.
    Pair 4 is matched correctly:
    It is an archipelago consisting of 70 individual islands, of which only 20 are inhabited.
    The Orkney Islands can be found roughly 10 miles off the north coast of Scotland (which is part of the United Kingdom).
    The islands have been inhabited since prehistoric times and are home to numerous archaeological sites, including Neolithic stone circles, chambered tombs (such as Maeshowe).
    The four monuments that make up the Heart of Neolithic Orkney are unquestionably among the most important Neolithic sites in Western Europe.
    These are the Ring of Brodgar, Stones of Stenness, Maeshowe and Skara Brae.
    The Heart of Neolithic Orkney is designated as a UNESCO world Heritage site.
    It has been in news because Orkney Islands is looking at ways to split off from the U.K. and potentially become a self-governing territory of Norway.

  4. Question 4 of 5
    4. Question

    4. Consider the following statements about ocean relief:
    1. A seamount never reaches the sea surface like guyots.
    2. Ocean trenches are a result of tectonic activity, which describes the movement of the Earth’s lithosphere.
    3. The continental slope is the seaward extension of the continent from the shoreline to the continental edge.
    How many of the above statements are correct?

    Correct

    Answer: C
    Explanation: All statements are correct
    Statement 1 is correct:
    Seamount:
    It is a mountain with pointed summits, rising from the seafloor that does not reach the surface of the ocean. Seamounts are volcanic in origin.
    These can be 3,000-4,500 m tall. The Emperor seamount, an extension of the Hawaiian Islands in the Pacific Ocean, is a good example.

    Guyots:
    It is a flat topped seamount. They show evidence of gradual subsidence through stages to become flat topped submerged mountains.
    They develop under the sea surface.
    It is estimated that more than 10,000 seamounts and guyots exist in the Pacific Ocean alone.

    Statement 2 is correct:
    Ocean trenches are long, narrow depressions on the seafloor.
    These chasms are the deepest parts of the ocean—and some of the deepest natural spots on Earth.
    Ocean trenches are a result of tectonic activity, which describes the movement of the Earth’s lithosphere.
    In particular, ocean trenches are a feature of convergent plate boundaries, where two or more tectonic plates meet.
    At many convergent plate boundaries, dense lithosphere melts or slides beneath less-dense lithosphere in a process called subduction, creating a trench.

    Statement 3 is correct:
    The continental shelf is the seaward extension of the continent from the shoreline to the continental edge. Whereas, the continental slope connects the continental shelf and continental rise.
    Continental shelf ends where there is a sharp increase in the slope angle.

    Incorrect

    Answer: C
    Explanation: All statements are correct
    Statement 1 is correct:
    Seamount:
    It is a mountain with pointed summits, rising from the seafloor that does not reach the surface of the ocean. Seamounts are volcanic in origin.
    These can be 3,000-4,500 m tall. The Emperor seamount, an extension of the Hawaiian Islands in the Pacific Ocean, is a good example.

    Guyots:
    It is a flat topped seamount. They show evidence of gradual subsidence through stages to become flat topped submerged mountains.
    They develop under the sea surface.
    It is estimated that more than 10,000 seamounts and guyots exist in the Pacific Ocean alone.

    Statement 2 is correct:
    Ocean trenches are long, narrow depressions on the seafloor.
    These chasms are the deepest parts of the ocean—and some of the deepest natural spots on Earth.
    Ocean trenches are a result of tectonic activity, which describes the movement of the Earth’s lithosphere.
    In particular, ocean trenches are a feature of convergent plate boundaries, where two or more tectonic plates meet.
    At many convergent plate boundaries, dense lithosphere melts or slides beneath less-dense lithosphere in a process called subduction, creating a trench.

    Statement 3 is correct:
    The continental shelf is the seaward extension of the continent from the shoreline to the continental edge. Whereas, the continental slope connects the continental shelf and continental rise.
    Continental shelf ends where there is a sharp increase in the slope angle.

  5. Question 5 of 5
    5. Question

    5. Consider the following statements:
    Statement-I: Jammu and Kashmir has emerged as the powerhouse of ‘Purple Revolution’ in India.
    Statement-II: Lavender cultivation is suitable for temperate regions.
    Which one of the following is correct in respect of the above statements?

    Correct

    Answer: A
    Explanation: Both Statement-I and Statement-II are correct and Statement-II is the correct explanation for Statement-I
    Statement-I is correct:
    The Purple Revolution or Lavender Revolution, launched by the Ministry of Science & Technology, aims to promote the indigenous aromatic crop-based agro economy through the ‘aroma mission’ of the Council of Scientific and Industrial Research (CSIR).

    Aim: The mission aims to increase the income of the farmers and promote lavender cultivation on a commercial scale.
    Lavender oil, which sells for, at least, Rs. 10,000 per liter, is the main commodity. Other popular products include medicines, incense sticks, soaps, and air fresheners.
    The cultivation of lavender is very cost-effective as it yields revenue immediately.
    Under the One District One Product-Districts as Export Hubs (ODOP-DEH) initiative, lavender cultivation in Jammu and Kashmir has experienced a significant boom. Lavender has been designated by the central government as a “Doda brand product” to promote the rare aromatic plant and boost the morale of farmers, entrepreneurs, and agribusinesses involved in its cultivation as part of this Aroma Mission.
    Statement-II is correct and also the correct explanation for Statement-I
    Jammu and Kashmir climatic conditions are conducive to lavender cultivation, since the aromatic plant can withstand both chilly winters and pleasant summers.
    Apart from the requirement of temperate climatic conditions, efforts from CSIR have made it possible that Jammu and Kashmir has emerged as a top destination.
    Jammu and Kashmir climatic conditions are conducive to lavender cultivation, since the aromatic plant can withstand both chilly winters and pleasant summers.
    Additionally, it is a low maintenance crop, which can be used from its second year of plantation and blossoms for fifteen years.
    In its entirety, lavender production gives better returns when compared to other traditional crops.
    CSIR has developed an elite variety of lavender suitable for cultivation in temperate regions of J&K and provided free saplings and end-to-end agro-technologies to farmers and also installed distillation units for essential oil extraction in several regions of J&K.

    Incorrect

    Answer: A
    Explanation: Both Statement-I and Statement-II are correct and Statement-II is the correct explanation for Statement-I
    Statement-I is correct:
    The Purple Revolution or Lavender Revolution, launched by the Ministry of Science & Technology, aims to promote the indigenous aromatic crop-based agro economy through the ‘aroma mission’ of the Council of Scientific and Industrial Research (CSIR).

    Aim: The mission aims to increase the income of the farmers and promote lavender cultivation on a commercial scale.
    Lavender oil, which sells for, at least, Rs. 10,000 per liter, is the main commodity. Other popular products include medicines, incense sticks, soaps, and air fresheners.
    The cultivation of lavender is very cost-effective as it yields revenue immediately.
    Under the One District One Product-Districts as Export Hubs (ODOP-DEH) initiative, lavender cultivation in Jammu and Kashmir has experienced a significant boom. Lavender has been designated by the central government as a “Doda brand product” to promote the rare aromatic plant and boost the morale of farmers, entrepreneurs, and agribusinesses involved in its cultivation as part of this Aroma Mission.
    Statement-II is correct and also the correct explanation for Statement-I
    Jammu and Kashmir climatic conditions are conducive to lavender cultivation, since the aromatic plant can withstand both chilly winters and pleasant summers.
    Apart from the requirement of temperate climatic conditions, efforts from CSIR have made it possible that Jammu and Kashmir has emerged as a top destination.
    Jammu and Kashmir climatic conditions are conducive to lavender cultivation, since the aromatic plant can withstand both chilly winters and pleasant summers.
    Additionally, it is a low maintenance crop, which can be used from its second year of plantation and blossoms for fifteen years.
    In its entirety, lavender production gives better returns when compared to other traditional crops.
    CSIR has developed an elite variety of lavender suitable for cultivation in temperate regions of J&K and provided free saplings and end-to-end agro-technologies to farmers and also installed distillation units for essential oil extraction in several regions of J&K.

window.wpAdvQuizInitList = window.wpAdvQuizInitList || []; window.wpAdvQuizInitList.push({ id: '#wpAdvQuiz_646', init: { quizId: 646, mode: 0, globalPoints: 10, timelimit: 0, resultsGrade: [0], bo: 0, qpp: 0, catPoints: [10], formPos: 0, lbn: "Finish quiz", json: {"3014":{"type":"single","id":3014,"catId":0,"points":2,"correct":[0,1,0,0]},"3015":{"type":"single","id":3015,"catId":0,"points":2,"correct":[0,0,0,1]},"3016":{"type":"single","id":3016,"catId":0,"points":2,"correct":[0,0,1,0]},"3017":{"type":"single","id":3017,"catId":0,"points":2,"correct":[0,0,1,0]},"3018":{"type":"single","id":3018,"catId":0,"points":2,"correct":[1,0,0,0]}} } });